комплексные числа

advertisement
Министерство транспорта и связи Украины
Государственная администрация связи
ОДЕССКАЯ НАЦИОНАЛЬНАЯ АКАДЕМИЯ СВЯЗИ им. А.С. ПОПОВА
Кафедра высшей математики
КОМПЛЕКСНЫЕ ЧИСЛА
Учебное пособие
для иностранных студентов
технических специальностей
Одесса 2009
2
Авторы: И.В. Стрелковская, А.Г. Буслаев, В.Н. Паскаленко
В учебном пособии изложены первоначальные теоретические сведения по
теме «Комплексные числа». Рассмотрены алгебраическая, тригонометрическая
и показательная форма комплексных чисел, действия над комплексными числами, заданными в разных формах, формулы перевода комплексных чисел из
одной формы в другую. Разобраны примеры. Имеется задание для лабораторной работы по теме «Комплексные числа», тесты, задачи для самостоятельного
решения, вопросы для самопроверки, тренировочные упражнения. Предназначено для студентов технических специальностей, является введением в дисциплину «Теория функций комплексного переменного».
УТВЕРЖДЕНО
на заседании кафедры высшей математике
и рекомендовано к изданию
Протокол № 2 от 15.09.09
3
СОДЕРЖАНИЕ
Глава I КОМПЛЕКСНЫЕ ЧИСЛА
1. Основные понятия…………………………………………………................. 5
1.1 Понятие мнимого числа………………………………………… ……. 5
1.2 Понятие комплексного числа……………………………………….
6
1.3 Геометрическое изображение мнимых и комплексных чисел…… 7
1.4 Примеры………………………………………………..…………….. 11
2. Действия над комплексными числами, заданными в алгебраической
форме…………………………………………………………………………. 18
2.1 Алгебраическая форма комплексного числа……………………… 18
2.3 Вычитание комплексных чисел…………………………………….. 18
2.4 Умножение комплексных чисел……………………………………. 19
2.5 Деление комплексных чисел………………………………………... 20
2.6 Примеры………………………………………………………………. 21
3. Действия над комплексными числами, заданными в тригонометрической форме………………………………………………………………… 28
3.1 Тригонометрическая форма комплексного числа………………… 28
3.2 Умножение комплексных чисел……………………………………. 28
3.3 Возведение в степень комплексных чисел. Первая формула
Муавра………………………………………………………………... 28
3.4 Деление комплексных чисел……………………………………….. 30
3.5 Извлечение корня из комплексного числа. Вторая формула
Муавра………………………………………………………………... 30
3.6 Примеры……………………………………………………………… 31
4. Действия над комплексными числами, заданными в показательной
форме…………………………………………………………………………. 36
4.1 Показательная форма комплексного числа……………………….. 36
4.2 Умножение комплексных чисел…………………………………… 36
4.3 Возведение в степень комплексных чисел………………………… 37
4.4 Деление комплексных чисел……………………………………….. 37
4.5 Извлечение корня из комплексного числа………………………… 37
4.6 Формулы Эйлера…………………………………………………….. 38
4.7 Примеры……………………………………………………………… 38
5. Векторная характеристика операций над комплексными числами……..
46
5.1 Сложение комплексных чисел……………………………………… 46
5.2 Вычитание комплексных чисел…………………………………….. 46
5.3 Умножение комплексных чисел…………………………………… 47
5.4 Деление комплексных чисел……………………………………….. 49
5.5 Примеры……………………………………………………………… 49
6. Некоторые приложения комплексных чисел в теории электрических
цепей…………………………………………………………………………… 52
4
6.1 Электромагнитные процессы, описываемые синусоидными
функциями……………………………………………………………. 52
6.2 Основные понятия о методе комплексных амплитуд…………….. 53
Контрольные вопросы …………………………………………………………. 56
Тренировочные упражнения……………………………………………… ……..59
Ответы к тренировочным упражнениям……………………………………… 65
Лабораторная работа…………………………………………………………… 67
Проверочные тесты…………………………………………………………….. 74
Задачи для самостоятельного решения………………………………………. 77
Ответы к задачам для самостоятельного решения...………………………… 87
Список рекомендованной литературы…………………………………………. 100
5
Глава I
КОМПЛЕКСНЫЕ ЧИСЛА
1 ОСНОВНЫЕ ПОНЯТИЯ
В процессе изучения математики происходит последовательное знакомство с числами. Так, сначала рассматривается множество N натуральных чисел,
затем множество Z целых чисел, множество Q рациональных чисел и, наконец,
множество R действительных чисел.
Часто возникают ситуации, когда не существует решений уравнения на
множестве действительных чисел. Так, на множестве R не имеет решений квадратное уравнение x 2 + 1 = 0 . Следовательно, возникает необходимость введения
такого вида чисел, который расширяет множество действительных чисел. С
этой целью ознакомимся с множеством мнимых чисел, а затем с комплексными
числами.
1.1 Понятие мнимого числа
О п р е д е л е н и е. Число, квадрат которого равняется –1, называется
мнимой единицей.
Мнимая единица обозначается символом і. Как следует из определения,
мнимая единица удовлетворяет условию
і2 = –1.
(1.1)
О п р е д е л е н и е. Числа, которые имеют вид bi, где b – вещественное
число, называются мнимыми числами.
1
Например, числа 2і; − i ; 5,3і являются мнимыми.
3
Между мнимыми и действительными числами существует взаимно однозначное соответствие, поскольку каждому действительному числу b может
быть поставлено в соответствие одно мнимое число bi.
ЗАМЕЧАНИЕ. В физике и ее приложениях, где символом і обозначается
сила электрического тока, для обозначения мнимой единицы используют символ j.
О п р е д е л е н и е. Два мнимых числа b1i и b2i называются равными
между собой, если b1 = b2.
О п р е д е л е н и е. Мнимое число (– bi) называется противоположным
мнимому числу bi.
1
Например, число bі является противоположным числу (–6і), а число − і
3
1
– противоположно числу і.
3
6
Операции сложения, вычитания, умножения и деления, введенные для
вещественных чисел, распространяются и на мнимые числа. О возведении
мнимой единицы в натуральную степень идет речь в следующей теореме.
Теорема. Любая натуральная степень числа і равняется одному из
чисел: 1, і, −1, −і .
Д о к а з а т е л ь с т в о.
Рассмотрим выражение іn, где n – натуральное число. Возможны четыре
случая:
где k = 1, 2, …;
2) n = 4k + 1, где k = 0, 1, 2, …;
1) n = 4k,
3) n = 4k + 2, где k = 0, 1, 2, …;
4) n = 4k + 3, где k = 0, 1, 2, … .
n
4k
4 k
k
Если n = 4k,
то і = i = (i ) = 1 = 1;
Если n = 4k + 1, то і n = i 4k + 1 = i 4k i = i;
Если n = 4k + 2, то і n = i 4k + 2 = i 4k i 2 = –1;
Если n = 4k + 3, то і n = i 4k + 3 = i 4k i 3 = i 2 i = –i.
1.2 Понятие комплексного числа
О п р е д е л е н и е 1. Комплексным числом называется число вида
а + bi, где а и b – вещественные числа, i – мнимая единица. Число а называется
действительной частью комплексного числа, число b – его мнимой частью.
Комплексные числа обычно обозначают одной буквой, чаще всего используют букву z:
z = а + bi.
(1.2)
Символически действительную и мнимую части комплексного числа z
обозначают так : а = Re z, b = Im z. В основе этих обозначений использованы
первые буквы латинских слов Realis, что означает „действительный” и
Imaginaris, что означает „мнимый”.
ЗАМЕЧАНИЕ. Иногда мнимой частью комплексного числа z = а + bi называют число bi.
Наравне с определением 1 комплексного числа используют и такое определение.
О п р е д е л е н и е 2. Комплексным числом называется упорядоченная
пара действительных чисел а и b.
Комплексное число в таком случае символически обозначается как
z=(а; b).
Следует обратить внимание на то, что комплексные числа, в отличие от
действительных чисел, не используются для измерений, сравнений и т.д. Комплексные числа – это новый класс чисел, о свойствах которого будет идти речь
далее.
О п р е д е л е н и е. Два комплексных числа z1 = a1 + b1i и z2 = a2 + b2i
называются равными между собой, если Re z1 = Re z2; Im z1 = Im z2.
Для комплексных чисел не существует понятий «больше» или «меньше»,
то есть комплексные числа не сравнимы между собой.
7
Комплексное число z = а + bi считается действительным числом а, если
Im z = 0, и мнимым числом bі, если Re z = 0.
Комплексное число z = а + bi считается нулем, если Re z = 0 и Im z = 0.
О п р е д е л е н и е. Комплексное число (–а – bi) называется противоположным комплексному числу а + bi.
О п р е д е л е н и е. Два комплексных числа, у которых действительные
части являются равными между собой, а мнимые части являются противоположными друг другу, называются комплексно-сопряжёнными числами и обозначаются соответственно как z = а + bi и z = а – bi.
ЗАМЕЧАНИЕ. В некоторых электротехнических дисциплинах число, комплексно-сопряжённое числу z, иногда обозначают символом z .
О п р е д е л е н и е. Модулем комплексного числа z = а + bi называется
неотрицательное число r = |z|, определяемое формулой
r = a 2 + b2 .
Модуль любого комплексного числа определяется однозначно. Комплексно-сопряжённые числа имеют равные модули
|z| =
a 2 + b2 ;
| z | = a 2 + ( −b ) 2 = a 2 + b 2 = | z | .
Следовательно,
| z | = a 2 + b2
(1.3)
| z | = | z |.
(1.4)
и
1.3 Геометрическое изображение мнимых и комплексных чисел
Комплексное число, согласно определению 2, – это упорядоченная пара
действительных чисел. Каждой упорядоченной паре действительных чисел, то
есть каждому комплексному числу, отвечает определенная точка на плоскости.
Плоскость хОу, используемая для изображения комплексных чисел, называется
комплексной числовой плоскостью и обозначается символом С. Буква О обозначает начало системы координат. Положение точки О отвечает нулевой отметке на осях координат Ох и Оу.
Пусть точка М(а; b) – любая точка на комплексной плоскости. Эта точка
является геометрической интерпретацией комплексного числа z = (a;b) и
z = a + bi. На комплексной плоскости символы z и (а; b) означают одно и то же.
Точка М (а; b) называется аффиксом комплексного числа z = а + bi, а комплексное число z называется комплексной координатой точки М.
8
Следовательно, между точками комплексной плоскости и множеством
комплексных чисел существует взаимно однозначное соответствие.
Как отмечалось ранее, частными случаями комплексных чисел z = а + bi
являются действительные числа z = а, если b = 0 и мнимые числа z = bi, если
а = 0. Это значит, что множество действительных чисел а и множество мнимых
чисел bi являются частными случаями множества комплексных чисел. Поскольку геометрической интерпретацией действительнных чисел является
множество точек (а; 0), то действительные числа изображаются точками на оси
Ох, которая на комплексной числовой плоскости называется действительной
осью и обозначается Ох или Re z. Геометрической интерпретацией мнимых чисел является множество точек (0; b). Следовательно, мнимые числа изображаются точками на оси Оу, которая на комплексной числовой плоскости называется мнимой осью и обозначается Оу или Im z (рис. 1.1).
Каждой точке M(а; b) на комплексной числовой плоскости отвечает ее
радиус-вектор ОМ . Выходит, что множество комплексных чисел находится во
взаимно однозначном соответствии с множеством всех радиус-векторов на
комплексной числовой плоскости. При этом, если комплексное число z = а + bi
является комплексной координатой точки M(а; b), то комплексное число
z = а + bi называется комплексной координатой радиус-вектора точки
M(а; b), а сам вектор ОМ называется комплексным вектором. Иногда в электротехнических дисциплинах комплексные векторы обозначаются символом
•
. Следовательно, комплексная координата радиус-вектора точки равняется
ÎÌ
комплексной координате этой точки.
Im z
Обратимся к рис. 1.1. Рассмотрим
M (a; b)
b
прямоугольный треугольник ОАМ. В этом
r
треугольнике |OA| = а, |MA| = b. Найдем гипотенузу |OM|:
ϕ
О
A
a
Re z
|OM| =
Выходит, что:
a 2 + b2 .
1) проекции вектора ОМ на действительную и мнимую оси равняются соответ–b
M1(а; –b)
ственно действительной и мнимой частям
комплексной координаты этого вектора, то
Рисунок 1.1 − Геометрическое
uuuur r
uuuur r
изображения комплексных чисел есть пр Rez OM = a пр Imz OM = b .
9
2) модуль радиус-вектора ОМ совпадает с модулем комплексного числа
z, то есть | ОМ | = |z| = r.
Следовательно, модуль комплексного числа равняется расстоянию от точки О(0; 0) комплексной числовой плоскости до аффикса комплексного числа.
Если некоторый вектор
М 1М 2
равняется комплексному вектору
ОМ ={а;b}, то вектор М 1М 2 также называется комплексным вектором с ком-
плексной координатой z = а + bi.
О п р е д е л е н и е. Аргументом комплексного числа z = а + bi (z ≠ 0)
называется угол ϕ между положительным направлением оси Ох и радиусвектором комплексного числа z.
Аргумент комплексного числа называется положительным, если угол отсчитываетсятся от оси Ох в направлении против часовой стрелки, и отрицательным, если угол отсчитывается от оси Ох в направлении по часовой стрелке.
Для обозначения аргумента используется символ Arg z, то есть
ϕ = Arg z.
(1.5)
У каждого комплексного числа существует бесконечное множество аргументов, которые отличаются между собой на число 2nπ, где n ∈ Z.
О п р е д е л е н и е. Главным значением аргумента комплексного числа
называется тот из его аргументов, который удовлетворяет условию
–π < ϕ ≤ π.
(1.6)
ЗАМЕЧАНИЕ. Наравне с предположением (1.6), существует и предположение
0 ≤ ϕ < 2π.
(1.7)
В дальнейшем будем использовать предположение (1.6).
Для обозначения главного значения аргумента используется символ
ϕ = arg z.
(1.8)
У каждого комплексного числа существует одно главное значение аргумента. Исключение составляет лишь число 0 + 0і, аргумент которого является
неопределённым.
Аргументы комплексно-сопряжённых чисел являются противоположными числами:
arg z = –arg z .
(1.9)
Действительно, поскольку комплексно-сопряжённые числа z = а + bi и
z = а – bi являются симметричными относительно оси Ох, то отсюда и следует
справедливость равенства (1.9).
10
Для нахождения аргумента комплексного числа удобно пользоваться
формулами
arctg
b
,
a
если z ∈ I четверти;
b
, если z ∈ IІ четверти;
a
b
–π + arctg , если z ∈ IІІ четверти;
a
b
arctg ,
если z ∈ IV четверти;
a
π,
если а < 0, b = 0;
0,
если а > 0, b = 0;
π
,
если а = 0, b > 0;
2
π
– ,
если а = 0, b < 0,
2
π + arctg
arg z =
если аргумент удовлетворяет условию (1.6)
или
b
если
arctg ,
a
b
π + arctg , если
a
b
π + arctg , если
a
b
2
π–
arctg
, если
arg z =
a
π,
если
0,
если
π
,
если
2
3π
,
если
2
(1.10)
(1.10)
z ∈ I четверти;
z ∈ IІ четверти;
z ∈ IІІ четверти;
z ∈ IV четверти;
(1.11)
(1.11)
а < 0, b = 0;
а > 0, b = 0;
а = 0, b > 0;
а = 0, b < 0,
если аргумент удовлетворяет условию (1.7).
Аргумент комплексного числа можно также отыскать, исходя из приведенных далее рассуждений.
11
a
b
, sin ϕ = . Аналиr
r
зируя знаки cos ϕ и sin ϕ, находим такое значение ϕ, которое принадлежит соответствующей четверти.
Обратимся снова к рис. 1.1. Очевидно, что cos ϕ =
1.4 Примеры
Пример 1.1 Преобразовать заданные выражения, пользуясь понятием
мнимой единицы:
1) − 16 ;
2) − 25 ; 3) 5 − − 64 ;
4) 6 + − 9 .
Решение.
1) − 16 = − 1⋅ 16 = − 1 16 = ±4 і;
2) − 25 = − 1⋅ 25 = − 1 25 = ±5 і;
3) 5 − − 64 = 5 − − 1 ⋅ 64 = 5 − − 1 64 = 5 – (±8і) = 5 m 8 і;
4) 6 + − 9 = 6 + − 1 ⋅ 9 = 6 + − 1 9 = 6 + 9i = 6 ± 3i .
О т в е т: 1) ±4і; 2) ±5і; 3) 5 m 8і; 4) 6 ± 3i .
Пример 1.2 Найти значение выражения А = 4і17 + 8і.
Решение.
А = 4і17 + 8і = 4і16 + 1 + 8і = 4і16⋅і + 8і = 4(і4)4і + 8і = 4і + 8і = 12і.
О т в е т: А = 12і.
ЗАМЕЧАНИЕ. Для того, чтобы найти степень мнимой единицы, удобно
пользоваться таким правилом:
1) разделить показатель степени n на 4;
2) заменить іn на ір, где р – остаток, полученный при делении n на 4, то
есть число р определяется из равенства n = 4k + р, где р < 4.
Пример 1.3 Вычислить значение выражения А = 5і7 – 4і11 + 9і17.
Решение.
А = 5і4 + 3 – 4і2⋅4 + 3 + 9і4⋅4 + 1 = 5і3 – 4і3 + 9і = і3 + 9і = – і + 9і = 8і.
О т в е т: А = 8і.
Пример 1.4 Для заданных чисел z1 = 3; z2 = –3i; z3 = 4 – 5i; z4 = –5 + 6i
найти числа:
1) комплексно-сопряженные; 2) противоположные.
Решение.
Запишем решение в виде таблицы:
Комплексноz
Противоположное число
сопряжённое число
z1 = 3
3
–3
z2 = –3i
3i
3i
z3 = 4 – 5i
4 + 5i
–4 + 5i
z4 = –5 + 6i
–5 – 6i
5 – 6i
12
Пример 1.5 Доказать равенство ( z ) = z .
Решение.
Пусть z = а + bi. Тогда z = а – bi. Отсюда ( z ) = a + bi . Следовательно,
(z ) = z .
(1.12)
Пример 1.6 Какими должны быть вещественные числа х и у, чтобы комплексные числа z1 = 9x2 – 4 + 10yi7 и z2 = 8x2і4 – 20i13 были комплексносопряжёнными?
Решение.
Упростим z1 и z2:
z1 = (9x2 – 4) − 10yi;
z2 = 8x2 – 20i.
Числа z1 и z2 должны удовлетворять условию z 2 = z1 или z 1 = z2 . По скольку
z1 = (9x2 – 4) + 10yi,
то должно выполняться равенство
(9x2 – 4) + 10y⋅i = 8x2 – 20i.
Рассматривая это равенство как условие равенства двух комплексных чисел,
приходим к такой системе уравнений:
9 x 2 − 4 = 8 x 2 ;
(1.13)

10
y
=
−
20
.

Решим эту систему уравнений:
 x 2 = 4;

 y = −2,
откуда х = –2; у = –2 или х = 2; у = –2.
О т в е т: х = –2; у = –2 или х = 2; у = –2.
ЗАМЕЧАНИЕ. Система уравнений (1.13) была получена из предположения,
что z2 = z1 но, допустив, что z1 = z2 получим также систему уравнений, эквивалентную системе (1.13).
Пример 1.7 Решить уравнение (1 + і) х + (–2 + 5і) у = –4 + 17і.
Решение.
В заданном уравнении раскроем скобки:
х + х і – 2у + 5у і = –4 + 17і.
Исходя из того, что комплексные числа являются равными между собой,
если равны их действительные и равны мнимые части, запишем последнее равенство так, чтобы и левая и правая части были представлены в виде суммы
действительной и мнимой частей комплексного числа.
(х – 2у) + (х + 5у)і = –4 + 17і.
Опираясь на условие равенства двух комплексных чисел, получим систему уравнений
13
 x − 2 y = −4;

 x + 5 y = 17.
От второго уравнения системы вычтем первое уравнение, тогда 7у = 21
или у = 3. Подставив у = 3 в первое уравнение системы, получим х = –4 + 23
или х = 2. Следовательно, решением уравнения является х = 2; у = 3.
О т в е т: х = 2; у = 3.
ЗАМЕЧАНИЕ. Понятие равенства двух комплексных чисел даёт возмож-
ность решать одно уравнение с двумя неизвестными.
Пример 1.8 Найти действительные числа х и у из уравнения
9 + 2іх + 4іу = 10і + 5х – 6у.
Решение.
Заданное уравнение можно рассматривать как равенство двух комплексных чисел:
9 + (2х + 4у)і = (5х – 6у) + 10і.
Исходя из определения равенства комплексных чисел, получим систему уравнений
5 x − 6 y = 9;
5 x − 6 y = 9;
или


 x + 2 y = 5.
2 x + 4 y = 10 ,
Решим эту систему уравнений
 x = 5 − 2 y;
 у = 1;
 x = 5 − 2 y;



 х = 3.
16 у = 16,
5(5 − 2 у ) − 6 y = 9,
Следовательно, х = 3, у = 1.
О т в е т: х = 3, у = 1.
Пример 1.9 Решить квадратное уравнение z2 – 10z + 34 = 0.
Решение.
Решим это уравнение, воспользовавшись формулой для нахождения корней квадратного уравнения az2 + bz + c = 0:
− b ± b 2 − 4ac
z1,2 =
.
2a
Подставим значение а, b, c в эту формулу и получим
10 ± 6i
10 ± 100 − 4 ⋅ 1 ⋅ 34
10 ± − 36
z1,2 =
; z1,2 =
; z1,2 =
;
2 ⋅1
2
2
z1 = 5 – 3i, z2 = 5 + 3i.
О т в е т: z1 = 5 – 3i, z2 = 5 + 3i.
ЗАМЕЧАНИЕ. Следует обратить внимание на то, что на множестве
комплексных чисел квадратное уравнение с действительными коэффициентами
всегда имеет два корння. Если дискриминант D > 0, то корнями являются
действительные разные числа; если D = 0, то корнями являются действительные равные числа; если D < 0, то корнями являются комплексно-сопряжённые
числа.
14
Пример 1.10 Решить уравнение
х2 + 1 = 0
1) на множестве действительных чисел; 2) на множестве комплексных
чисел.
Решение.
Решая уравнение на множестве действительных чисел, получим
1)
2
х = –1. Такое уравнение решения не имеет.
2) Решая уравнение на множестве комплексных чисел, получим х2 = –1, откуда х = ± і. Следовательно, х1 = – і; х2 = і.
О т в е т: 1) х ∈ ∅; 2) х1 = – і; х2 = і.
Пример 1.11 Изобразить точками на комплексной плоскости числа:
z1 = –3i; z2 = –3; z3 = 5 + 2i; z4 =5 – 2i; z5 = –5 + 2i; z6 = –5 – 2i.
Решение.
Im z
M5
2
1
M2
–5
M6
–3
M3
О
M1
–1
–2
–3
Рисунок 1.2
5
M4
На комплексной плоскости изобразим
заданные
комплексные
числа
(рис. 1.2).
Re z
Следует учесть, что числу z1 = –3i
отвечает точка М1(0; –3), а числу z2 = –3 –
точка М2(–3; 0). Дальше соответствен но имеем точки
М3(5; 2), М4(5; –2), М5(–5; 2), М6(–5; –2).
Пример 1.12 Записать аффиксы заданных комплексных чисел (рис. 1.3) и
построить соответствующие им комплексные векторы: z1 = 2 + 3i; z2 = –2 + 3і;
z3 = –2 − 3i; z4 = 2 – 3i; z5 = 3i; z6 = 2.
Im z
Решение.
Поскольку аффиксом комплексного числа
M2 3 M5 M1
z = а + bi называется точка на комплексной
плоскости, то аффиксами данных комплексных чисел соответственно являются точки
M6 Re z
О
М1(2; 3); М2(–2; 3); М3(–2; –3); М4(2; –3);
2
–2
М5(0; 3); М6(2; 0).
Комплексными векторами соответст–3 M
венно являются векторы ОМ 1 = {2; 3};
M3
4
Рисунок 1.3
ОМ 2 = {–2; 3}; ОМ 3 = {–2; –3}; ОМ 4 = {2; –3};
ОМ 5 = {0; 3}; ОМ 6 = {2; 0}.
Пример 1.13 Построить на комплексной плоскости множество точек, которые удовлетворяют заданному условию:
1) Re z ≤ 2; 2) Re z ≥ 3; 3) –1 ≤ Re z ≤ 4; 4) Im z ≥ 6; 5) –1 ≤ Im z ≤ 3;
15
−3 ≤ Re z ≤ 1;
 2 ≤ Im z ≤ 4.
6) 
Решение.
Пусть z = x + iy.
1) Условие Re z ≤ 2 равносильно условию х ≤ 2. Соответствующее множество точек изображено на рис. 1.4.
2) Из условия Re z ≥ 3 выходит, что х ≥ 3. Соответствующее множество
точек изображено на рис. 1.5.
Im z
Im z
2
Re z
Re z
3
О
х=3
х=2
О
Рисунок 1.4
Рисунок 1.5
3) Из условия –1 ≤ Re z ≤ 4 выходит, что –1 ≤ х ≤ 4. Соответствующее
множество точек изображено на рис. 1.6.
4) Из условия Im z ≥ 6 выходит, что у ≥ 6. Соответствующее множество
точек изображено на рис. 1.7.
Im z
Im z
у=6
Re z
4
х=4
х = –1
–1 О
6
Re z
О
Рисунок 1.6
Рисунок 1.7
5) Условие –1 ≤ Im z ≤ 3 приводит к неравенству –1 ≤ у ≤ 3. Соответствующее множество точек изображено на рис. 1.8.
6) Из условия
16
выходит, что –3 ≤ х ≤ 1; 2 ≤ у ≤ 4. Соответствующее множество точек изображено на рис. 1.9.
Im z
4
у=4
Im z
у=2
у=3
2
3
О
–1
х=1
х = –3
у = –1
О 1
–3
Re z
Re z
Рисунок 1.9
Рисунок 1.8
Пример 1.14 Построить на комплексной плоскости множество точек, которые удовлетворяют заданному условию:
1 ≤ r < 2;
π

1) |z| ≤ 3; 2) |z –2| ≤ 2; 3) 1 ≤ |z – і| ≤ 3; 4) 0 ≤ arg z ≤ ; 5)  π
3π
4
≤
ϕ
≤
.
 4
4
Решение.
Пусть z = x + iy.
1) Условие |z| ≤ 3 равносильно неравенству
x 2 + y 2 ≤ 3 или х2 + у2 ≤ 9.
Последнее неравенство определяет множество точек круга с центром в начале
координат и радиусом r = 3 и на окружности, которая его ограничивает. Соответствующее множество точек изображено на рис. 1.10.
2) Запишем неравенство |z –2| ≤ 2 в форме |x + iy – 2| ≤ 2, откуда |(x –2) + iy | ≤ 2
или (х – 2)2 + у2 ≤ 4. Последнее неравенство задает круг с центром в точке (2; 0),
радиус которого r = 2 (рис. 1.11) и ограничивающую его окружность.
Im z
3
Im z
(х2 – 2)+ у2 = 4
х2 + у2 = 9
2
Re z
О
Рисунок 1.10
3
Re z
О
2
Рисунок 1.11
4
17
3) Из неравенства 1 ≤ |z – і| ≤ 3 выходит 1 ≤ |х + і(у – 1)| ≤ 3,
1≤
х 2 + ( у − 1) 2 ≤ 3. Отсюда, имеем неравенство 1 ≤ х2 + (у – 1)2 ≤ 9, которое
определяет кольцо, изображенное на рис. 1.12.
π
4) Неравенство 0 ≤ arg z ≤ определяет множество точек, которые огра4
π
ничены осью Re z и лучом, выходящим из начала координат под углом ϕ = к
4
оси Re z (рис. 1.13).
Im z
4
Im z
х2 + (у – 1)2 = 9
2
Re z
1
π/4
О
О
2
–2
Re z
2
х + (у – 1) = 1
Рисунок 1.12
Рисунок 1.13
5) Условие
1 ≤ r < 2;

π
3π
 4 ≤ ϕ ≤ 4
определяет ту часть кольца 1 ≤ х2 + у2 < 2, которая ограничена лучами, выходяπ
3π
щими из начала координат под углами ϕ = и ϕ =
к оси Re z. При этом точ4
4
ки, которые лежат на большей окружности, к отмеченному множеству точек не
принадлежат. Соответствующее множество точек изображено на рис. 1.14.
Im z
2
1
О
Рисунок 1.14
Re z
18
2 ДЕЙСТВИЯ НАД КОМПЛЕСНИМИ ЧИСЛАМИ, ЗАДАННЫМИ
В АЛГЕБРАИЧЕСКОЙ ФОРМЕ
2.1 Алгебраическая форма комплексного числа
Как отмечалось ранее, комплексное число можно задавать в виде z = а + bi
или z = (а; b). Последующее изучение комплексных чисел показывает, что комплексные числа можно задавать и другими способами.
Комплексное число, заданное в виде z = а + bi, называется комплексным
числом в алгебраической форме.
Рассмотрим действия над комплексными числами, заданными в алгебраической форме.
2.2 Сложение комплексных чисел
О п р е д е л е н и е. Суммой двух комплексных чисел называется такое
комплексное число, действительная часть которого равняется сумме действительных частей слагаемых, а мнимая часть – сумме мнимых частей слагаемых,
то есть
(a1 + b1 i) + (a2 + b2 i) = (a1 + a2) + (b1 + b2) i.
(2.1)
Следовательно, чтобы сложить два комплексных числа, нужно сложить
их действительные части, что дает действительную часть суммы, и сложить
мнимые части, что дает мнимую часть суммы.
Сумма комплексно-сопряжённых чисел всегда является действительным
числом.
z + z = (а + bi) + (а – bi) = 2a.
Следовательно,
z + z = 2Re z.
(2.2)
Свойства суммы комплексных чисел
1. Сложение комплексных чисел является коммутативным, то есть для
любых комплексных чисел z1 = a1 + b1i и z2 = a2 + b2i справедливо равенство
(a1 + b1i) + (a2 + b2i) = (a2 + b2i) + (a1 + b1i).
(2.3)
2. Сложение комплексных чисел является ассоциативным, то есть для любых комплексных чисел z1 = a1 + b1i, z2 = a2 + b2i, z3 = a3 + b3i справедливо равенство
((a1 + b1i) + (a2 + b2i)) + (a3 + b3i) = (a1 + b1i) + ((a2 + b2i) + (a3 + b3i)). (2.4)
2.3 Вычитание комплексных чисел
О п р е д е л е н и е. Разностью двух комплексных чисел называется такое число, которое в сумме с вычитаемым дает уменьшаемое.
19
Вычитание комплексных чисел является всегда возможным.
Теорема Для любых комплексных чисел z1 = a1 + b1i и z2 = a2 + b2i всегда существует разность z = z1 – z2, определяемая однозначно.
Доказательство
Докажем, что существует такое число z = x + iy, которое удовлетворяет условию z1 = z + z2, то есть что a1 + b1i = x + iy + a2 + b2i или a1 + b1i = (x + a2) +
(y + b2)i. На основании равенства комплексных чисел приходим к системе уравнений
a1 = x + a2 ;

b1 = y + b2 .
Эта система уравнений имеет решение, и к тому же лишь одно, а именно:
х = а1 – а2, у = b1 – b2,
что и нужно было доказать.
Следовательно, разность двух комплексных чисел – это такое комплексное число, действительная и мнимая части которого равняются соответственно
разности действительных и мнимых частей уменьшаемого и вычитаемого, то
есть
(2.5)
(a1 + b1i) – (a2 + b2i) = (a1 – а2) + (b1 – b2) i.
Разность комплексно-сопряжённых чисел всегда является мнимым числом.
z – z = (a + bi) – (a – bi) = 2bi,
или
z + z = 2(Im z) і.
(2.6)
2.4 Умножение комплексных чисел
О п р е д е л е н и е. Произведением двух комплексных чисел z1 = a1 + b1i
и z2 = a2 + b2i называется комплексное число, определяемое формулой
(a1 + b1 i) (a2 + b2 i) = (a1a2 – b1b2) + (a1b2 + a2b1) i.
(2.7)
Практически, чтобы умножить комплексные числа, следует умножить их
по правилу умножения многочленов, заменив при этом і2 на –1, и привести подобные члены.
a1 + b1i)(a2 + b2i) = a1a2 + a1b2i + a2b1i + b1b2i2 =
= (a1a2 – b1b2) + (a1b2 + a2b1)i.
Произведение комплексно-сопряжённых чисел всегда является действительным числом.
z⋅ z = (a + bi) (a – bi) = a2 – (bi)2 = a2 + b2.
Следовательно,
z z = a2 + b2,
(2.8)
или
z z = r2.
(2.9)
20
Свойства произведения комплексных чисел
1. Умножение комплексных чисел является коммутативным, то есть для
любых комплексных чисел z1 = a1 + b1i и z2 = a2 + b2i справедливо равенство
(a1 + b1i)(a2 + b2i) = (a2 + b2i)(a1 + b1i).
(2.10)
2. Умножение комплексных чисел является ассоциативным, то есть для
любых комплексных чисел z1 = a1 + b1i, z2 = a2 + b2i, z3 = a3 + b3i справедливо
равенство
((a1 + b1i)(a2 + b2i))(a3 + b3i) = (a1 + b1i)((a2 + b2i)(a3 + b3i)). (2.11)
3. Умножение комплексных чисел является дистрибутивным относительно сложения, то есть для любых комплексных чисел z1 = a1 + b1i, z2 = a2 + b2i,
z3 = a3 + b3i справедливо равенство
((a1 + b1i) + (a2 + b2i)) (a3 + b3i) = (a1 + b1i) (a3 + b3i) + (a2 + b2i) ⋅(a3 + b3i). (2.12)
2.5 Деление комплексных чисел
О п р е д е л е н и е. Частным от деления комплексных чисел называется
такое комплексное число, которое в произведении с делителем дает делимое,
если делитель отличается от нуля.
Докажем, что всегда существует частное от деления двух комплексных
чисел, если знаменатель отличается от нуля.
a +bi
Теорема Частное 1 1 определяется однозначно для любых комa 2 + b2 i
плексных чисел a1 + b1i и a2 + b2 i , если a2 + b2 i ≠ 0.
Д о к а з а т е л ь с т в о.
Пусть z1 = a1 + b1 i, z2 = a2 + b2 i. Докажем, что существуют такие числа х
и у, которые удовлетворяют уравнению (х + іу) (a2 + b2i) = a1 + b1i. Выполнив
умножение, получим:
(хa2 – уb2) + (хb2 + уа2) і = a1 + b1i.
Исходя из равенства комплексных чисел, имеем систему уравнений
 xa2 − yb2 = a1 ;

 xb2 + ya2 = b1 .
Решив эту систему уравнений, находим
a a +bb
a b −ab
x = 1 22 12 2 ; y = 2 21 12 2 .
a2 + b2
a2 + b2
Следовательно, система уравнений имеет решение, и к тому же единственное. Тогда
a1 + b1i a1a2 + b1b2 a1b1 − a1b2
=
+ 2
i.
(2.13)
a2 + b2 i
a22 + b22
a2 + b22
ЗАМЕЧАНИЕ. Деление комплексных чисел в алгебраической форме удобно
выполнять следующим образом. Числитель и знаменатель следует умножить
на число, комплексно-сопряженное знаменателю, после чего в числителе и зна-
21
менателе выполнить умножение комплексных чисел по правилу умножения
многочленов. Полученный результат записать в алгебраической форме.
2.6 Примеры
Пример 2.1 Выполнить действия:
1) (4 – 3і) + (2 + 8і);
2) (–7 + 2і) – (2 + 4і);
3) 5(2 – і) + 6(–1 + 2і) – 7(–2 + 3і);
4) (7 – 6і) + (7 + 6і);
5) (–9 – 5і) – (–9 + 5і).
Решение
Использовав формулы (2.1), (2.2), (2.5), (2.6), получим:
1) (4 – 3і) + (2 + 8і) = 4 – 3і + 2 + 8і = 6 + 5і;
2) (–7 + 2і) – (2 + 4і) = –7 + 2і – 2 – 4і = –9 – 2і;
3) 5(2 – і) + 6(–1 + 2і) – 7(–2 + 3і) = 10 – 5і – 6 + 12і + 14 – 21і = 18 – 14і;
4) (7 – 6і) + (7 + 6і) = 14;
5) (–9 – 5і) – (–9 + 5і) = –10і.
О т в е т: 1) 6 + 5і; 2) –9 – 2і; 3) 18 – 14і; 4) 14; 5) –10і.
Пример 2.2 Найти значение выражения А = (6 – 3і) (9 + і) – (5 – і) (5 + і).
Решение
Воспользовавшись правилом умножения многочленов, имеем
А = (54 – 27і + 6і – 3і2) – (25 – і2) = (57 – 21і) – (25 + 1) = 31 – 21і.
О т в е т: А = 31 – 21і.
Пример 2.3 Выполнить действия:
1) (6 + 4і) (7 + 8і);
2) (–9 +2і) (5 – 3і);
4) (–2 – 5і) (–2 + 5і).
3) (1 – і) (2 + і) (3 – 4і);
Решение
Воспользуемся правилом умножения многочленов:
1) (6 + 4і) (7 + 8і) = 42 + 28і + 48і + 32і2 = 42 + 76і – 32 = 10 + 76і;
2) (–9 +2і) (5 – 3і) = –45 + 10і + 27і – 6і2 = –45 + 37і +6 = –39 + 37і;
3) (1 – і) (2 + і)(3 – 4і) = (2 – 2і + і – і2) (3 – 4і) = (2 – і + 1) (3 – 4і) =
2
= (3 – і) (3 – 4і) = 9 – 3і – 12і + 4і = 9 – 15і – 4 = 5 – 15і;
4) По формуле (2.8) имеем:
(–2 – 5і) (–2 + 5і) = (–2)2 – (5i)2 = 29.
О т в е т: 1) 10 + 76і; 2) −39 + 37і; 3) 5 – 15і; 4) 29.
Пример 2.4 Выполнить действия:
1
2−i
4 − 5i
(2 − 3i )(4 + i )
1)
; 2)
; 3)
; 4)
.
3 − 4i
3+i
2 − 3i
5−i
Решение
Деление комплексных чисел можно выполнять по формуле (2.13), но
проще это сделать, умножив числитель и знаменатель на число, комплексносопряжённое знаменателю.
22
1
1
3 + 4i 3 + 4i 3 + 4i 3
4
=
⋅
=
=
=
+ i;
3 − 4i 3 − 4i 3 + 4i 9 + 16
25
25 25
2
2 − i 2 − i 3 − i 6 − 3i − 2i + i
6 − 5i − 1 5 − 5i 1 1
2)
=
⋅
=
=
=
= − i;
3+i 3+i 3−i
9 +1
10
10
2 2
2
4 − 5i 4 − 5i 2 + 3i 8 − 10i + 12i − 15i
8 + 2i + 15 23 + 2i 23 2
=
⋅
=
=
=
= + i;
3)
2 − 3i 2 − 3i 2 + 3i
4+9
13
13
13 13
2
(2 − 3i )(4 + i ) 8 − 12i + 2i − 3i
8 − 10i + 3 11 − 10i 5 + i
4)
=
=
=
⋅
=
5−i
5−i
5−i
5−i 5+i
− i + 11
+i − 10
−i 2 2 55 − 39
− i + 10
+ 65 − 39
− i 65 39
55 − 50
5 3
=
=
=
=
− i = − i.
25 + 1
26
26
26 26
2 2
3
4
1 1
23 2
5 3
О т в е т: 1)
+ i ; 2) − i ; 3)
+ i ; 4) − i .
2 2
2 2
25 25
13 13
1)
Пример 2.5 Доказать, что для любых комплексных чисел z1 и z2 справедливы равенства
1) z1 + z 2 = z1 + z2 ; 2) z1 – z 2 = z1 − z2 .
Решение
Пусть z1 = a1 + b1i; z2 = a2 + b2i. Тогда z1 = a1 – b1i; z 2 = a2 – b2i.
1) Найдем z1 + z 2 = (a1 – b1i) + (a2 – b2i) = (a1 + а2) – (b1 + b2)i. Далее на ходим z 1 + z 2 = (a 1 + b 1 i) + (a 2 + b 2 i) = (a 1 + а 2 ) + (b 1 + b 2 )i, откуда
z1 + z2 = (a1 + а2) – (b1 + b2)i.
Таким способом убеждаемся в справедливости равенства
z1 + z 2 = z1 + z2 .
(2.14)
2) Теперь найдем
z1 – z 2 = (a1 – b1i) – (a2 – b2i) = (a1 – а2) + (–b1 + b2) i = (a1 – а2) – (b1 – b2) i.
Переходим к нахождению z1 – z2 и z1 − z2 .
z1 – z 2 = (a1 + b1i) – (a2 + b2i) = (a1 – а2) + (b1 – b2)i, тогда
z1 − z2 = (a1 – а2) – (b1 – b2)i.
Следовательно, становится очевидным равенство
z1 – z 2 = z1 − z2 .
(2.15)
Пример 2.6 Разложить на комплексные множители заданные выражения.
1) 64х2 + 625у2; 2) m + n; 3) 137; 4) 11.
Решение
1) 64х2 + 625у2 = 64х2 – ( −625 y 2 ) = 64х2 – ( 625y 2i 2 ) = (8х)2 – (25уі)2 =
= (8х – 25уі) (8х + 25уі);
2) m + n = m – (–n) = m – ( i 2 n ) =
( m ) – (i n )
2
=
(
m− ni
)(
)
m+ ni ;
23
3) 137 = 121 + 16 = 121 – (–16) = 121 – (16i 2 ) = (11)2 – ( 4i ) =
2
= (11 – 4i) (11 + 4i);
4) 11 = 5 + 6 = 5 – (–6) = 5 – ( 6 i 2 ) =
2
( 5 ) –(
В і д п о в і д ь: 1) (8х – 25уі) (8х + 25уі); 2)
3) (11 – 4i) (11 + 4i); 4)
(
5− 6i
)(
6i
(
2
) =(
5− 6i
m− ni
)(
)(
)
5+ 6i .
)
m+ ni ;
)
5+ 6i .
Пример 2.7 Одним из корней квадратного уравнения с действительными
3 4
коэффициентами есть число z1 = − i . Найти второй корень этого уравнения
5 5
и записать само уравнение.
Решение
Поскольку искомое уравнение является квадратным уравнением с действительными коэффициентами, то, зная один из корней уравнения, можно найти
3 4
3 4
и второй корень, а именно, если z1 = − i то z 2 = z1 = + i .
5 5
5 5
Соответствующее квадратное уравнение имеет вид z2 + pz + q = 0, где
6
 3 4   3 4 
p = –(z1 + z2) = – (z1 + z1 ) = –   − i  +  + i   = − ;
5
 5 5   5 5 
2
2
9 16
 3 4  3 4   3   4 
q = z1z2 = z1 z1 =  − i  + i  =   +   =
+
= 1.
25 25
 5 5  5 5   5   5 
Следовательно, приходим к уравнению
6
z 2 − z + 1 = 0 или 5z2 – 6z + 5 = 0.
5
3 4
О т в е т: z2 = + i ; 5z2 – 6z + 5 = 0.
5 5
Пример 2.8 Доказать, что для любых комплексных чисел z1 и z2 справедливо равенство
z1 z2 = z1 z2 .
Решение
Пусть z1 = a1 + b1i; z2 = a2 + b2i. Тогда z1 = a1 – b1i; z 2 = a2 – b2i.
Найдем z1 z2 = (a1 – b1i) (a2 – b2i) = (a1a2 – b1b2) + (–a1b2 – a2b1) i =
= (a1a2 – b1b2) – (a1b2 + a2b1) i.
Теперь найдем z1 z2 и z1 z2 .
z1 z2 = (a1 + b1i) (a2 + b2i) = (a1a2 – b1b2) + (a1b2 + b1a2) i;
z1 z2 = (a1a2 – b1b2) – (a1b2 + a2b1) i.
Тем самым доказано равенство
24
z1 z2 = z1 z2 .
Пример 2.9 Найти значение выражения А =
(2.16)
3−i
.
3+i
Решение
3 − i 3 − i 3 − 2i 3 + i 2 23 − 2 3 i 2 − 2 3 i 1
3
А=
⋅
=
=
=
= −
i.
2
2
3 +1
4
2 2
3 +i 3 −i
( 3) − i
1
3
О т в е т: А = −
i.
2 2
Пример 2.10 Выполнить действия:
8 1
3  1
3 
(1 + 2i ) 2 13 + 12i
i  +
i  ; 2)
+
;
1) 5  − +
2+i
6i − 8
i  2 2  2 2 
1+ i 1− i
(1 − i ) 3 − (1 + 2i ) 2
+
;
4)
.
1− i 1+ i
(2 + i ) 2 − (3 + 2i ) 3
Решение
8 1
3  1
3  8  1 3 2  8i  1 3 
1) 5  − +
i  +
i  =  − + i  =  − −  = −8i ( −1) = 8i ;
i  2 2  2 2  i  4 4  i 2  4 4 
3)
(1 + 2i) 2 13 + 12i 1 + 4i + 4i 2 2 − i 13 + 12i (−3i − 4)
2)
+
=
⋅
+
⋅
=
2+i
6i − 8
2+i
2 − i 2(3i − 4) (−3i − 4)
(1 + 4i − 4)(2 − i ) − 39i − 52 − 36i 2 − 48i (−3 + 4i )(2 − i ) − 52 + 36 − 87i
=
+
=
+
=
4 + 1i
2(16 + 9)
4 − i2
2(−9i 2 + 16)
−6 + 8i + 3i − 4i 2 −16 − 87i −6 + 4 + 11i −16 − 87i
+
=
+
=
5
50
5
50
−2 + 11i −16 − 87i
2 11 16 87
18 23
=
+
= − + i − − i = − + i;
5
50
5 5
50 50
25 50
=
3)
1 + i 1 − i (1 + i ) 2 + (1 − i ) 2 1 + 2i + i 2 + 1 − 2i + i 2 2 + 2i 2 2 − 2
+
=
=
=
=
= 0;
1− i 1+ i
(1 − i )(1 + i )
2
2
1 − i2
(1 − i ) 3 − (1 + 2i ) 2
1 − 3i + 3i 2 − i 3 − 1 − 4i − 4i 2
=
=
4)
(2 + i ) 2 − (3 + 2i ) 3 4 + 4i + i 2 − 27 − 54i − 36i 2 − 8i 3
=
−7i − 3 + i + 4
1 − 6i
1 − 6i
1 1 − 6i 2 + 7i
=
=
= ⋅
⋅
=
−23 − 50i − 1 + 36 + 8i 12 − 42i 6 ( 2 − 7i ) 6 2 − 7i 2 + 7i
25
1 2 − 12i + 7i − 42i 2 1 2 − 5i + 42 1 44 − 5i
44
5
= ⋅
=
⋅
=
⋅
=
−
i=
6
6
4 + 49
6
53
6 ⋅ 53 6 ⋅ 53
4 − 49i 2
=
22
5
−
i.
159 318
О т в е т: 1) 8і; 2) −
18 23
22
5
+ i ; 3) 0; 4)
−
i.
25 50
159 318
Пример 2.11 Доказать тождество:
1− a + i 1+ a
1+ a + i 1− a
−
= −2a .
1− a − i 1+ a
1+ a − i 1− a
Решение
1− a + i 1+ a
1+ a + i 1− a
1− a + i 1+ a 1− a + i 1+ a
−
=
⋅
−
1− a − i 1+ a
1+ a − i 1− a
1− a − i 1+ a 1− a + i 1+ a
2
2
1 + a + i 1 − a 1 + a + i 1 − a (1 − a ) + 2i 1 − a + i (1 + a )
−
⋅
=
−
1+ a − i 1− a 1+ a + i 1− a
(1 − a ) − i 2 (1 + a )
1 + a ) + 2i 1 − a 2 + i 2 (1 − a ) 1 − a + 2i 1 − a 2 − 1 − a 1 + a + 2i 1 − a 2 − 1 + a
(
−
=
−
=
1− a +1+ a
1+ a +1− a
(1 + a ) − i 2 (1 − a )
2i 1 − a 2 − 2a 2i 1 − a 2 + 2a
=
−
= i 1 − a 2 − a − i 1 − a 2 − a = −2 a .
2
2
Следовательно, тождество доказано.
Пример 2.12 Доказать, что для любых комплексных чисел z1 и z2 (z2 ≠ 0),
справедливо равенство
z1  z1 
= .
z 2  z2 
Решение
Пусть z1 = а1 + b1i, z2 = а2 + b2i ≠ 0. Тогда z1 = а1 – b1i, z 2 = а2 – b2i.
z
Найдем частное 1 .
z2
26
z1 a1 − b1i a1 − b1i a2 + b2 i (a1a 2 + b1b2 ) + (a1b2 − a 2 b1 )i
=
=
⋅
=
=
z 2 a 2 − b2i a2 − b2i a2 + b2 i
a22 + b22
a1a2 + b1b2 a1b2 − a2 b1
+
i.
a22 + b22
a22 + b22
z
Теперь найдем частное 1 .
z2
z1 a1 + b1i a1 + b1i a 2 − b2 i (a1a2 + b1b2 ) + (− a1b2 + a2 b1 )i
=
=
⋅
=
=
z 2 a 2 + b2 i a2 + b2 i a 2 − b2 i
a22 + b22
=
=
(a1a2 + b1b2 ) − (a1b2 − a2 b1 )i a1a2 + b1b2 a1b2 − a2 b1
=
−
i.
a 22 + b22
a 22 + b22
a22 + b22
Тогда
 z1  a1a2 + b1b2 a1b2 − a2b1
+
i.
 =
2
2
2
2
z
a
+
b
a
+
b
 2
2
2
2
2
Следовательно, доказано равенство
z1  z1 
= .
(2.17)
z 2  z2 
Следует заметить, что, кроме операций сложения, вычитания, умножения
и деления над комплексными числами, заданными в алгебраической форме,
можно выполнять операции возведения в натуральную степень и нахождения
корней квадратных. При этом возведение в натуральную степень следует рассматривать как последовательное умножение числа самого на себя, а нахождение квадратного корня выполняется по формуле.

a2 + b2 + a
b
a 2 + b 2 − a 

a + bi = ±
+i
.
(2.18)


2
|b|
2


Пример 2.13 Найти z10, если z = 1 – i.
Решение
(1 – i)10 = ((1 – i)2)5 = (1 – 2i + i2)5 = (1 – 2i – 1)5 = (–2i)5 = –32i5 = –32i.
О т в е т: −32i .
Пример 2.14 Найти 1 − i .
Решение
Будем опираться на формулу (2.18). По условию z = 1 – і; а = 1; b = −1 .


2 + 1 ( −1)
2 −1 
2 +1
2 − 1 
 ; 1 − i = ±
1− i = ±
+i
−i
.



2
1
2 
2
2




Следовательно, найдены два значения 1 − i , а именно:
27

2 +1
z1 = − 
−i

2

Сделаем проверку:
 
z12 =  − 
 
 
=
2
2 +1
−i
2
2 +– 1
− 2i
2
= i2
2 − 1 
и z2 =
2 
2 − 1   
= i
2   

2 +1
−i
2
2 −1
.
2
2
2 +– 1
−i
2
2 −+ 1 
=
2 
2 −1 2 +1
2 +1
2 −1
1
2 +1
⋅
+
=−
− 2i ⋅
=
2 −1 +
2
2
2
2
2
2
2 +1− 2 +1
− i = 1 − i.;
2

z 22 = 


2
2 +1
−i
2
2 − 1 
2 +1
=
− 2i
2 
2
2 +1 2 −1 2 2 −1
⋅
+i
=
2
2
2
2 +1
1
2 −1
2 +1− 2 +1
2 −1 −
− 2i ⋅
=
− i = 1 − i.
2
2
2
2

2 +1
2 +1
2 − 1 
О т в е т: z1 = − 
и z2 =
−i
−i


2
2
2


=
2 −1
.
2
28
3 ДЕЙСТВИЯ НАД КОМПЛЕКСНЫМИ ЧИСЛАМИ,
ЗАДАННЫМИ В ТРИГОНОМЕТРИЧЕСКОЙ ФОРМЕ
3.1 Тригонометрическая форма комплексного числа
Рассмотрим комплексное число z = а + bi. Обратимся к рис. 1.1. Из прямоугольного треугольника ОАМ выходит, что а = ОА; b = АМ или
 a = r cos ϕ;
(3.1)

=
sin
ϕ
.
b
r

В результате этого комплексное число z = а + bi можно представить в виде
z = r(cos ϕ + i sinϕ).
(3.2)
Комплексное число, заданное в виде (3.2), называется комплексным числом в тригонометрической форме.
О п р е д е л е н и е. Два комплексных числа z1 = r 1 (cos ϕ 1 + i sin ϕ 1 )
и z2 = r2(cos ϕ2 + i sin ϕ2) називаются равными, если модули этих чисел равны
между собой, а их аргументы либо равны между собой, либо отличаются
на число, кратное 2π, то есть r1 = r2 и ϕ1 = ϕ2 + 2 π n, где n ∈ Z.
Над комплексными числами в тригонометрической форме удобно выполнять такие алгебраические действия, как умножение, возведение в степень, деление, извлечение корня степени n.
3.2 Умножение комплексных чисел
Теорема Произведением двух комплексных чисел, заданных в тригонометрической форме, является такое комплексное число, модуль которого равняется произведению модулей сомножителей, а аргумент равняется сумме
аргументов сомножителей.
Доказательство
Пусть z1 = r1(cos ϕ1 + i sin ϕ1), z2 = r2(cos ϕ2 + i sin ϕ2).
Тогда z1 z2 = r1(cos ϕ1 + i sin ϕ1) r2(cos ϕ2 + i sin ϕ2) =
= r1r2((cos ϕ1 cos ϕ2 – sin ϕ1 sin ϕ2) + і(sin ϕ1 cos ϕ2 + sin ϕ2 cos ϕ1)) =
= r1r2(cos(ϕ1 + ϕ2) + і sin(ϕ1 + ϕ2)).
Следовательно, доказана формула
r1(cos ϕ1 + i sinϕ1) r2(cos ϕ2 + i sinϕ2) = r1r2(cos(ϕ1 + ϕ2) +і sin(ϕ1 + ϕ2)). (3.3)
3.3 Возведение в степень комплексных чисел. Первая формула Муавра
О п р е д е л е н и е. Произведение n равных между собой комплексных
чисел z называется n-й степенью комплексного числа и обозначается символом zn, где n ∈ N.
Теорема Если комплексное число z задано в виде z = r(cos ϕ + i sin ϕ), то
для любого целого числа n справедлива формула
n
(3.4)
( r ( cos ϕ + i sin ϕ ) ) = rn(cos nϕ + i sin nϕ), где n ∈ Z.
29
Д о к а з а те л ь с т в о
z = r (cos ϕ + i sin ϕ).
Тогда
z2 = z ⋅ z = r2(cos 2ϕ + i sin 2ϕ);
z3 = z ⋅ z ⋅ z = r3(cos 3ϕ + i sin 3ϕ);
z4 = z ⋅ z ⋅ z ⋅ z = r4(cos 4ϕ + i sin 4ϕ);
…………….………………………..
Продолжая процесс умножения, придем к выводу, что справедливо равенство
zk = rk(cos k ϕ + i sin k ϕ),
где k ∈ N.
Покажем, что тогда будет справедливо и равенство
z k +1 = r k +1 ( cos ( k + 1) ϕ + i sin ( k + 1) ϕ ) .
Имеем
zk + 1 = zkz = r k ( cos k ϕ + i sin k ϕ ) r(cos ϕ + i sin ϕ).
По формуле (3.3) получим
z k +1 = r k +1 ( cos ( k + 1) ϕ + i sin ( k + 1) ϕ ) .
Следовательно, согласно методу математической индукции, приходим к
выводу, что формула (3.4) является справедливой для любого натурального
числа n.
Далее покажем, что эта формула справедлива и для любого целого числа
n. Пусть n – целое отрицательное число. Тогда рассмотрим число m = –n. Понятно, что т – число натуральное. Тогда
z n = z −m =
1
1
1
=
=
=
m
z m ( r ( cos ϕ + i sin ϕ ) )
r m ( cos mϕ + i sin mϕ )
1
cos mϕ − i sin mϕ
cos mϕ − i sin mϕ
⋅
= r −m
=
2
cos mϕ + i sin mϕ cos mϕ − i sin mϕ
cos 2 mϕ − i 2sin
ϕ
sin2m
mϕ
= r − m ( cos ( − m ) ϕ + i sin ( −m ) ϕ ) = r n ( cos nϕ + i sin nϕ ) .
= r −m
Выходит, что формула (3.4) справедлива, как для положительных, так и
для отрицательных n .
Поскольку по договоренности считают, что z0 = 1, то можно утверждать,
что формула (3.4) является справедливой для любого целого числа n ∈ Z.
Формула (3.4) называется первой формулой Муавра.
30
3.4 Деление комплексных чисел
Теорема. Частным двух комплексных чисел, заданных в тригонометрической форме, является такое комплексное число, модуль которого равняется
частному модулей делимого и делителя, а аргумент – разности аргументов
делимого и делителя при условии, что заданные числа отличаются от нуля.
Доказательство
Пусть z1 = r1(cos ϕ1 + i sin ϕ1), z2 = r2(cos ϕ2 + i sin ϕ2). Обозначим частное
z
через z . Тогда из равенства z = 1
выходит, что z1 = z⋅ z2, где
z2
z = r ( cos ϕ + i sin ϕ ) или r1(cos ϕ1 + i sin ϕ1) = r⋅ r2(cos(ϕ + ϕ2) +і sin(ϕ + ϕ2)).
Из равенства комплексных чисел выходит, что
 r1 = r ⋅ r2 ;

ϕ1 = ϕ + ϕ2 ,
поэтому с точностью до 2nπ, где n ∈ N, имеем r =
r1
; ϕ = ϕ1 – ϕ2.
r2
Следовательно, получена формула
r1 ( cos ϕ 1+ i sin ϕ 1 )
r2 ( cos ϕ 2 + i sin ϕ 2 )
=
r1
( cos ( ϕ 1− ϕ 2 ) + i sin ( ϕ 1− ϕ 2 ) )
r2
(3.5)
3.5 Извлечение корня из комплексного числа. Вторая формула Муавра
Теорема Любое комплексное число, заданное в тригонометрической форме и отличное от 0, имеет n значений корня n-й степени, определяемых по
формуле
ϕ + 2πk
ϕ + 2πk 
n r ( cos ϕ + i sin ϕ ) = n r  cos
+ i sin

,
n
n 

где n ∈ N; k = 0, 1, 2 …, n – 1.
Доказательство
Пусть z = r (cos ϕ + i sin ϕ) ≠ 0.
Допустим, что существует корень n-й степени из комплексного числа z и
обозначим его ω = ρ(cos θ + isin θ). Тогда
n r ( cos ϕ + i sin ϕ ) = ρ ( cos θ + i sin θ ) .
Если обе части этого равенства возвести в степень n, то по первой формуле
Муавра получим равенство
r ( cos ϕ + i sin ϕ ) = ρn ( cos nθ + i sin nθ ) ,
из которого выходит
31
n
1) равенство модулей: r = ρ , откуда
ρ= n r;
(3.6)
2) равенство аргументов с точностью до числа, кратного 2π: ϕ + 2kπ = n θ,
где к ∈ Z, откуда
ϕ + 2πk
θ=
, где k = 0, 1, 2, …, n – 1.
(3.7)
n
ϕ
В частности, если к = 0,
то θ = ;
n
ϕ 2π
если k = 1,
то θ = +
;
n n
ϕ 2π
если k = 2,
то θ = +
2;
n n
..................................................................
ϕ 2π
если k = n – 1, то θ = +
⋅ ( n − 1) .
n n
Если же допустить, что к = n, к = n + 1 ..., а также если к = –1, k = –2,
k = 3 ..., то будем получать значения θ, которые отличаются от уже найденных
на углы, кратные 2π. То есть никаких новых значений корня, кроме уже найденных n значений, не будет. Значит,
ϕ + 2πk
ϕ + 2πk 
n r ( cos ϕ + i sin ϕ ) = n r  cos
+ i sin
(3.8)

,
n
n 

где n ∈ N; k = 0, 1, 2 ..., n – 1.
Эта формула называется второй формулой Муавра.
В соответствии с формулой (3.8), корень степени n из комплексного
числа имеет ровно n значений. Каждое из них имеет один и тот же модуль r,
а это значит, что все комплексные числа, равные значениям n z , размещаются на окружности радиуса r с центром в точке О(0; 0). Аргументы этих ком2π
плексных чисел при k = 0, 1, 2, …, n – 1 отличаются на величину k , а люn
2π
. Отсюда выходит,
бые два соседних значения аргументов отличаются на
n
что, если все значения n z разместить на окружноти радиуса r с центром в точке О(0; 0), то расстояние между соседними аффиксами будет одинаковым. А
если построить замкнутую ломаную с вершинами в этих точках, то получим
правильный n-угольник.
3.6 Примеры
Пример 3.1 Записать в тригонометрической форме комплексные
числа z1 = 5 – 5i и z2 = 3 + i. Найти значение выражения z = z12 : z 23 .
32
Решение
Число z1 принадлежит ІV четверти. По формулам (1.2) и (1.10) имеем
π
| z1 |= 5 2 + (−5) 2 = 50 = 5 2 ; arg z1 = arctg (–1) = – arctg 1 = – .
4
Число z2 принадлежит І четверти. Аналогично, по формулам (1.2) и (1.10), имеем
2
π
1
| z2 |=
3 + 12 = 4 = 2 ; arg z2 = arctg
= .
6
3
π
π
  π
 π 

Тогда z1 = 5 2  cos −  + i sin −   ; z 2 = 2 cos + i sin  .
6
6
 4 

  4
Воспользовавшись первой формулой Муавра, получим

  π
 π
 π 
 π 
z12 == (5 2 ) 2  cos 2 −  + i sin 2 −   = 50 cos −  + i sin  −   ;
 4
 4 
 2 

  2

π
π
 π
 π  
z 23 = 23  cos  3 ⋅  + i sin  3 ⋅   = 8  cos + i sin  .
2
2
 6
 6  

По формуле (3.5) находим искомую величину

 π
 π 
50  cos  −  + i sin  −  
2
z
 π π
 π π 
 2
 2   25 
z = 13 = 
=  cos  − −  + i sin  − −   =
π
π

4
z2
 2 2
 2 2 
8  cos + i sin 
2
2

25
= ( cos ( −π ) + i sin ( −π ) ) .
4
Полученный результат нельзя считать комплексным числом в стандартной форме, поскольку аргумент этого числа не удовлетворяет условию (1.6).
Поэтому число z можно задать таким образом:
( )
z=
25
25
25
cos ( −π ) + i sin ( −π ) ) = ( cos ( −π + 2π ) + i sin ( −π + 2π ) ) = ( cosπ + i sinπ ) .
(
4
4
4
В случае необходимости можно перейти к алгебраической форме комплексного числа:
25
25
z = (−1 + i ⋅ 0) = − .
4
4
25 25
О т в е т: z = −
= ( cos π + i sin π ) .
4
4
Пример 3.2
1) Построить многоугольник, вершинами которого являются значения
корня 6 − 64 .
2) Вычислить площадь этого многоугольника.
33
3) Найти сумму всех значений корня.
Решение
1) Запишем заданное число в тригонометрической форме, опираясь на
формулы (1.2) и (1.10):
z = –64 = –64 + 0i; r = (−64) 2 + 0 2 = 64;
Следовательно, –64 = 64(cos π + i sin π).
ϕ = π.
В соответствии с формулой (3.8) находим 6 − 64 :
π + 2πk
π + 2πk 

− 64 = 6 64(cos π + sin π) = 6 64 ⋅  cos
+ i sin
,
6
6


где к = 0, 1, 2, 3, 4, 5.
Находим ρ = 6 64 = 2 .
π
π
π
π


Если к = 0, то ω0 = 2  cos + i sin  ; если k = 1, то ω1 = 2  cos + i sin  ;
6
6
2
2


5π
5π 
7π
7π 


если к = 2, то ω2 = 2  cos + i sin  ; если k = 3, то ω3 = 2  cos + i sin  ;
6
6 
6
6 


3π 
11π 
 3π
 11π
если к = 4, то ω4 = 2  cos + i sin  ; если k = 5, то ω5 = 2 cos
+ i sin
.
2
2
6
6 


Три последних значения следует привести к стандартной форме, поскольку их аргументы выходят за пределы промежутка (–π; π]. Для этого, пользуясь
периодичностью функций, выполним такие преобразования:
Im z
7π
7π 

ω3 = 2 ⋅  cos
+ i sin  =
6
6 
π

ω1 2
  7π

 7π

= 2 ⋅  cos
− 2π  + i sin
− 2π   =
ω0
ω2
π

 6

  6
5π
6
6
  5π 
 5π  
Re z
= 2 ⋅  cos −  + i sin −  ;
 6 
О
  6 
ω5
ω3
3π
3π 

π
–
ω4 = 2 ⋅  cos + i sin  =
– 5π
6
2
2


6
ω4 – π
  3π

 3π

2
= 2 ⋅  cos − 2π  + i sin  − 2π   =
 2


  2
Рисунок 3.1
  π
 π 
= 2  cos −  + i sin  −   ;
 2 
  2
11π
11π 
  11π


 11π

ω5 = 2 ⋅  cos
+ i sin
− 2π  + i sin
− 2π   =
 = 2 ⋅  cos
6
6 


 6

  6
6
  π
 π 
= 2 ⋅  cos −  + i sin  −  .
 6 
  6
34
Отложив на окружости радиуса r = 2 найденные значения корня, построим шестиугольник (рис. 3.1).
2) Построенный шестиугольник является правильным шестиугольником.
Его площадь равняется
1
S = 6 ⋅ 22 sin 60o = 6 3 (ед. кв.).
2
3) Находим сумму всех значений корня:
W = ω0 + ω1 + ω2 + ω3 + ω4 + ω5 =
π
π
π
π
5π
5π

 5π 
+ cos −  +
= 2⋅  cos + i sin + cos + i sin + cos + i sin
6
6
2
2
6
6
 6 

 5π 
 π
 π
 π
 π 
+ i sin −  + cos −  + i sin  −  + cos −  + i sin −   =
 6 
 2
 2
 6
 6 
 3 1
3 1
3 1
3 1
+i +0+i−
+i −
−i +0−i+
− i  = 0 .
= 2 
2
2
2
2
2
2
2
2

О т в е т: S = 6 3 (кв. eд.); W = 0 .
ЗАМЕЧАНИЕ. Сумма всех значений корня из комплексного числа всегда
равняется нулю. Эту информацию можно использовать для проверки найденых
значений корня из комплексного числа.
Пример 3.3 Найти все значения 4 i . Ответ записать в тригонометрической и алгебраической форме.
Решение
Представим число i в тригонометрической форме, пользуясь формулами
(1.2) и (1.10):
π
π
π

i = 0 + 1і⋅; а = 0; b = 1; r = 4 0 2 + 12 = 1; ϕ = arg z = ; i = 1  cos + i sin  .
2
2
2

π
π

Определим число ω = 4 1 cos + i sin  из равенства
2
2

4
π
π

1 cos + i sin  = ρ (cos θ + i sin θ).
2
2

1π

Тогда по формулам (3.6), (3.7) ρ = 4 1 = 1; θ =  + 2πk  , где k = 0, 1, 2, 3.
42

π
π
π
π
π

Если к = 0, то θ0 = ;
ω0 = 1  cos + i sin  = cos + i sin ;
8
8
8
8
8

5π
5π
1π
5π
5π 


если к = 1, то θ1 =  + 2π  ; ω1 = 1  cos + i sin  = cos + i sin ;
8
8 
4 2
8
8


35
9π
9π
1π
9π
9π 


 + 4π  ; ω2 = 1  cos + i sin  = cos + i sin ;
8
8 
4 2
8
8


1π
13π
13π 
13π
13π


если к = 3, то θ3 =  + 6π  ; ω3 = 1  cos
+ i sin
 = cos + i sin .
8
8 
4 2
8
8


Найденые значения ω2 и ω3 записаны в нестандартной форме, так как аргумент
выходит за пределы промежутка (–π; π]. Пользуясь периодичностью функций
sin x и cos x, выполним необходимые преобразования.
9π
9π 

 9π

 9π

 7π 
 7π 
ω2 = cos + i sin = cos − 2π  + i sin − 2π  = cos −  + i sin −
;
8
8 

 8

 8

 8 
 8 
если к = 2, то θ2 =
13π
13π   13π

 3π 
 3π 

 13π

+ i sin
ω3 = cos
− 2π  + i sin 
− 2π  = cos −  + i sin − .
= cos
8
8   8

 8
 8

 8

В алгебраической форме имеем ω0 ≈ 0,92 + 0,38 і ; ω1 ≈ –0,38 +
0,92 і ; ω2 ≈ –0,92 – 0,38і; ω3 ≈ 0,38 – 0,92і.
π
π
5π
5π
9π
9π
О т в е т: ω0 = cos + i sin ; ω1 = cos + i sin ; ω2 = cos + i sin ;
8
8
8
8
8
8
13π
13π
ω3 = cos + i sin ; ω0 ≈ 0,92 + 0,38і; ω1 ≈ –0,38 + 0,92і; ω2 ≈ –0,92 – 0,38і;
8
8
ω3 ≈ 0,38 – 0,92і.
36
4 ДЕЙСТВИЯ НАД КОМПЛЕКСНЫМИ ЧИСЛАМИ,
ЗАДАННЫМИ В ПОКАЗАТЕЛЬНОЙ ФОРМЕ
4.1 Показательная форма комплексного числа
Весьма часто в математике и ее приложениях применяется выражение
cos ϕ + i sin ϕ. Для этого выражения используется сокращенное обозначение еіϕ
или ехр іϕ,что называется мнимой экспонентой. При этом имеет место равенство
еіϕ = cos ϕ + i sin ϕ,
(4.1)
которое будет доказано при дальнейшем изучении высшей математики.
Пусть
z = r (cos ϕ + i sin ϕ).
Тогда комплексное число z можно записать в форме
z = r еіϕ.
(4.2)
Число, заданное в виде (4.2), называется комплексным числом в показательной форме.
О п р е д е л е н и е. Два комплексных числа z1 = r1 eiϕ1 и z2 = r2 eiϕ2 называются равными между собой, если их модули равны между собой, их аргументы либо равны между собой, либо отличаются на число, кратное 2π, то есть
r1 = r2 и ϕ1 = ϕ2 + 2πn, где n ∈ Z.
іϕ
ЗАМЕЧАНИЕ. В технических приложениях функцию r е иногда обозначают символом cis ϕ = r еіϕ и называют сисоидной функцией. Поскольку
cis ϕ = r (cos ϕ + i sin ϕ), то это объясняет термин сисоидная как сокращение от
терминов косинус и синус.
Над комплексными числами, заданными в показательной форме, удобно
выполнять такие алгебраические действия, как умножение, возведение в
степень, деление, извлечение корня.
4.2 Умножение комплексных чисел
Теорема Произведением двух комплексных чисел, заданных в показательной форме, является такое комплексное число, модуль которого равен произведению модулей сомножителей, а аргумент – сумме аргументов сомножителей.
Доказательство
Пусть z1 = r1 eiϕ1 ; z2 = r2 eiϕ2 . Запишем эти числа в тригонометрической
форме
z1 = r1(cos ϕ1 + і sin ϕ1 ); z2 = r2(cos ϕ2 + і sin ϕ2 ).
Опираясь на формулу (3.3), имеем
z1 z2 = r1 eiϕ1 r2 eiϕ2 = r1(cos ϕ1 + і sin ϕ1 )r2(cos ϕ2 + і sin ϕ2 ) =
i ( ϕ +ϕ )
= r1r2(cos( ϕ1 + ϕ2 ) + і sin( ϕ1 + ϕ2 )) = r1r2 е 1 2 .
37
Следовательно.
r1 eiϕ1 r2 eiϕ2 = r1r2 еi ( ϕ1 +ϕ2 ) .
(4.3)
4.3 Возведение в степень комплексных чисел
Пусть z = rеіϕ. Найдем zn, где n ∈ N.
n
n
zn = ( zeiϕ ) = ( r (cos ϕ + i sin ϕ) ) .
Опираясь на первую формулу Муавра, получим результат
zn = r n (cos nϕ + i sin nϕ) = r n einϕ ,
откуда
( re )
iϕ
n
= r neinϕ .
(4.4)
Получена первая формула Муавра в показательной форме.
4.4 Деление комплексных чисел
Теорема. Частным двух комплексных чисел, заданных в показательной
форме, является такое комплексное число, модуль которого равен частному
модулей делимого и делителя, а аргумент – разности аргументов делимого и
делителя при условии, что заданные числа отличны от нуля.
Доказательство
Пусть z1 = r1eiϕ1 , z2 = r2eiϕ 2 . Запишем эти числа в тригонометрической
форме
z1 = r1(cos ϕ1 + і sin ϕ1), z2 = r2(cos ϕ2 + і sin ϕ2).
Опираясь на формулу (3.5), получим
z1 r1eiϕ1 r1 (cos ϕ1 + i sin ϕ1 ) r1
r
= iϕ2 =
= (cos(ϕ1 − ϕ2 ) + i sin(ϕ1 − ϕ2 )) = 1 ei ( ϕ1 −ϕ2 ) .
z2 r2e
r2 (cos ϕ1 + i sin ϕ1 ) r2
r2
Окончательно имеем
r1eiϕ1 r1 i ( ϕ1 −ϕ2 ) .
(4.5)
= e
iϕ2
r2e
r2
4.5 Извлечение корня из комплексного числа
Теорема Любое комплексное число, заданное в показательной форме и
отличное от нуля, имеет n значений корня n-й степени, определяемых по формуле
n
iϕ
n
i ( ϕ + 2 πk )
re n
re =
, где k = 0, 1, 2, ..., n – 1.
Доказательство
Пусть z = rеіϕ. Запишем это число в тригонометрической форме
z = r(cos ϕ + i sin ϕ ). Опираясь на формулу (3.8), получим
38
n
ϕ + 2πk
ϕ + 2πk  n

z = r e = r ( cos ϕ + i sin ϕ ) = r  cos
+ i sin
= re
n
n 

где n ∈ N; k = 0, 1, 2, ..., n – 1.
Окончательно выходит
n
iϕ
n
n
n
iϕ
n
i ( ϕ+ 2 πk )
n
re = r e
,
где n ∈N; k = 0, 1, 2, ..., n – 1.
Получена вторая формула Муавра в показательной форме.
i ( ϕ+ 2 πk )
n
,
(4.6)
4.6 Формулы Эйлера
Равенство (4.1) называется формулой Эйлера. На основании этой формулы можно получить еще несколько важных формул.
Например, если в равенстве (4.1) аргумент ϕ заменить на –ϕ, то придем к
равенству
е–іϕ = cos ϕ – i sin ϕ,
(4.7)
которое также называется формулой Эйлера. Если равенства (4.1) и (4.7) почленно сложить, то получим
еіϕ + е–іϕ = cos ϕ + i sin ϕ + cos ϕ – i sin ϕ = 2cos ϕ,
откуда
1
cos ϕ = (еіϕ + е–іϕ).
(4.8)
2
Аналогично, почленно вычитая равенство (4.7) от равенства (4.1), получим
еіϕ – е–іϕ = cos ϕ + i sin ϕ – cos ϕ + i sin ϕ = 2і sin ϕ,
откуда
1
sin ϕ = (еіϕ – е–іϕ).
(4.9)
2i
1
i
i
i
Так как
= 2 = − , то sin ϕ = − (еіϕ – е−іϕ) или
2
2i 2i
2
i
sin ϕ = (е–іϕ – еіϕ).
(4.10)
2
4.7 Примеры
Пример 4.1 Записать в тригонометрической и показательной форме
комплексное число z = – 3 – i и построить соответствующий комплексный
вектор.
Решение
Так как комплексное число z = – 3 – i заданно в алгебраической
форме, то а = 3 ; b = –1. Для того, чтобы комплексное число записать в тригонометрической или показательной форме, следует найти модуль и аргумент
комплексного числа. По формуле (1.2) находим | z |:
|z|=r=
(− 3)
2
2
+ ( −1) = 4 = 2.
39
Так как число z принадлежит ІІІ четверти, то по формуле (1.10) имеем
π
5π
 −1 
 1 
arg z = –π + arctg 
 = –π + arctg 
 = –π + = – .
6
6
− 3
 3
Тогда
Im z
5π
−i

 5π 
 5π  
z = – 3 – i = 2  cos  −  + i sin  −   = 2e 6 .
 6 
 6 
Re z

− 3
О
Построим комплексный вектор, соответствующий заM
–1
(
)
данному комплексному числу. Точка M − 3; − 1 яв⋅
Рисунок 4.1
ляется аффиксом комплексного числа z, а вектор OM –
его комплексным вектором (рис. 4.1).
5π
−i

 5π 
 5π  
О т в е т: 2  cos  −  + i sin  −   ; 2e 6 .
 6 
 6 

Пример 4.2 Заданы комплексные числа z1 = – 4 – 4і; z2 = 3 – 3 3 і. Опz
ределить 1) z1 z2; 2) 1 ; 3) z14 ; 4) 3 z 2 . Действия выполнить в тригонометриz2
ческой и показательной форме.
Решение
Запишем числа z1 и z2 в тригонометрической и показательной форме. Обратимся к формулам (1.2) и (1.10):
(−4) 2 + (−4) 2 = 4 2 .
Число z1 принадлежит ІІІ четверти, следовательно,
π
3π
−4
ϕ1 = arg z1 = –π + arctg 
 = –π + arctg 1 = –π + = – .
4
4
−4
Тогда
z1 = – 4 – 4і; а1 = –4; b1 = –4; r1 =
3π
−i
  3π 
 3π  
z1 = 4 2  cos −  + i sin −   = 4 2e 4 ;
 4 
  4 
(
z2 = 3 – 3 3 і; а1 = 3; b1 = –3 3 ; r2 = 32 + − 3 3
Число z2 принадлежит ІV четверти. Следовательно,
−3 3
π
 = –arctg 3 = – ;
ϕ2 = arg z2 = arctg 
3
 3 
)
2
= 6.
π
−i

 π
 π 
z2 = 6  cos  −  + i sin  −   = 6e 3 .
 3
 3 

Выполним заданные действия в тригонометрической форме.
40
1) По формуле (3.3) находим z1 z2.
π
π 
3π
3π  

z1 z2 = 4 2  cos  −  + i sin  −   ⋅6  cos −  + i sin  −   =
 3
 3 
 4 
 4  

= 24 2  cos −

3π π 
 3π π   24 2   13π 
 13π  
−  + i sin  −
−  =
 cos  −
 + i sin  −
 =
 4 3
 4 3 
 12  
  12 
= 24 2  cos  2 π −


13 π 
13 π   

 11π 
 11π   .
 + i sin  2 π −
  =  cos 
 + i sin 

12 
12   

 12 
 12  
2) По формуле (3.5) находим
z1
.
z2
  3π 
 3π  
4 2  cos −  + i sin  −  
z1
 3π π  
 4   2 2   3π π 
  4 
=
=
+  + i sin  −
+  =
 cos −
z2
3   4 3
  π
 π 
 4 3 
6 cos −  + i sin −  
 3 
  3
=
2 2   5π 
 5π  
 cos −  + i sin −   .
3   12 
 12  
3) Для нахождения z14 используем первую формулу Муавра:
4

z14 =  4
4

 3π 
 3π   
 (−3π) 
 (−3π)  
2  cos  −  + i sin  −    = 4 2  cos  4
 + i sin  4
 =
4
4
4
4













= 1024(cos(–3π) + i sin(–3π)).
Так как значение arg z не принадлежит промежутку (–π; π], то полученый результат нужно записать в стандартной форме. Воспользовавшись периодичностью тригонометрических функций, получим:
z14 = 1024(cos(4π – 3π) + i sin(4π – 3π)) = 1024(cos π + i sin π).
4) Для нахождения
ω=
3
(
3
)
z 2 используем вторую формулу Муавра:


1 π
1 π
 π
 π 


6 cos  −  + i sin  −   = 3 6  cos   − + 2 πk   + i sin   − + 2 πk    ,
 3
 3 



3 3
3 3

где k = 0, 1, 2.
  π
 π 
6  cos −  + i sin  −   ;
 9 
  9
5π
5π 

если k = 1, то ω1 = 3 6  cos + i sin  ;
9
9 


1 π
1 π


если k = 2, то ω2 = 3 6  cos  − + 4π  + i sin   − + 4π    =
3 3


 3 3

11π
11π  3   11π


 11π

= 3 6  cos
+ i sin
− 2π  + i sin
− 2π   =
 = 6  cos
9
9 


 9

  9
Если k = 0, то ω0 =
3
41
  7π 
 7π  
= 3 6  cos −  + i sin −   .
 9 
  9 
Теперь выполним те же действия в показательной форме.
1) По формуле (4.3) находим
3π
3π π
13π

 − i π 
−i −i
−i
−i
3
4
3
4




6e
4 2 ⋅ 6e
= 24 2 e 12 =
z1 z2 = 4 2 e


=



11π
i
11π
11π 
  13π 
 13π  

= 24 2  cos −
+ i sin
 + i sin −
  = 24 2  cos
 = 24 2e 12 .
12
12 
 12  

  12 
2) По формуле (4.5) находим
−i
3π
4
 3π π 
5π
z1
4 2e
4 2 i  − 4 + 3  2 2 −i 12
=
e
=
e
.
=
π
−i
6
3
z2
6e 3
3) По формуле (4.4) находим
4
3π
3π


−i
− i ⋅4
4
4
4
 = (4 2 ) e 4 = 1024e − i 3π =
z1 =  4 2 e




= 1024(cos(–13π) + isin(–13π)) = 1024(cos π + isin π) = 1024eiπ.
4) По формуле (4.6) имеем
3
z2 =
3
6e
−i
π
3
3
= 6e
1 π

i  − + 2 πk 
3 3

Если k = 0, то ω0 =
если k = 2, то ω2 =
3
3
, где k = 0, 1, 2.
6e
6e
i
−i
π
9
11π
9
; если k = 1, то ω1 =
3
6e
i
5π
9
;
11π
11π 

= 3 6  cos
+ i sin
=
9
9


7π
−i
  7π 
 7π  
= 6  cos −  + i sin  −   = 3 6e 9 .
 9 
  9 
11π
i
  11π 
 11π  
О т в е т: 1)  cos
 + i sin 
  ; 24 2e 12 ;
12
12



 
3
5π
2 2   5π 
 5π   2 2 −i 12
2)
e ; 3) 1024(cos π + isin π); 1024eiπ;
 cos −  + i sin −   ;
3   12 
3
 12  
4) ω0 =
3
  π
 π 
6  cos −  + i sin  −   ; ω1 =
 9 
  9
3
  7π 
 7π  
ω2 = 6  cos −  + i sin −   ; ω0 =
 9 
  9 
3
5π
5π 

6  cos + i sin  ;
9
9 

3
6e
−i
π
9
; ω1 =
3
6e
i
5π
9
; ω2 =
3
6e
−i
7π
9
.
42
Пример 4.3 Заданы комплексные числа z1 = 3 – 5i и z2 = –6 + 2i. Найти
( )
комплексное число z , если z = 3 z1
( 5 z2 ) . Действия выполнить: 1) в алгеб-
раической форме; 2) в тригонометрической форме; 3) в показательной форме.
Решение
1) Если z1 = 3 – 5i, то z1 = 3 + 5i.
Тогда 3 z1 = 9 + 15 i; 5z2 = –30 + 10 i, откуда z = (9 + 15 i)(–30 + 10 i) =
2
= –270 – 450і + 90 і + 150 і = –270 – 360 і – 150 = –420 – 360 і.
Следовательно, z = –420 – 360 і, а z = –420 + 360 і.
2) Запишем заданные комплексные числа в тригонометрической форме:
|z1| = 32 + (−5) 2 = 34 ; |z2| = (−6) 2 + 2 2 = 40 .
Комплексное число z1 принадлежит ІV четверти, а z2 – ІІ четверти. Следоb
5
 5
≈
вательно, arg z1 = arctg
, то есть arg z1 = arctg  −  = –arctg
a
3
 3
≈ –59,03624347°.
Аналогично:
1
1
2
 2
arg z2 = π + arctg  −  = π – arctg   = π – arctg
≈
= 180° – arctg
3
3
 6
6
≈ 161,5650512°.
Тогда z1 ≈ 34 (cos(–59,03624347°) + i sin(–59,03624347°));
z2 ≈ 40 (cos 161,5650512° + i sin 161,5650512°).
Далее находим z1 . Поскольку arg z = –arg z ,то
z1 ≈
34 (cos 59,03624347° + i sin 59,03624347°).
Представим число 3 в виде 3 = 3 + 0 i = 3(cos 0° + i sin 0°), тогда
3 z1 ≈ 3 34 (cos(0° + 59,03624347°)+ i sin(0 + 59,03624347)) =
= 3 34 (cos 59,03624347° + i sin 59,03624347°).
Аналогично: 5 = 5(cos 0° + i sin 0);
5z2 = 5 40 (cos(0° + 161,5650512°)+ i sin(0° + 161,5650512°)) =
=5 40 (cos 161,5650512° + i sin 161,5650512°).
Теперь находим z.
z ≈ (3 z1 ) (5z2) = 3 34 (cos 59,03624347° + i sin 59,03624347)×
×5 40 (cos 161,5650512° + i sin 161,5650512) =
= 15 1360 (cos(59,03624347° + i sin 59,03624347)×
×(cos 161,5650512° + i sin 161,5650512°) ≈
≈ 553,1726674(cos 220,6012947° + i sin 220,6012947).
43
Поскольку arg z выходит за пределы промежутка (–180°; 180°], то, пользуясь периодичностью тригонометрических функций, представим результат в
виде
z ≈ 553,1726674(cos(–139,3987053°)+ i sin (–139,3987053°)).
Отсюда имеем
z ≈ 553,1726674(cos 139,3987053° + i sin 139,3987053).
Для проверки полученый результат переведем в алгебраическую форму:
z ≈ 553,1726674(–0,759256601 + i 0,650791374)
или
z = –420 + 360 і.
3) Теперь выполним указанные действия в показательной форме.
o
Из предыдущего решения выходит , что z 1 ≈ 34 е −59,03624347 i ;
o
o
z1 ≈ 34 е59,03624347 i ; z2 ≈ 40 е161,5650512 i ; 3 = 3е0і, 5 = 5е0і.
o
o
Отсюда 3 z1 ≈ 3 34 е59,03624347 i ; 5z2 ≈ 5 40 е161,5650512 i ,
o +161,5650512o ) i
z = (3 z1 ) (5z2) = 3 34 е(59,03624347
o
= 3 34e220,60129471 i =
= 3 34(cos220,6012947o + i sin220,6012947o ) =
o
o
= 3 34(cos( −139,3987053o ) + i sin( −139,3987053o )e220,60129471 i =553,172667 е −139,3987053 i .
Тогда
o
z ≈ 553,1726674 е139,3987053 i .
О т в е т: z ≈ 553,1726674(cos 139,3987053° + i sin 139,3987053)
139o3987053i
z ≈ 553,1726674 е ,
.
Пример 4.4 Решить уравнение z5 – 243 = 0. Найденые значения корней
записать в тригонометрической, показательной и алгебраической формах. В окружность радиуса r = 3 вписать правильный пятиугольник.
Решение
Из уравнения z5 – 243 = 0 имеем: z5 = 243 или 5 z 5 = 5 243 . Тогда z = 5 243 .
Поскольку решение уравнения мы ищем в комплексной области, то существует
пять значений 5 243 . Найдем все корни уравнения. Для этого число 243 представим в виде 243 = 243 + 0і. Это число в тригонометрической форме имеет вид
243 = 243(cos 0 +i sin 0). Тогда
5
243 = 5 243(cos 0 + i sin 0) = ρ (cos θ + i sin θ), откуда по формулам
0 + 2πk
(3.6) и (3.7) ρ = 5 243 = 3; θ =
, где k = 0, 1, 2, 3, 4.
5
0
Если k = 0, то θ0 = = 0, а ω0 = 3(cos 0 +i sin 0);
5
2π
2π
2π 

если k = 1, то θ1 =
,
а ω1 = 3  cos
+ i sin  ;
5
5
5 

44
4π
4π 

ω2 = 3  cos
+ i sin  ;
5
5 

6π
6π 

если k = 3, то
а ω3 = 3  cos + i sin  =
5
5 

  6π
  4π 

 6π

 4π  
= 3  cos − 2π  + i sin − 2π   = 3  cos −  + i sin −   ;

 5

 5 
  5
  5 
8π
8π
8π 

если k = 4, то θ4 =
,
а
ω4 = 3  cos + i sin  =
5
5
5 

  8π
  2π 

 8π

 2π  
= 3  cos − 2π  + i sin  − 2π   = 3  cos −  + i sin −   .

 5

 5 
  5
  5 
Найдены все корни уравнения. Запишем их в показательной и алгебраической форме:
если k = 2, то
0i
4π
,
5
6π
θ3 =
,
5
θ2 =
ω0 = 3 е ; ω1 = 3 е
2π
i
5
; ω2 = 3 е
а
4π
i
5
; ω3 = 3 е
−
4π
i
5
; ω4 = 3 е
−
2π
i
5
или
ω0 ≈ 3;
ω1 ≈ 3(0,30901699 + 0,95105651і) = 0,92705097 + 2,85316953і;
ω2 ≈ 3(–0,80901699 + 0,58778525і) = –2,42705097 + 1,76335575і;
ω3 ≈ 3(–0,80901699 – 0,58778525і) = –2,42705097 – 1,76335575і;
ω4 = 3(0,30901699 – 0,95105651 і) = 0,92705097 – 2,85316953 і.
Для того, чтобы в окружность радиуса
Im z
r = 3 вписать правильный пятиугольник,
можно построить на комплексной плоскости
ω1
2,85
ω2
аффиксы корней ω0, ω1, ω2, ω3, ω4 (рис. 4.2).
1,76
Аффиксы этих комплексных чисел принад3 Re z
лежат окружности радиуса r = 3, и любые два
ω0
0,9
–2,42 0
соседних аффикса находятся на одинаковом
–1,76
ω3
расстоянии один от другого. Следовательно,
–2,85
эти точки можно принять за вершины праω4
вильного пятиугольника.
Рисунок 4.2
Пример 4.5 Найти корни уравнения z3 + 7 = 0. Ответы записать в тригонометрической, показательной и алгебраической форме.
Решение
Из уравнения z3 + 7 = 0 имеем z3 = –7, а z = 3 − 7 .
Запишем число –7 в тригонометрической форме:
–7 = –7 +0і = 7(cos π + і sin π).
45
3
7(cos π + i sin π) = ρ (cos θ + i sin θ), где ρ =
Если k = 0, то
θ0 =
если k = 1, то
θ1 =
если k = 2, то
=
3
π
,
3
3
ω0 =
а
π + 2πk
, k = 0, 1, 2.
3
π
π

7  cos + i sin  ;
3
3

7;θ=
3
π + 2π
= π,
а ω1 = 3 7 (cos π + i sin π) ;
3
π + 4 π 5π
5π
5π 

θ2 =
=
, а ω2 = 3 7  cos + i sin  =
3
3 
3
3

  5π
  π

 5π

 π 
7  cos − 2π  + i sin  − 2π   = 3 7  cos −  + i sin −   .

 3

 3 
  3
  3
Значение корней в показательной форме:
π
3
3
i
π
3
πi
3
− i
3
ω0 = 7 e ; ω1 = 7 e ; ω2 = 7 e .
Значение корней в алгебраической форме:
ω0 ≈ 1,91293118(0,5 + 0,86602540і) ≈ 0,9564656 + 1,65664699і;
ω1 ≈ 1,91293118(–1 + 0 і) = –1,91293118;
ω2 ≈ 1,91293118(0,5 – 0,86602540і) ≈ 0,9564656 + 1,65664699і.
π
π

О т в е т : ω 0 = 3 7  cos + i sin  ; ω 1 = 3 7 (cos π + i sin π) ;
3
3

π
π
i
− i
  π
 π 
πi
3
3
3
3
3
ω2 = 7  cos −  + i sin −   ; ω0 = 7 e ; ω1 = 7 e ; ω2 = 7 e 3 ;
 3 
  3
ω0 ≈0,9564656 + 1,65664699і; ω1 ≈–1,91293118; ω2 ≈ 0,9564656 + 1,65664699і.
46
5 ВЕКТОРНАЯ ХАРАКТЕРИСТИКА ОПЕРАЦИЙ
НАД КОМПЛЕКСНЫМИ ЧИСЛАМИ
5.1 Сложение комплексных чисел
Пусть комплексные числа z 1 = a 1 + b 1 i и z2 = a2 + b2i являются комплексными координатами соответсвующих комIm z
плексных векторов
ОМ 1 = {a1; b1} и ОМ 2 = {a2; b2} (рис. 5.1).
M
Найдём комплексный вектор с комплексной координаM2
той z1 + z2. Для этого на векторах ОМ 1 и ОМ 2 как
на сторонах построим параллелограмм ОМ1ММ2. В
M1
Re z соответствии с правилом параллелограмма, вектор
ОМ является суммой векторов ОМ 1 и ОМ 2 .
О
Тогда ОМ = {a1 + a2; b1 + b2},
Рисунок 5.1
а
z1 + z2 = (a1 + a2) + (b1 + b2) i.
Правило сложения комплексных чисел в векторной
форме
Для того, чтобы найти сумму двух комплексных чисел, заданных своими
комплексными векторами OM1 и OM 2 нужно на этих векторах, как на сторонах построить параллелограмм, после чего по правилу параллелограмма определить сумму OM заданных комплексных векторов.
Рассмотрим свойство модуля суммы комплексних чисел, для чего обратися к рисунку 5.1 На стороне М2М треугольника ОМ2М построим вектор
М 2 М = ОМ 1 . По свойству сторон треугольника имеем | ОМ | ≤ | ОМ 2 | + | М 2 М |
или
|z1 + z2| ≤ |z1| + |z2|.
(5.1)
5.2 Вычитание комплексных чисел
Пусть комплексные числа z 1 = a 1 + b 1 i и
Im z
z2 = a2 + b2i являются комплексними координатами
M2
соответствующих комплексних векторов
ОМ 1 = {a1; b1} и ОМ 2 = = {a2; b2} (рис. 5.2).
M1
Найдём комплексный вектор, соответствующий комRe z
плексному числу z1 – z2. Для этого построим векторы
О
ОМ 1 и ОМ 2 . Вектор М 2 М 1 , найденный по правилу
M
треугольника, является разностью векторов ОМ 1 и M 2′
ОМ 2 .
Рисунок 5.2
Выполним дополнительное построение. На
47
прямой ОМ2 построим вектор OM 2′ = – ОМ 2 . На векторах ОМ 1 и ОМ 2′ как на
сторонах построим параллелограмм ОМ 2′ MM 1 . На диагонали этого параллелограмма лежит вектор
Ì 22 ) == ОМ 1 − ОМ 2 .
ОМ = ОМ 1 + OM 2′ = ОМ 1 + (–−ÎОМ
Значит,
z1 – z2 = ОМ = {a1 – a2; b1 – b2}.
Правило вычитания комплексных чисел в векторной
форме
Для того, чтобы найти разность двух комплексних чисел, заданных
своими комплексными векторами, нужно на векторах ОМ 1 и ОМ 2′ = – ОМ 2
построить параллелограмм. По правилу параллелограмма найти сумму векторов ОМ 1 и ОМ 2′ , которая равна разности векторов ОМ 1 и ОМ 2 .
Рассмотрим свойство модуля разности комплексних чисел.
Обратимся к рисунку 5.2. Из параллелограмма ОММ1М2 выходит, что
ОМ = М 2 М 1 . Из треугольника ОМ1М2, согласно свойству сторон треугольника,
имеем
| М 2 М 1 | ≥ | ОМ 1 | − | ОМ 2 | , | ОМ | ≥ | ОМ 1 | − | ОМ 2 | .
Поскольку каждому из векторов в последнем неравенстве отвечает определённое комплексное число, приходим к выводу, что является справедливым
неравенство
|z1 – z2| ≥ |z1| – |z2|.
Im z
5.3 Умножение комплексных чисел
ϕ1 + ϕ2
ϕ2
M
Пусть комплексным числам z1 = r1(cos ϕ1 + і sin ϕ1) и
z2 = r2(cos ϕ1 + і sin ϕ2) отвечают комплексные векторы
M2
ϕ1
(5.2)
M1
Re z
О
Рисунок 5.3
ОМ 1 и ОМ 2 (рис. 5.3). Найдём комплексный вектор с
комплексной координатой z1 ⋅ z2. В соответствии с
формулой (3.3) имеем:
z1 ⋅ z2 = r1r2 ( cos ( ϕ1 + ϕ2 ) + i sin ( ϕ1 + ϕ2 ) ) .
Поэтому
|z1 ⋅ z2| = r1 ⋅ r2;
arg (z1 ⋅ z2) = ϕ1 + ϕ2.
Правило умножения комплексных чисел в векторной
форме
Для построения вектора ОМ = z1 ⋅ z2 нужно выполнить такие преобразования:
1) вектор ОМ 1 повернуть против часовой стрелки на угол ϕ2 и провести
луч под полученным углом ϕ1 + ϕ2 ;
48
2) на этом луче построить вектор ОМ такой, что | ОМ |= r1 ⋅ r2;
3) найденный комплексный вектор считать за вектор, который отвечает комплексному числу z1 ⋅ z2.
Пусть z = reiϕ– любое комплексное число, а ОМ – его комплексный вектор. Рассмотрим некоторые специальные комплексные множители, которые
достаточно часто используются в разнообразных электротехнических приложениях.
i
π
1. При умножении комплексного числа z на комплексное число z1 = e 2
происходит вращение комплексного вектора ОМ , соответствующего комπ
плексному числу z , на угол ϕ1 = против часовой стрелки.
2
i
π
2
Комплексное число z1 = e называется оператором вращения, который
заданый вектор ОМ переводит в перпендикулярный ему вектор ОМ 1 , при
этом | ОМ 1 | = | ОМ |.
2. При умножении комплексного числа z на комплексное число z2 = e iπ
происходит поворот комплексного вектора ОМ , отвечающего комплексному
числу z , на угол ϕ2 = π против часовой стрелки.
Комплексне число z2 = eiπ называєтся оператором вращения, который
переводит заданный вектор ОМ в противоположный ему вектор ОМ 1 = – ОМ ,
при этом | ОМ 1 | = | ОМ |.
3. При умножении комплексного числа z на комплексное число z3 = r3 ei 0 = r3,
где r3 > 1, происходит растяжение вектора ОМ , соответствующего комплексному числу z , в r3 раз.
Комплексное число z3 = r3 ( r3 > 1) называется оператором растяжения,
который вектор ОМ переводит в сонаправленный вектор ОМ 3 , при этом
| ОМ 3 | = r3 | ОМ |.
4. При умножении комплексного числа z на комплексное число z4 = r4 ei 0 = r4,
где r4 < 1, происходит сжатие вектора ОМ , соответствующего комплексному
числу z , в r4 раз.
Комплексное число z4 = r4 называется оператором сжатия, который переводит вектор ОМ в сонаправленный вектор ОМ 4 , при этом | ОМ 4 | = r4 | ОМ |.
49
5.4 Деление комплексних чисел
Пусть комплексным числам z1 = r1(cos ϕ1 + i sin ϕ1)
и z2 = r2(cos ϕ1 + i sin ϕ2) отвечают комплексные векторы
ОМ 1 и ОМ 2 (рис. 5.4). Найдём комплексный вектор,
z
соответствующий комплексному числу 1 . В соответстz2
О
вии с формулой (3.5) имеем ,
z1 r1
= ( cos ( ϕ1 − ϕ2 ) + i sin ( ϕ1 − ϕ2 ) ) .
z2 r2
Поэтому
z 
z1 r1
= ; arg  1  = ϕ1 – ϕ2.
z 2 r2
 z2 
Im z
ϕ1
ϕ2
M1
M2
ϕ1 − ϕ2
Re z
M
Рисунок 5.4
Правило деления комплексных чисел в векторной
форме
z
Для построения комплексного вектора ОМ = 1 нужно выполнить таz2
кие преобразования:
1) вектор ОМ 1 повернуть по часовой стрелке на угол ϕ2 и провести луч
под. полученным углом (ϕ1 – ϕ2);
r
2) на этом луче построить вектор ОМ , такой, что Î Ì = 1 ;
r2
3) полученный комплексный вектор ОМ считать вектором, который
z
соответствует комплексному числу 1 .
z2
Im z
О
–3
Re z
4
M2
–3
M1
–4 3
M
Рисунок 5.5
5.5 Примеры
Пример 5.1 Заданы комплексные числа z1 =
4 – 4 3 і и z2 = –3 – 3і. Найти: 1) z1 + z2; 2) z1 –z2.
Действия выполнить в геометрической форме.
Решение
1) Построим комплексные векторы ОМ 1 и
ОМ 2 , соответствующие комплексным числам z1 и
z2 (рис. 5.5). На этих векторах как на сторонах построим параллелограмм ОМ1ММ2. Тогда вектор
ОМ является комплексным вектором комплексного числа z1 + z2:
50
ОМ 1 = {4; –4 3 }, ОМ 2 = {−3; –3},
то есть
ОМ = ОМ 1 + ОМ 2 = {4 – 3; –4 3 – 3} =
= {1; –4 3 – 3}
или
z1 + z2 = 1 + (–4 3 – 3)і.
2) Построим комплексные векторы
ОМ 1 и ОМ 2 , соответствующие комплексным числам z1 и z2 (рис. 5.6). Далее построим вектор OM 2′ = – ОМ 2 . На векто-
Im z
M 2′
О
–3
4
M2
–3
Re z
7
M
рах ОМ 1 и OM 2′ как на сторонах построим
параллелограмм ОМ 2′ ММ 1 . При этом вектор ОМ является комплексным вектором
комплексного числа z1 – z2:
ОМ 1 = {4; –4 3 }; ОМ 2 = {3; –3};
OM 2′ = {3; 3}.
–4 3
M1
Рисунок 5.6
ОМ = ОМ 1 + ОМ 2′ = {4 + 3; –4 3 + 3}=
= {7; –4 3 + 3}
или
z1 – z2 = 7 + (3 – 4 3 )i.
Пример 5.2 Заданы комплексные числа z1 = –4 + 4і и z2 = 2 + 2і. Найти
z
1) z1 ⋅ z2; 2) 1 . Действия выполнить в геометрической форме.
z2
Решение
1) Построим комплексные векторы ОМ 1 и ОМ 2 , соответствующие ком-
плексным числам z1 и z2 (рис. 5.7). Далее вектор ОМ 1 повернём на угол ϕ2 в направлении против часовой стрелки.Под полученным углом ϕ1 + ϕ2 проводим
луч. В этом случае луч попадёт на отрицательную часть действительной оси. На
луче построим вектор ОМ , модуль котрого равен | ОМ 1 || ОМ 2 |. Так как
| ОМ 1 | = 4
2 > 1, | ОМ 2 | = 2 2 > 1, то отрезок | ОМ 1 | нужно растянуть в 2 2
раза. Значит, ОМ = {–16; 0}, так как 4 2 ⋅ 2 2 = 16 , то есть z1 ⋅ z2 = –16.
51
Im z
M1
Im z
M1
4
2
ϕ1
M2
ϕ2
M
–16
–4
О
4
M
ϕ1
M2
ϕ2
Re z
2
Рисунок 5.7
–4
О
Re z
2
Рисунок 5.8
2) Построим комплексные векторы ОМ 1 и ОМ 2 , соответствующие ком-
плексным числам z1 и z2 (рис. 5.8). Затем вектор ОМ 1 повернём на угол ϕ2 по
часовой стрелке. Под полученным углом (ϕ1 – ϕ2) проведём луч. Этот луч совпадёт с положительной частью мнимой оси. На луче построим вектор ОМ , модуль которого равен
OM 1
OM 2
. Так как | ОМ 2 | = 2 2 >1, то отрезок | ОМ 1 | нужно
сжать в 2 2 раза.
Следовательно ОМ = {0; 2} или
z1
= 2 і.
z2
52
6 НЕКОТОРЫЕ ПРИЛОЖЕНИЯ КОМПЛЕКСНЫХ ЧИСЕЛ
В ТЕОРИИ ЭЛЕКТРИЧЕСКИХ ЦЕПЕЙ
6.1 Электромагнитные процессы,
описываемые синусоидными функциями
Методы элементарной алгебры позволяют достаточно просто исследовать
электрические цепи постоянного тока.
Исследование электрических цепей переменного тока осуществляется с
помощью комплексных чисел.
Рассмотрим периодический электромагнитный процесс в электрической
цепи. Период этого процесса обозначим через Т. Тогда величина
1
f=
(6.1)
Т
называется циклической частотой колебаний в электрической цепи. Частота
определяет количество периодов за единицу времени. Измеряется частота в герцах (Гц).
Основные характеристики большинства периодических процессов в электрических цепях такие, как ток, напряжение, электродвижущая сила и др., могут быть описаны с помощью тригонометрических функций - синуса или косинуса.
Например, напряжение можно представить в виде
u = um sin(ωt + ψ),
(6.2)
где um – амплитуда колебаний; ω – угловая частота, то есть скорость изменения аргумента (угла).
Угловая скорость определяется формулой
ω=2πf
(6.3)
и измеряется в радианах в секунду (рад/с). Величина ψ называется начальной
фазой. Начальная фаза характеризует смещение синусоиды относительно начала координат. Угол ψ измеряется в радианах.
За аргумент функции (6.2) принимают или время t, если процесс Тпериодический, или угол ωt, если процесс является 2π-периодическим. При
этом следует иметь в виду, что угол ωt измеряется в радианах, как и начальная
фаза. Если же начальную фазу задавать в градусах, то и аргумент t также нужно
перевести в градусы, период в таком случае равняется 360°.
Величина ωt + ψ называется фазовым углом или фазой.
Иногда зависимость (6.2) записывают в виде
u = umсоs(ω t + ψ1),
π
где ψ1 = ψ – .
2
Аналогично, с помощью тригонометрических функций можно описать
ток, электродвижущую силу и другие характеристики электрической цепи.
Так, сила тока описывается формулой
Ι = Іm sin (ω t + ψ).
(6.4)
53
6.2 Основные понятия о методе комплексных амплитуд
Для расчета электрических цепей,
которые не принадлежат к самым простым, используется так называемый
метод комплексных амплитуд.
Ознакомимся с главной идеей
этого метода.
Im z
•
Am
Am
ωt
•
Пусть
•
Am e jωt
ψ
– некоторое ком-
Re z
О
•
плексное число, модуль | Am | которого
•
равняется амплитуде А0, а arg Am равняется начальной фазе ψ синусоидной
функции Amsin(ω t + ψ), когда
Рисунок 6.1
t = 0 (рис. 6.1).
ЗАМЕЧАНИЕ. Комплексное число z в некоторых формулах теории электрических цепей иногда обозначают символом z& .
•
Комплексное число Am , заданное в виде
•
•
Am = Amеjψ или Am = Am (соs ψ + j sin ψ),
называется комплексной амплитудой колебания. Ему соответствует ком•
плексный вектор Am = {Amсоs ψ; Amsin ψ}.
ЗАМЕЧАНИЕ. В некоторых технических приложениях комплексный вектор
обозначают тем самым символом, что и комплексное число, то есть z& .
•
Умножим комплексное число Am на комплексное число e jωt . Получим
•
Am e jωt = Am e j ( ωt +ψ )
(6.5)
или
•
Am e jωt = Am(cos(ω t + ψ)+ j sin( t + ψ )).
(6.6)
Очевидно,
•
Amsin(ω t + ψ) = Im Am e jωt ;
(6.7)
•
Amcos(ω t + ψ) = Re Am e jωt .
(6.8)
Для исследования характеристик электрических цепей можно использовать как функцию (6.7), так и функцию (6.8).
•
На комплексный вектор Am множитель e jωt повлиял как оператор вращения на угол ωt. В результате этой операции получим комплексный вектор, соответствующий значению синусоидной функции в момент времени t:
54
•
Am e jωt = {Amсоs(ω t + ψ); Amsin(ω t + ψ}.
(6.9)
ЗАМЕЧАНИЕ. В технических приложениях действительная и мнимая оси
на комплексной плоскости, кроме отмеченных раньше символов, обозначаются
также и символами соответственно +Re и +Im или +1 и +j.
Исследование синусоидных функций sin(ω t + ψ) или соs(ω t + ψ) удобно
осуществлять с помощью комплексного вектора (6.9).
•
•
Теперь рассмотрим две функции A1 = A1msin(ω t + ψ1) и A2 = A2msin(ω t – ψ2),
которые имеют одинаковую угловую частоту ω и разные начальные фазы.
Сдвиг фаз ϕ определяется как разность начальных фаз, а именно: ϕ = ψ1 –
– (–ψ2) = ψ1 + ψ2. Непосредственное исследование взаимосвязи между этими
функциями удобно заменить на исследование взаимосвязи между соответствующими этим функциям комплексными векторами.
Примем во внимание, что когда функции имеют одинаковую угловую
частоту, то с изменением времени t углы между соответствующим этим функциям комплексными векторами остаются неизменными. Если сдвиг фаз ϕ= 0,
то соответствующие комплексные векторы являIm z
ются сонаправленными, а если ϕ=π, то комплексные векторы являются направленными противоположно.
Изображение совокупности векторов, соотА&1
Re z ветствующих синусоидным функциям, построенψ1
ным с учетом их взаимной ориентации относиО
ψ2
тельно фазы, называется векторной диаграммой
&А
(рис. 6.2).
2
Операции сложения и вычитания синусоРисунок 6.2
идных функций, частоты которых совпадают,
можно заменить на операции сложения и вычитания векторов векторной диаграммы, что значительно проще.
Метод анализа гармонических колебаний, в котором операции над функциями, описывающими эти колебания, заменяются на операции над их символическими изображениями, называется символическим методом анализа гармонических колебаний или методом комплексных амплитуд.
Метод комплексных амплитуд позволяет любую синусоидную функцию в
момент времени t однозначно изображать соответствующим комплексным вектором, которому в свою очередь соответствует комплексное число. Это комплексное число при каждом t также можно считать символом синусоидной
функции. При этом модуль комплексного числа равняется амплитуде синусоидной функции, а аргумент комплексного числа – начальной фазе синусоидной
функции.
Символический метод комплексных амплитуд характеризует амплитуду и
начальную фазу синусоидной функции. Символический метод не отображает
состояние третьего параметра ω из-за того, что в пределах одной электрической
55
цепи действуют источники одной угловой частоты. Угловая частота ω считается за заданную величину.
Пример 6.1 Найти комплексное изображение напряжения
π

u =5sin  ω t +  (В).
3

Решение
π
π
π


Согласно условию,5sin  ω t +  =Im u& m e jωt . Тогда u& m = 5 cos + j sin  .
3
3
3


1
3 
Отсюда u& m = 5  +
j  В или u& m ≈ (2,5 + 4,33j) В.
2 2 
Комплексная амплитуда колебаний напряжения найдена.
О т в е т: u& m ≈ (2,5 + 4,33j) (В).
56
КОНТРОЛЬНЫЕ ВОПРОСЫ
1. Что называется мнимой единицей?
2. Что называется мнимым числом?
3. Какие мнимые числа называются равными между собой?
4. Какие мнимые числа называются противоположными?
5. Как определяется сумма мнимых чисел?
6. Как определяется разность мнимых чисел?
7. Каким способом определяется произведение мнимых чисел?
8. Каким числам может равняться натуральная степень мнимой единицы?
9. Что называется комплексным числом?
10. Что называется аффиксом комплексного числа?
11. Какая плоскость называется числовой комплексной плоскостью?
12. Что называется комплексной координатой точки на комплексной плоскости?
13. Какие комплексные числа называются равными между собой?
14. Существуют ли сравнимые комплексные числа?
15. Какие комплексные числа называются противоположными?
16. Какие комплексные числа называются комплексно-сопряжёнными?
17. Какая связь существует между комплексными и действительными числами?
18. Какая связь существует между комплексными и мнимыми числами?
19. Что называется действительной осью?
20. Что называется мнимой осью?
21. Как изображаются действительные числа на комплексной плоскости?
22. Как изображаются мнимые числа на комплексной плоскости?
23. Как изображаются комплексные числа на комплексной плоскости?
24. Что называется радиус-вектором комплексного числа?
25. Какой вектор называется комплексным?
26. Что называется модулем комплексного числа?
27. Какую геометрическую информацию содержит модуль комплексного
числа?
28. По какой формуле определяется модуль комплексного числа?
29. Что называется аргументом комплексного числа?
30. Что называется главным значением аргумента комплексного числа?
31. Можно ли утверждать, что два комплексных числа являются равными
между собой, если у них модули одинаковы?
32. По какой формуле определяется arg z, если z принадлежит первой четверти комплексной плоскости?
33. По какой формуле определяется arg z, если z принадлежит второй четверти комплексной плоскости?
34. По какой формуле определяется arg z, если z принадлежит третьей четверти комплексной плоскости?
57
35. По какой формуле определяется arg z, если z принадлежит четвертой
четверти комплексной плоскости?
36. Можно ли считать равными между собой комплексные числа, если главные значения аргумента у них одинаковы?
37. Можно ли считать равными между собой комплексные числа, если модули и главные значения аргумента у них одинаковы?
38. Можно ли считать равными между собой комплексные числа, если у них
модули одинаковы, а их аргументы отличаются на число 2nπ, где n ∈ Z?
39. Что называется алгебраической формой комплексного числа?
40. Как выполняется сложение комплексных чисел, заданных в алгебраической форме?
41. Всегда ли при сложении комплексных чисел выходит комплексное число?
42. Как выполняется вычитание комплексных чисел, заданных в алгебраической форме?
43. Всегда ли разность комплексных чисел является комплексным числом?
44. Как выполняется умножение комплексных чисел, заданных в алгебраической форме?
45. Всегда ли произведение комплексных чисел является комплексным числом?
46. Как выполняется деление комплексных чисел, заданных в алгебраической форме?
47. Каким законам подчиняются действия над комплексными числами, заданными в алгебраической форме?
48. Что называется тригонометрической формой комплексного числа?
49. В каком случае два комплексных числа, заданных в тригонометрической
форме, считаются равными между собой?
50. Как выполняется умножение комплексных чисел, заданных в тригонометрической форме?
51. Как выполняется возведение в натуральную степень комплексных чисел,
заданных в тригонометрической форме?
52. Как выполняется деление комплексных чисел, заданных в тригонометрической форме?
53. Как извлекается корень степени n, где n ∈ N, из комплексного числа, заданного в тригонометрической форме?
54. Сколько значений имеет корень степени n, где n ∈ N, из комплексного
числа, заданного в тригонометрической форме?
55. Что называется показательной формой комплексного числа?
56. В каком случае два комплексных числа, заданных в показательной форме, считаются равными между собой?
57. Как выполняется умножение комплексных чисел, заданных в показательной форме?
58. Как выполняется возведение в натуральную степень комплексного числа,
заданного в показательной форме?
58
59. Как выполняется деление комплексных чисел, заданных в показательной
форме?
60. Как извлекается корень степени n, где n ∈ N из комплексного числа, заданного в показательной форме?
61. Как выполняется сложение комплексных чисел, заданных в векторной
форме?
62. Как выполняется вычитание комплексных чисел, заданных в векторной
форме?
63. Как выполняется умножение комплексных чисел, заданных в векторной
форме?
64. Как выполняется деление комплексных чисел, заданных в векторной
форме?
65. В чем заключается свойство модуля суммы комплексных чисел?
66. В чем заключается свойство модуля разности комплексных чисел?
67. Какое комплексное число можно считать оператором поворота на угол
π
?
2
68. Какое комплексное число можно считать оператором, который переводит
комплексный вектор в противоположный ему вектор?
69. Запишите формулы Эйлера.
70. Используются ли комплексные числа в теории электрических цепей? В
чем заключается метод комплексных амплитуд?
59
ТРЕНИРОВОЧНЫЕ УПРАЖНЕНИЯ
№1. Заданы комплексные числа z1 = x1 + i y1 и z2 = x2 + i y2 , а также действительные числа a, b, c . Найти число z . Действия выполнять в алгебраической
форме. Получившиеся результаты, при необходимости, записать с точностью
до двух знаков после запятой.
№ x
y1 x2 y2
z
a
c
b
1
п/п
1.01
–5
3
4
2
(
i 64
log 2 3 − tg 45o
o
tg17 ⋅ ctg17
o
1
 
3
)
−2
1.02
1
2
3
–4
1.03
2
3
–4
1
i12
1.04
3
–4
1
2
4
lg3
1.05
–4
3
2
1
2
3
1.06
5
1
4
3
5
3 64
1.07
1
5
3
–4
sin 750o
4
7
1.08
–1
5
–3
4
–1
cos 2 135o
2
1.09
4
–1
2
5
–2
3
tg −2 30o
1.10
–1
4
5
2
tg 2 60o
4
i8
1.11
–1
4
–2
5
–3
1.12
1
3
3
2
–2
1.13
1
4
4
5
1 − cos
1 + cos
1
9
–1
2
π
1 
1+  
2 
−3
 2tg15o 
 1 + tg 215o 


(
−2
3
2
1.15
1.16
1.17
1.18
1.19
6
1
–2
1
tg 405o
7
5
3
5
–1
2
1
log 2 + log 2 48
3
1
5
–1
3
4
1
–1
1.20
1.21
1.22
1.23
2
3
5
–1
1
 
4
1
−
2
–1
−16
4
5
3
–1
(i )
–1
–5
3
2
1
7
3
–5
2
tg 135o
5
15
+ log 2 48
7
−c
sin 7o cos 23o + sin 23o cos7o
3
2
( i )−24
–1
2
3
sin 2 765o
2
3
–5
1
lo g 3  
3
1.24
1
1
4
–4
6
cos2 135o
5
1.25
3
4
–4
1
5
4
(sin123o cos3o − sin 3o cos123o )
)
)
(
)
(
)
a ( z1 + z 2 ) − b
)
4
2
z1 + z 2
+ c z 23
z2
z1
+ c z 23
z1 + z 2
z1
a z1 + z 2 − b
+ c z13
z1 − z 2
z2
a z1 + z2 − b
+ c z13
z1 − z 2
z1
a ( z1 − z 2 ) − b
+ c z1 z 22
z1 + z 2
z + z2
a z1 − z 2 − b 1
+ c z12 z 2
z1 − z 2
z + z2
a z 2 − z1 − b 1
+ c z1 z 22
z1 − z 2
z
a z1 − z 2 − b 1 + c z12 z 2
z1z2
z
a z1 − z 2 − b 2 + c z1 z 22
z1 z 2
3
z2
a ( z 1 z 22 ) − b
+ c z2
z1 + z 2
z
2
a ( z12 z 2 ) − b 1 + c ( z1 − z 2 )
z2
2
z
a z1 z 2 − b 2 + c ( z 1 − z 2 )
z1
2
(
)
(
)
(
)
(
)
(
)
(
)
(
)
a z1 + z2
log 2 64
2
z1
z2
2
z 2 − z1
+ c z2
z1 + z 2
a z1 z 2 + b
4
1
2
z1
z2
z1
+ c z1 − z 2
z2
z
a z13 + b 1 + c z1 − z2
z2
z + z2
a z1 z 2 + b 1
+ c z 23
z2
z + z2
a z1 z 2 + b 1
+ c z13
z1
2
z + z2
2
+ c z1
a ( z1 z 2 ) − b 1
z1 − z 2
13π
3
1.14
5
(
2
log3
2
z1
+ c z1 z 2
z2
z 
3
a z 2 + b  1  + c z1 + z2
 z2 
a z13 + b
eln 3
–2
3
b
( z1 − z 2 )
(
32
cos
−c
2
2
4
6
–1
b
( z1 − z 2 )
z 
a  1  − b ( z12 − z2 ) + c z1 z2
 z2 
a z12 + b
6
3
z1 z 22
(
1
3
)
a z1 − z 2 +
–1
1
2
)
( z1 − z 2 )
z1
z
+ b 1 + c z13 z 23
z2
z2
a
cos9o cos51o − sin 9o sin 51o
7π
3
1
π
(
a z1 − z 2 +
4cos2 6o + 4sin 2 6o
lg 2 16
cos
a z12 − b z 2 + c
sin 210o
(
)
−b
60
№2. Задано комплексное число z в алгебраической, тригонометрической
или показательной форме. Записать это же число в других формах. Получившиеся результаты при необходимости записать с точностью до двух знаков после запятой.
№ Алгебраическая Тригонометрическая Показательная
п/п
форма z
форма z
форма z
3 + 4i
2.01
−4 + 3 i
2.02
−5 − 6 i
2.03
4 − 5i
2.04
2.05
2.06
2.07
2.08
2.09
2.10
2.11
2.12
2.13
2.14
2.15
2.16
2.17
2.18
2.19
2.20
(
)
3 ( cos 7o + i sin 7o )
1
(2 cos89o + i sin 89o )
3 ( cos16o + i sin16o )
1
(8 cos179o + i sin179o )
5 cos 4o + i sin 4o
π
π

4  cos + i sin 
5
5

π
π

5  cos + i sin 
4
4

3π
3π 

9  cos + i sin 
5
5 

5π
5π 

7  cos + i sin 
8
8 

19π
19π 

1 cos
+ i sin

20
20 

11π
11π 

8  cos
+ i sin

15
15 

3π
3π 

4  cos + i sin 
8
8 

9e
8e
7e
6e
4π
i
5
3π
i
5
4π
i
7
5π
i
8
61
oi
2.21
5e−6
2.22
6e −12
2.23
9e−170
2.24
7e13
oi
oi
oi
2.25
9e
π
i
8
№ 3. Выполнить действия над заданными комплексными числа-
ми. Ответ представить в тригонометрической форме. Получившиеся
результаты в случае необходимости записать с точностью до двух
знаков после запятой.
5
3.02
 
π
π 
 3  cos 4 + i sin 4  

 
.
π
π 
3π
3π 

2  cos + i sin  ⋅ 5  cos + i sin 
10
10  
10
10 

3.04
5π    6π
6π  
 5π
2  cos + i sin  ⋅  3 cos + i sin  
11   
7
7 
 11
.
π
 π
5  cos + i sin 
4
 4
3.06
 
3π
3π  
 5  cos 8 + i sin 8  

 
.
5π
5π  
5π
5π 

2  cos + i sin  ⋅ 4  cos + i sin 
9
9  
7
7 

3.08
1
9π
9π  
9π
9π 
 cos + i sin  ⋅ 4  cos + i sin 
2
10
10  
11
11 
.
5π
5π  
5π
5π 

3 cos + i sin  ⋅ 4  cos + i sin 
6
6  
7
7 

3.01
π
π 
π
π

4  cos + i sin  ⋅ 5  cos + i sin 
7
7 
6
6

.
4
 
π
π 
 2  cos 5 + i sin 5  

 
3.03
 
π
π  
7π
7π 
 4  cos 7 + i sin 7   ⋅ 2  cos 12 + i sin 12 
 

 
.
8π
8π 

9  cos + i sin 
9
9

3.05
π
π 
π
π

7  cos + i sin  ⋅ 8  cos + i sin 
7
7 
8
8

.
π
π 
π
π

9  cos + i sin  ⋅10  cos + i sin 
9
9 
10
10 

3.07
 
6π
6π  
 4  cos 7 + i sin 7  

 
.
7π
7π  
8π
8π 

9  cos + i sin  ⋅ 7  cos + i sin 
8
8  
9
9 

3.09
  2π 
π 
 2π    π
4 cos −  + i sin  −   7 cos + i sin  
8 
 5    8
  5
.
π
π

5 cos + i sin 
12 
 12
2
3
4
7
2
3.11
3.10
3π
3π 
5π
5π 


2  cos + i sin 
8  cos + i sin 
7
7 
8
8 


⋅
.
4π
4π 


π
π 


3  cos
+ i sin
 7  cos  −  + i sin  −  
9
9 

 9
 9 

8π 
 8π
7  cos + i sin 
6π
6π 
9
9


⋅ 5 cos + i sin .
2
7
7 
 
7π
7π   
5
cos
+
i
sin

 
11
11  
 
4
3.12
 
 π
 π 
 3  cos  −  + i sin  −   
 8
 8 
 
.
3
 
π
π 
 5  cos 9 + i sin 9  

 
62
2
3.13
 
π
π 
2  cos + i sin  

5
5 
8π
8π 


⋅
4  cos + i sin  ⋅ 
5π
5π 
9
9  

3 cos + i sin 
6
6 

3.15
6π 
 6π
9 cos + i sin 
12
π
12
π
7
7



6 cos
+ i sin
.
⋅
4
13
13

   2π
2π  
 5 cos 3 + i sin 3  

 
3.14
5π
5π 

5  cos + i sin 
7π
7π 
6
6 

8  cos + i sin  ⋅ 
.
2
9
9   

3π
3π  
 4  cos 10 + i sin 10  

 
3.16
(
) (
2 cos27o + i sin27o ⋅ 4 cos16o + i sin16o
) ( ( )
(
)
( ))
3 cos28o + i sin 28o ⋅ 5 cos −3o + i sin −3o
.
) : 5( cos74 + i sin 74 ) . 3.18 5 ( cos81 + i sin 81 ) : cos31 + i sin 31 .
3.17 (
(
)
3( cos94 + i sin 94 ) 6 ( cos 47 + i sin 47 )
7 ( cos 72 + i sin 72 )
7 cos7 + i sin7 )
( cos3 + i sin3 ) .
. 3.20 2( cos ( −32 ) + i sin ( −32 ) ) :
3.19 ( 4( cos( −9 ) + i sin ( −9 )) ) : (
4( cos7 + i sin7 )
8( cos8 + i sin8 )
9( cos9 + i sin9 )
7 cos81 + i sin81 )
.
. 3.22 ( 4 ( cos12 + i sin12 ) ) :
3.21 ( 5( cos14 + i sin14 )) : (
2( cos21 + i sin21 )
(3(cos16 + i sin16 ))
4
o
o
+
2
cos92
i
sin
92
3
cos17
+
i
sin17
(
)
)) . 3.24 (
):
((
1
1
3.23
:
.
3
o
o
o
o
5
cos27
+
i
sin27
8
cos
2
+
i
sin
2
(
)
(
)
( 2(cos23 + isin23 ))
(5(cos 63 + i sin 63 ))
6
7 ( cos ( −84o ) + i sin ( −84o ) ) )
(
1
:
.
3.25
4
o
o
o
o
3
cos
5
+
i
sin
5
(
)
( 2 ( cos15 + i sin15 ) )
2 cos 49o + i sin 49o
o
o
o
o
o
o
o
o
o
o
o
8
o
o
o
o
o
3
5
o
2
3
 45π i   35π i 
4.01. e ⋅  5e  :  6e  .

 

2
3
3π
−
i
 − π5 i   78π i 
4.03.  7 e  :  8e  : 6e 10 .

 

π
i
2
4.05. 7 e
4.07.
−
π
i
8
4
 − π9 i 
1
⋅  9e  :
.
5
6π
i




7
 5e 


o
o
o
5
o
o
3
o
o
 3e−8o i 




2
5
:
4e20
5e
21o i
.
4
 − π3 i  89π i  811π i 
4.02.  5e  ⋅ e :  2e  .




4
π
−7 π
−7 π
i 
i
i
5
10
11
4.04. 2e ⋅  5e
:
9e
.



5
 3e18o i 



 : 1 .
4.06.
2
o
 4e −5o i  5e16 i




oi
1
o
10
o
№ 4. Выполнить указанные действия.
−
5
o
o
o
o
o
o
o
o
o
o
o
o
o
4.08.
1
oi
3e−19
 1
:
 4e8o i

5

 : 1 .
 16e7o i

o
o
o
4
63
 2e −13o i 

4.09. 
 5e4o i 


 πi 
9e
4.11.  π 

i 
2 
8e


2
3
10
 8e7o i 
 .
:
 5e −8o i 


2
4
7π
i 

2e 8 

:
.
 − 6π i 
 5e 7 


 −14π i 
2e

4.12. 
π

i 

9
3e


2
1
4.13.
2e
4.15.
3π
−
i
4
3e
9e
3π
i
7
5π
i
6
⋅ 5e
⋅e
4π
i
5
5π
i
6
⋅ 8e
7π
i
8


3


⋅
.
8π 
 − 7 π i
i
10
⋅ 3e 11 
 2e
4.16.
3
 3e10o i   5e11o i   6e91o i 

 
 


 
 

3
 3e84o i 


21o i
18o i 
 .
4.19. 5e ⋅ 6e ⋅
o
5e17 i
5
4.20.
4
o
:  5e12,5 i 


2
 5e


7o i 
2

  6e
 
6e
33o i
oi
5e28
 8 
 1 
17o i
 ⋅e :
 .
4.23. 
 7 e32o i 
 2e 25o i 




6
2


4


:
.
3π 
 π i
i
5
4 
 3e ⋅ 6e 
9e −10
o
2
o
4.25.  2e−0,5 i 


⋅ 8e
5
π
i
7
8o i 
3

  2e
 
 1
⋅
 3e95o i

4
.
5
.


 5e−16o i 




4

 .


 1 

4.22. 
o
18
i
 3e 


4
 8 

 .
:
o
21
i
 7e 


 e5o i
4.24. 
 3e25o i

4
 8e63o i
:
 −6o i
 7e




3o i 
1
o
4.18. 8e135 i ⋅ 7 e−127 i ⋅
.
2
 1 
5e
 .
4.21.
:
4
o
80
i
 6e0,5o i   8e 




32o i
2e
π
i
8
4
 79π i 
5e
:  7π  .


 6e 10 i 


oi
oi
4
4.14.
−
3
2
 π9 i 
:  8e  .


7e19
4.17.
5
 8e91o i   5e17o i 
 :
 .
4.10. 
 7 e92o i   3e−9o i 

 

2
3

 .


4
o
:  3e11 i  .


№5. Решить заданное уравнение на множестве комплексных чисел. Найти
величину А, равную сумме квадратов всех корней уравнения. Получившиеся
результаты в случае необходимости записать с точностью до двух знаков после
запятой.
64
№
п/п
Уравнение
№
п/п
Уравнение
№
п/п
1
= 0.
3 −i
5.02
z 2 + 6 z + 58 = 0.
5.03
z3 +
5.05
z3 +
4
= 0.
− 3 +i
5.06
z 2 + 2 z + 17 = 0.
3
5.01
z +
5.04
z 2 − 14 z + 58 = 0.
5.07
4
= 0.
z −
−1 + i 3
5.08
z − 8 z + 17 = 0.
5.10
z − 10 z + 29 = 0.
5.11
z
5.13
z3 −
5.14
5.16
z2 + 36z + 52 = 0.
5.19
3
2
Уравнение
1
= 0.
3+i
( 2)
−
3
5.09
z3
5.12
z 2 − 4 z + 29 = 0.
z 2 − 8 z + 52 = 0.
5.15
z3 +
5.17
z6 + 64 = 0.
5.18
z 2 − 10z + 61 = 0.
z 4 + 16 = 0.
5.20
z2 +12z + 61 = 0.
5.21
z5 − 32 = 0.
5.22
z 2 − 9 z + 45 = 0.
5.23
z 4 + 81 = 0.
5.24
z2 +12z + 45 = 0.
5.25
z 4 + 625 = 0.
2
4
= 0.
2 − 2i
3
( 2)
−
1+ i
−1 + i
= 0.
3
= 0.
4
= 0.
2 + 2i
65
ОТВЕТЫ К ТРЕНИРОВОЧНЫМ УПРАЖНЕНИЯМ
1.01 z = 11.959 − 28.349 i .
1.04 z = −11.76 + 11.57 i .
1.07 z = −26.82 + 13.24 i .
z = −96.655 − 104.138 i .
1.10 z = 30.414 − 84.966 i .
1.13. z = −43.906 − 99.17 i .
1.16 z = −31.5 + 798.5 i .
1.19 z = −1.94 + 34.58 i .
1.22 z = 31.254 − 100.408 i .
1.25 z = 267.81 + 440.67 i .
1.02 z = −3.94 + 2.455 i .
1.05 z = −65 + 59 i .
1.08 z = 27.66 + 8.38 i .
1.11
1.14
1.17
1.20
1.23
z = 122.206 − 198.876 i .
z = 90.5 − 116.5 i .
z = −299.139 − 444.279i .
z = 125.351 + 31.892 i .
z = −934.647 + 501.412i .
1.03 z = 1.404 ⋅103 + 849.941i .
1.06 z = 534.84 + 388.12 i .
1.09
1.12
1.15
1.18
1.21
1.24
z = 193.04 + 109.2 i .
z = 32.7 + 13.4 i .
z = −44.36 + 247.992 i .
z = 88.154 + 24.769 i .
z = −32.125 − 102 i .
2.01 5(cos53.13° + isin53.13°); 5e53.13°i. 2.02 5(cos143.13° + isin143.13°); 5e143.13°i.
2.03 7.81(cos129.8°−isin129.8°); 7.81e−129.8°i. 2.04 6.4(cos51.34° − isin51.34°); 6.4e−51.34°i.
2.05 4.988 + 0.349i; 5e 4°i.
2.06 2.978+0.366i; 3e7°i.
2.07 8,726 ⋅ 10−3 + 0,5i; 1/2e89°i.
2.08 0.827 + 0.827i; 3e16°i.
2.09 0.12 + 0.034i; 1/8e179°i.
2.10 3.236 + 2.351i; 4e (π/5)i.
2.11 3.536 + 3.536i; 5e (π/4)i.
2.12 −2,781 + 8,56i; 9e (3π/5)i.
2.13 −2.679 + 6.467i; 7e (5π/8)i.
2.14 −0.988 + 0.156i; e (19π/20)i.
2.15 −5.353 + 5.945i; 8e (11π/15)i.
2.16 1.531 + 3.696i; 4e (3π/8)i.
2.17 −7.281 + 5.29i; 9(cos 4π/5 + isin 4π/5).2.18 −2.472 + 7.608i; 8(cos 3π/5 + isin 3π/5).
2.19 −1.558 + 6.824i; 7(cos 4π/7 + isin 4π/7). 2.20 −2.296 + 5.543i; 6(cos 5π/8 + isin 5π/8).
2.21. 4.973 – 0.523i; 5(cos6° − isin6°).
2.22. 5.869 – 1.247i; 6(cos12° − isin12°).
2.23. −8,863−1,563i, 9(cos170° − isin170°). 2.24 6.821 + 1.575i; 7(cos13° + isin13°).
2.25. 8.315 + 3.444i; 9(cos π/8 + isin π/8).
3.01. z = 2.277(cos(−24.144°) + i sin(−24.144°)). 3.02. z = 61.896(cos163.98°+ i sin163.98).
3.03. z = 3.556(cos(−3.571°) − i sin(−3.571)). 3.04. z = 10.8(cos139.675° + i sin139.675°).
3.05. z = 1.912(cos(−43.849°) + i sin(−43.849°)). 3.06. z = 78.125(cos(41.429°) + i
sin(41.429°)).
3.07. z = 260.063(cos42.5° + i sin42.5°). 3.08. z = 0.167(cos30.701° + i
sin30.701°).
3.09. z = 39.2(cos(−42°) + i sin(−42°)). 3.10. z = 7(cos85.195° + isin85.195°).
3.11. z = 0.19(cos129.643° + i sin129.643°). 3.12. z =16.2(cos(−110°) + i sin(−110°)).
3.13. z = 5.333(cos82° + i sin82°).
3.14. z = 2.5(cos(−178°) + i sin(−178°)).
3.15. z = 0.086(cos(−159.56° + i sin(−159.56°)). 3.16. z = 0.533(cos18° + i sin18°).
3.17. z = 0.8(cos(−72°) + i sin72°).
3.18. z = 0.714(cos(−146°) + i sin(−146)).
3.19. z = 74900(cos(71°) + i sin(−71°)). 3.20. z = 8(cos(−55°) + i sin(−55°)).
3.21. z = 35.174(cos(−18°) + i sin(−18°)). 3.22. z = 9216(cos115° + i sin115°).
3.23. z = 0.005787(cos139° − isin139°). 3.24. z = 1.002(cos(−176.646°)+i sin(−176.646°)).
3.25. z = 22060 (cos161° + isin161°).
66
4.01. −0.068 − 0.094i.
4.04. 137.984 + 15.832 i.
4.07. 3.88 ⋅10−3 +3.37 ⋅10−3i .
4.10. −0.226 − 0.19i.
4.13. 0.024 − 8.064 ⋅10−3i .
4.16. 5.20⋅10−5 −9.02⋅10−5i .
4.19. 11.301 − 61.605i.
4.22. 8.18⋅10-3 − 4.72⋅10−3i .
4.25. 9.76 ⋅ 10−3 − 0.03i.
4.02. −0.866 − 1.301i.
4.05. 9.018⋅107 −1.116⋅108i .
4.08. 2.22 ⋅103 +4.98⋅103i .
4.11. −1.121 + 9.948i.
4.14. 62.654 − 16.537i.
4.17. −1.07 ⋅10−5 +1.59 ⋅10−5i .
4.20. 0.014 − 3.83 ⋅ 10−3i .
4.23. 22.814 − 21.275i.
4.03. −0.01 − 0.012 i.
4.06. −33.289 + 68.252 i.
4.09. 7.453 ⋅10−3 − 0.038i .
4.12. −0.555 − 0.264i.
4.15. −3.29⋅10−4 −5.61⋅10−4i .
4.18. 6.254 ⋅ 10−4 +0.018i .
4.21. −0.243 − 0.043i.
4.24. 2.41⋅10−3 +7.90 ⋅10−3i .
5.01. 0.5102 − 0.608i; −0.7816 − 0.1378i; 0.2715 + 0.7458i. 5.02. −3−7i; −3 + 7i.
5.03. 0.512 + 0.608i; −0.782 − 0.138i; 0.271 − 0.746i.
5.04. 7−3i; 7 + 3i.
5.05. 1.241 + 0.219i; −0.431 − 1.184i; −0.81 + 0.965i.
5.06. −1 − 4i; −1 + 4i.
5.07. 1.184 − 0.431i; −0.219 − 1.241i; −0.965 − 0.81i.
5.08. 4 − і; 4 + i.
5.09. −1.217 − 0.326i; 0.326 + 1.217i; 0.891 − 0.891i.
5.10. 5 − 2i; 5 + 2i.
5.11. 1.217 − 0.326i; −0.326 + 1.217i;−0.891 − 0.891i.
5.12. 2 − 5i; 2 + 5i.
5.13. 1.428 + 0.592i; −0.202 − 1.533i;−1.227 + 0.941i.
5.14. 4 − 6i; 4 + 6i.
5.15. 0.702 + 0.914i; 0.441 − 1.065i; −1.143 + 0.15i.
5.16. −34.492; −1.508.
5.17. −2i; 2i; 1.732 −i; 1.732 +i; −1.732 − i; −1.732 + i.
5.18. 5 − 6i; 5 + 6i.
5.19. −1.414 − 1.414i; −1.414 + 1.414i; 1.414 − 1.414i; 1.414 + 1.414i. 5.20. −6 − 5i; −6 + 5i.
5.21. 2; 0.618 − 1.902i; 0.618 + 1.902i; −1.618 − 1.9176; −1.618 + 1.176i. 5.22. 4.5 − 4.97i; 4.5 + 4.97i.
5.23. −2.121 − 2.121i; −2.121 + 2.121i; 2.121 − 2.121i; 2.121 + 2.121i. 5.24. −6 − 3i; −6 + 3i.
5.25. −5; 5; −5i; 5i.
67
ЛАБОРАТОРНАЯ РАБОТА
ПО ТЕМЕ «КОМПЛЕКСНЫЕ ЧИСЛА»
1. Цель работы
Целью работы является приобретение навыков работы с комплексными
числами в алгебраической, тригонометрической и показательной формах.
2. Ключевые положения
Одно и то же комплексное число можно записать в различных формах:
а) алгебраической
z = a + bi ,
где a и b − действительные числа, а i = −1 − мнимая единица;
б) тригонометрической
z = r ( cos ϕ + i sin ϕ ) ,
где r = z − модуль комплексного числа, ϕ = Arg z . z ≥ 0 , причем z = 0
только при z = 0 . При z ≠ 0 Arg z имеет бесчисленное множество значений
Arg z = arg z + 2nπ ,
где n = ±1, ± 2, ± 3, K , arg z − главное значение аргумента. Обычно главное значение аргумента выбирают в промежутке −π < arg z ≤ π ;
с) показательной
z = reiϕ , где eiϕ = cos ϕ + i sin ϕ ,
а также задать геометрически с помощтью аффикса или комплексного вектора
или в виде r∠ϕ.
Сложение и вычитание комплексных чисел z1 = a1 + b1 i и z2 = a2 + b2 i
удобнее производить в алгебраической форме по формулам (2.1) и (2.5)
z1 + z2 = ( a1 + b1i ) + ( a2 + b2i ) = ( a1 + a2 ) + ( b1 + b2 ) i ,
z1 − z2 = ( a1 + b1i ) − ( a2 + b2i ) = ( a1 − a2 ) + ( b1 − b2 ) i ,
а умножение, деление и возведение в степень − в тригонометрической и показательной формах.
Если z1 = r1 ( cos ϕ1 + i sin ϕ1 ) , z2 = r2 ( cos ϕ2 + i sin ϕ2 ) ,
то в соответствии с формулами (3.3), (3.4), (3.5), (4.3), 4.4), (4.5), имеем
ϕ +ϕ i
z1 z2 = r1r2 e( 1 2 ) ,
z1 z2 = r1r2 ( cos ( ϕ1 + ϕ2 ) + i sin ( ϕ1 + ϕ2 ) ) ,
z1 r1
= ( cos ( ϕ1 − ϕ2 ) + i sin ( ϕ1 − ϕ2 ) ) ,
z2 r2
z1 r1 ( ϕ1 −ϕ2 )i
,
= e
z2 r2
z n = r n ( cos nϕ + i sin nϕ ) ,
z n = r n e nϕi .
68
Перевод комплексных чисел из алгебраической формы в показательную и
наоборот можно производить с помощью микрокалькулятора и в вычислительной среде MathCad.
С помощью микрокалькулятора модуль находится по формуле (1.3)
r = z = a2 + b2 ,
а аргумент комплексного числа z = a + bi находятся по формулам (1.10), если
arg z ∈ ( −π; π] ; (1.11), если arg z ∈ [ 0; 2π ) . В некоторых электротехнических кур π 3π 
сах предполагается, что arg z ∈  − ;
 . В этом случае arg z находится по
 2 2 
приведенной ниже формуле (*)
b

arctg
,
a > 0,

a
arg z = 
−90 ≤ arg z < 270o .
(* )
b
o
arctg + 180 , a < 0 ,

a
Пример 1 Записать в тригонометрической и показательной форме комплексное число z = 2 + 3 i .
Решение
b
3
r = z = 22 + 32 ≈ 3.606 , arg z = arctg = arctg ≈ 56.31° .
a
2
Тригонометрическая и показательная формы комплексного числа
z = 2 + 3 i соответственно имеют вид:
z ≈ 3.606 ( cos56.31° + i sin 56.31° ) ,
О т в е т: z ≈ 3.606 ( cos56.31° + i sin 56.31° ) ,
o
z ≈ 3.606e56.31 i .
o
z ≈ 3.606e56.31 i .
В MathCad модуль и аргумент комплексного числа z := a + bi находятся
по формулам
z = , arg ( z ) = .
ЗАМЕЧАНИЕ. Мнимая единица набирается на клавиатуре как «1i» (или
«1j») или кнопкой «і» на панели «Калькулятор» (см. рисунок ниже).
Модуль набирается кнопкой « x » на панели «Калькулятор».
Функция arg набирается на клавиатуре или выбирается из имеющегося в
MathCad списка встроенных функций на панели «Вставить функцию»
69
ЗАМЕЧАНИЕ 1. Единица измерения результата нахождения аргумента
(градусы или радианы) устанавливается на панели «Вставка единиц измерения», которая открывается после двойного щелчка по результату вычисления.
Зная модуль и аргумент комплексного числа можно восстановить первоначальную алгебраическую форму комплексного числа по формуле
z = z ⋅ cos(arg(z ))+ z ⋅ sin(arg(z )) ⋅ i .
ЗАМЕЧАНИЕ 2. В MathCad существуют еще две встроенные функции:
angle и atan2, позволяющие находить аргумент комплексного числа z := a + bi .
Команда angle(а, b) возвращает угол в радианах в промежутке [ 0, 2π ) или
)
в градусах в промежутке 0o , 360o . Команда atan2 (а, b) возвращает угол в радианах в промежутке [ −π, π ) или в градусах в промежутке  −180o , 180o .
Функция atan2 связана с функцией atan (арктангенс) и её аргументом
следующими соотношениями: atan2 (а, b) = atan (b/а) при a > 0 (хотя функция
atan возвращает значение между −π/2 и π/2) и atan2 (а, b) = arg ( a + b i ) .
Если результат применения этих функций положительный, то угол измерен против часовой стрелки от оси X. Если результат отрицательный, то угол
измерен по часовой стрелке от оси X.
)
Пример 2 Модуль и аргумент комплексного числа z := −2 − 3 i , найденные в MathCad, соответственно равны:
70
z ≈ 3.606 , arg ( z ) ≈ −123.69deg или
angle ( −2 , − 3) = 236.31deg ,
atan2 ( −2 ,−3) = −123.69deg .
Заметим, что
 −3 
atan   = 56.31deg .
 −2 
Само число z восстанавливается по формуле
z = z ⋅ cos(arg(z ))+ z ⋅ sin(arg(z )) ⋅ i = 2+3 i .
Действия над комплексными числами в тригонометрической форме в
MathCad можно упростить, если создать функцию пользователя
Z ( mod, arg ): = mod ⋅ ( cos(arg ⋅ deg)+i ⋅ sin(arg ⋅ deg) ) .
ЗАМЕЧАНИЕ. Градусы включены в формулу по определению. Если вы захотите перейти к радианной мере угла, вам следует удалить множитель единицы градуса deg .
Чтобы использовать созданную функцию пользователя
как оператор, надо использовать кнопки на панели «Вычисления».
В рассматриваемом случае для созданной нами функции «Z» используем
кнопку панели, помеченную как xfy, которая указывает, что название функции
следует записать в средней позиции. Щелчок по этой кнопке приводит к появлению трех помеченных позиций
.
В первой позиции печатаем величину модуля комплексного числа
z = 2 + 3 i , в средней позиции − название функции (в рассматриваемом случае
Z ), в третьей − величину угла (аргумента) в градусах комплексного числа
z = 2 + 3 i . Имеем
z1: = 3.606 Z 56.31 ,
тогда
z1 = 2 + 3 i .
Вы можете создать также функцию пользователя IZ , инверсную к функции названной Z , которая возвратит модуль и аргумент комплексного числа
z := 2 + 3 i . Для этого определим инверсную функцию IZ как матрицу с одной
строкой и двумя столбцами (вектор-строку)
 arg(z) 
IZ ( z ): =  z
.
deg


В результате применения этой функции к числу z = 2 + 3 i получим
IZ (z ) = ( 3.606 56.31) .
Пользовательский формат оператора может быть объединен с любыми
другими требуемыми операторами, как это показано на следующих примерах.
Пример 1 Пусть дано комплексное число z1 = 2 + 3 i . Сопряженным к нему будет комплексное число z1c: = 3.606 Z 56.31 , которое после вычисления дает
z1c = 2 − 3 i .
71
ЗАМЕЧАНИЕ. Черта сопряжения набирается на клавиатуре клавишей «“»,
при этом синяя линия подчеркивания числа, для которого мы ищем сопряженное число, должна быть направлена прямым углом вправо.
Действительная и мнимая части комплексного числа 3.606∠56.31 могут
быть найдены соответственно по формулам:
Re ( 3.606∠56.31) = 2 ; Im ( 3.606∠56.31) = 3 .
Нахождение модуля и аргумента комплексного числа 3.606∠56.31 дают
результаты, которые следовало ожидать:
3.606 Z 56.31 = 3.606 , arg ( 3.606 Z 56.31) = 56.31o .
3. Лабораторное задание
Выполнить вычисления в соответствии с условием примера по своему варианту. Для этого:
1) перевести все комплексные числа, над которыми производятся действия умножения, деления и возведения в степень, в показательную форму;
2) по правилам произвести необходимые действия над комплексными
числами;
3) если полученные результаты надо складывать или вычитать, то перевести числа в алгебраическую форму и выполнить необходимые действия;
4) окончательный результат перевести в показательную форму;
5) оформить протокол работы в соответствии с разделом 3.
Пример 2 Вычислить
2
z2 + 2 z3 − 2 + 3i )
(
6 z13
3
z0 =
+ ( 2iz4 ) +
2
z 4 − 8 z5
( z5 + z3i )
при следующих данных: z1 = 4.1 − 3.1i , z2 = 2.12 + 2.465 i , z3 = −3.51 + 4.94 i ,
z4 = 4.27 − 1.765 i , z5 = −9.669 − 10.05 i .
Результат вычисления записать в показательной форме.
Решение
1. Составим таблицу перевода комплексных чисел из алгебраической
формы в показательную:
z
z2 + 2 z3 − 2 + 3i
z4 − 8 z5
2iz4
z5 + z3i
z1
z4
a
−6,9
81,622
3,53
−14,609
4,1
4,27
b
15,345
78,635
8,54
−13,561
−3,1
−1,765
r
16,85
113,338
9,241
19,932
5,14
4,62
ϕo
114,211
43,932
67,542
−137,131
−37,093
−22,458
72
2. Вычислим отдельно каждое выражение, входящее слагаемым в пример:
2
z2 + 2 z3 − 2 + 3i ) 16.8252 ei⋅114.211°⋅2
(
z′ =
≈
≈ 2.498 ⋅ e −i⋅175.509° ,
0
113.338ei⋅43.932°
z4 − 8 z5
2
z0′′ = ( 2iz4 ) ≈ 9.2412 ⋅ ei⋅67.542° ,
z0′′′ =
°⋅3
6 z13
( z5 + z3i )
2
≈
6 ⋅ 5.143e−i⋅37.093
°⋅2
19.9322 e −i⋅137.131
°
≈ 2.051 ⋅ ei⋅162.983 .
3. Полученные комплексные числа переведем в алгебраическую форму:
z0′ ≈ 2.498 ⋅ e−i⋅175.509° ≈ −2.49 − 0.196 ⋅ i ,
°
z0′′ ≈ 9.2412 ⋅ ei⋅67.542 ≈ −60.471 + 60.292 i ,
z0′′′ ≈ 2.051 ⋅ ei⋅162.987° ≈ −1.961 + 0.6 ⋅ i .
4. Сложим найденные слагаемые:
z0 = z0′ + z0′′ + z0′′′ ≈ ( −2.49 − 0.196 ⋅ i ) + ( −60.471 + 60.292i ) + ( −1.961 + 0.6 ⋅ i ) = −64.922 + 60.697 ⋅ i
5. Полученный результат запишем в показательной форме. Для этого находим модуль и аргумент полученного результата:
r ≈ 88.876 , ϕ ≈ 136.927° .
О т в е т. z0 ≈ 88.876 ⋅ ei⋅136.927° .
4. Варианты заданий
№
вар.
3
1
( 2 z1 + z2 ) ( z2 + z3 )
2
z
+ 3 z z + 2 z z + 12
z5
2
1 2
z 4 − z5
3
2
( 4 z1 − z2 ) ( z2 + z3 )
− 3 z12 z2 + z42 − 5
z5
6
3
( 6 z1 + z4 ) ( z2 − 7 z3 )
( z1 − z2 )
3
z1 z23
z 4 + z5
2
2 3
4 5
2
z1
5
№
вар.
z0
2
+ 2 z42 z5 − z53 +
z1
z52
3
3
4
z13 + 5i
z5
18
z ( 2 + i ) i z42
−z + 3 2
+
z5
z3
3
5
2
z1 z4 + ( 2i )
− 5z + 2 +
z2
z3 + z5
7
z1 ( z4 + z3 )
8
z1 ( 2 z1 − 3 z2 )
z4 − 4 z53
3
+
+
z
+
5
z52
z 4 + z5
z3
3
5
z0
19
3
( z1 + 5 z2 ) − 3z z 3 − z + 2 z1 + z42
1 2
5
2
z52 z32 + z5
( z3 + z 4 )
2
( z2 − 5 z3 ) + 2 z z 3 − z 2 + z1 + z42 + z5
2 4
5
3
z52 z3 + 4 z5
( z3 + z 4 )
3
( z2 + z3 ) + z 3 + 6 z1 − z12 z2 ( z42 + z5 )
5
2
2
3
( z 4 − 8 z5 )
( z5 i )
( z1 + z2i )
3 + z5
3
2
+ ( z3 + 2i ) z2 − 5 z42 +
z1
z52
20
z13 z42 + z53
z + 2 z2
+
− 1
+ z1 z23 z5
2
z5 z 3 − 7 z5 ( z 4 + z5 )
21
z13
z1 + z2
z42 + z5
2
+
z
+
z
−
+
4
5
4
z52
( z3 + z 4 ) z3 + z5
3
73
№
вар.
9
№
вар.
z0
3 z12
( z3 + z 4 )
3
z 4 + z5
− z1 z23 + z42 +
3
10
11
12
2 z42 + z5
2i + z5
( z1 + z2 )
4
−5 z4 + iz5
3
− 5 z1 z23 − ( 2iz5 ) +
( z4 + 2i + 3)
2
2
3
22
3
z13 2 z 4 + ( 3 z 2 ) ( z1 − z2 )
+
−
+ 4 z1 z23
z5
z3 + z5
3i + z5
(z + z )
− 1 2
z13 + 2i
z5
z13
+
z
z
−
z33 + 2 z5
z52 + i
( z1 + z2 )
23
24
3
4
1 2
z0
25
( z1 + z2 ) ( z2 + z3 ) − z 7 z
z2 + 5iz3
3
2 z13
+
+ 3 z1 z +
z3 + z5 + 2i + 3
z5
13
( iz4 )
14
2iz13 ( z1 + z2 + 2 )
z 2 + 6i
+
− 7 z1 z 23 + z3 + 4
z5
z 4 + z5
z3 + z5
15
z42 − 2 z53 ( z3 + z4 )
2
+
+ ( z1 + 2i )1 z23 + z5
z3
z1 + z4
16
z4 z2 z53 + 3 i
z1 + 3 z2
+
−
+ z1 z23
2
2
z1
z3 + 5 z 5 ( 2 i z 4 + z5 )
17
z4 + z5 + 2 z13
2i + 4 z2
− +
+ z1 z52
2
2
− z 3 + 3 z 5 z5 ( 2 z 4 + z 5 )
26
z13 z4 + z53
+
z5
2i
2
z 3 ( 2 z 4 ) + z5
+ z z − 4 z5 + 1 +
z5
z3 + z5
2
1 2
( z3 − z 4 )
3
( z1 − 3z5 + 2 ) −
z2 + z4
2
( z2 − i ) z23 + z4 +
z4 + z53
z3 + z52
z
z42 + z53 + 5
2i
−
+ 3 z42 −
3
z
z 3 + z5
( z3 + z 2 ) ( z 4 + z 5 )
3
1
2
5
3
3
2
1
z3 + z5
2 −
z1 + z2
z5 ( z4 + 2i )
3
2
+ 2i − z1 z +
3
2
( 2i − z1 + z3 )
2
( z 4 + z5 )
z13 + 5
( z3 + 1)
3
2
+ ( 2 z 4 ) + z5 +
3
z13
z5
27
z1 z22 + 3
28
−2 z1
z22 + z5
2
+
+ ( z1 + 2i )1 z2 + z42
3
2
z3 + z5 ( z1 + z4 )
29
z3 − 6 i z5 ( z 4 + z5 )
z12
3
z 2 z5 +
−
− +
z4 + z53
z1 + 3 z3
z5
30
−
2
2
z1 + 2 z2
( z 4 + 2 i z5 )
2
+
z53 z42 + z5
+
− z1
z1 z3 − 3 z5
Числа z1 , z2 , z3 , z4 , z5 вычислить по следующим формулам:
z1 = 0.1( N + 10 ) − i ⋅ 0.1N ,
z2 = 0.1( N − 9.8 ) − i ⋅ 0.1(6.35 − N ) ,
z3 = −0.1( N + 4.1) + i ⋅ 0.2 ( N − 6.3) ,
z4 = 0.1( N + 11.7 ) − i ⋅ 0.25 ( 0.56 N − 10.3 ) ,
z5 = −0.3 ( N + 1.23) − i ⋅ 0.3 ( N + 2.5 ) ,
где N − номер студента, под которым студент записан в журнале группы.
ЗАМЕЧАНИЕ. В разобранном примере в лабораторной работе число N
было взято равным 31.
1.
2.
3.
4.
5.
6.
7.
5. Вопросы для самопроверки
Определение комплексного числа.
Различные формы комплексного числа.
Геометрическая интерпретация комплексного числа.
Модуль и аргумент комплексного числа.
Формулы перевода комплексного числа из одной формы в другую.
Правила действий над комплексными числами, записанными в разных
формах.
Тригонометрическая и показательная форма комплексных чисел в случае a = 0 и b = 0 .
74
ПРОВЕРОЧНЫЕ ТЕСТЫ
25
1. А = i10 . Какое из утверждений является справедливым:
1) А = і; 2) А = –і; 3) А = 1; 4) другой ответ?
2. А = i 20 ; В = i 21 . Какое из утверждений является справедливым:
1) А < В; 2) А > В; 3) А и В несравнимы; 4) другой ответ?
3. z = 3 + 4i. Какое из утверждений является справедливым:
1) |z| = 7; 2) |z| = 4; 3) |z| = 5; 4) другой ответ?
4. z = –12 + 5i. Какое из утверждений является справедливым: число z удалено
от точки О(0; 0) на комплексной плоскости на расстояние
1) 7; 2) 17; 3) 13; 4) другой ответ?
5. z1 = 3 + 4i; z2 = –3 + 4i. Какое из утверждений является справедливым:
1) |z 1 | + |z 2 | < |z 1 + z 2 |; 2) |z 1 | + |z 2 | = |z 1 + z 2 |; 3) |z 1 | + |z 2 | > |z 1 + z 2 |;
4) другой ответ?
6. z1 = 7 – 5i; z2 = 6 + 8i. Какое из утверждений является справедливым:
1) ||z1| – |z2|| = |z1 – z2|; 2) ||z1| – |z2|| < |z1 – z2|; 3) ||z1| > |z2|| > |z1 – z2|;
4) другой ответ?
1
7. z1 = –1 +
i; z2 = 3 – i. Какое из утверждений является справедливым:
3
1) arg z1 = arg z2; 2) arg z1 = 2 arg z2; 3) arg z1 = 3 arg z2; 4) другой ответ?
9π
7π
8. arg z1 =
; arg z2 =
. Какое из утверждений является справедливым:
10
10
63π
8π
2π
1) arg (z 1 z 2 ) =
; 2) arg (z 1 z 2 ) =
; 3) arg (z 1 z 2 ) = −
;
100
5
5
4) другой ответ?
8π
7π
9. arg z1 =
; arg z2 = −
. Какое из утверждений является справедливым:
9
9
z 
z 
z 
8
5π
π
1) arg  1  = ; 2) arg  1  =
; 3) arg  1  = − ;
7
3
3
 z2 
 z2 
 z2 
4) другой ответ?
π
10. arg z = . Число z было возведено в степень n, после чего получено число
11
n
z , такое что arg zn = π. Какое из утверждений является справедливым:
11π
1) n = 0; 2) n = 1; 3) n =
; 4) другой ответ?
10
11. z1 = 7 – 8i; z2 = –7 – 8i. Какое из утверждений является справедливым:
1) | z15 | = | z 25 |; 2) | z15 | < | z 25 |; 3) | z15 | и | z 25 | несравнимы; 4) другой ответ?
π
12. Комплексный вектор числа z с действительной осью Ох образует угол ϕ = .
4
На этот вектор подействовали оператором А, после чего комплексный вектор перешел в противоположный ему вектор. Какое из утверждений является справедливым:
75
π
i
2
π
−i
4
1) А = e ; 2) А = eiπ ; 3) А = e ; 4) другой ответ?
13. Комплексное число z1, заданное вектором OM 1 , было умножено на комплексное число z2, заданное вектором OM 2 , после чего модуль вектора
OM 1 стал втрое меньше. Что можно сказать о векторе OM 2 ? Какое из утверждений является справедливым:
1
1) | OM 2 | = 3; 2) | OM 2 | = 3| OM 1 |; 3) | OM 2 | = ; 4) другой ответ?
3
14. Число z1 = 9 – 2i является корнем квадратного уравнения с действительными
коэффициентами. Какое из утверждений является справедливым:
1) z2 = 2 – 9i; 2) z2 = –9 – 2i; 3) z2 = 9 + 2i; 4) другой ответ?
15. Сколько значений имеет корень 6 − 1 на множестве комплексных чисел?
Какое из утверждений является справедливым:
1) одно значение; 2) шесть значений; 3) ни одного значения; 4) другой
ответ?
o
o
16. А = cos 17 + i sin 17 . Записать величину А в показательной форме. Какое
из утверждений является справедливым:
o
o
1) А = 17еі; 2) А = 17еі17 ; 3) А = еі17 ; 4) другой ответ?
17. Какое из утверждений является справедливым:
1) z1 + z2 = z1 – z 2 ; 2) z1 + z 2 = z1 + z 2 ; 3) z1 + z 2 = z1 + z2; 4) другой
ответ?
18. Какое из утверждений является справедливым:
1) z1 z2 = z1 z 2 ; 2) z1 z2 = z1 : z 2 ; 3) z1 z2 = z1 z2; 4) другой ответ?
19. Какое из утверждений является справедливым:
z  z
z  z
z  z
1)  1  = 1 ; 2)  1  = 1 ; 3)  1  = 2 ; 4) другой ответ?
 z2  z 2
 z2  z 2
 z2  z1
20. Какое из утверждений является справедливым:
1) | z | = | z | ; 2) | z | =3 | z | ; 3) | z | = 4 | z | ; 4) другой ответ?
21. Какую часть комплексной плоскости описывает неравенство
| z | < 3?
Какое из утверждений является справедливым:
1) круг с центром в точке О(0; 0) и радиусом R = 3;
2) круг с центром в точке О(0; 0) и радиусом R = 3, которому не принадлежат точки на окружности;
3) множество точек комплексной плоскости , которые лежат вне
окружности с центром в точке О(0; 0) и радиусом R = 3;
4) другой ответ?
22. Какую часть комплексной плоскости описывает система неравенств
−1 ≤ Im z ≤ 3;

 0 ≤ Re z ≤ 4?
Какое из утверждений является справедливым:
76
1) часть комплексной плоскости, изображенная на рис. 1;
2) часть комплексной плоскости, изображенная на рис. 2;
3) часть комплексной плоскости, изображенная на рис. 3;
4) другой ответ?
Im z
Im z
3
О
–1
Im z
3
Re z
4
Рисунок 1
О
–1
3
Re z
4
Рисунок 2
О
–1
Re z
4
Рисунок 3
23. Области D1 и D 2 заданы соответственно системами неравенств
 | z | ≤ 4;
 | z | ≤ 2;


D1: 
D 2: 
π
π
0 ≤ arg z ≤ 4 ;
0 ≤ arg z ≤ 2 .
S1 и S2 – площади областей D1 и D2 соответственно. Какое из утверждений
является справедливым:
1) S1 = S2; 2) S1 = 2S2; 3) S1 =4S2; 4) другой ответ?
24. На комплексной плоскости задана область D такой системой неравенств:
3π
π
 ≤ arg z ≤ ;
4
4
 | z |≤ 1.
Какое из утверждений является справедливым:
1
1
1) S = π (кв. ед.); 2) S = π (кв. ед.); 3) S = π (кв. ед.), где S – площадь
4
2
области D; 4) другой ответ?
25. Все значения n z нанесены на окружность с центром в начале координат,
радиус которой r =| z | . При этом разность аргументов двух соседних значений равняется 45°. Какое из утверждений является справедливым:
1) n = 8; 2) n = 10; 3) n = 9; 4) другой ответ?
77
ЗАДАЧИ ДЛЯ САМОСТОЯТЕЛЬНОГО РЕШЕНИЯ
Задача № 1 Изобразить точками на комплексной плоскости такие числа:
1) z1 = 2 + і; 2) z2 = 2 – і; 3) z3 = –2 + 3і; 4) z4 = –3 – 2і; 5) z5 = 0 + 5і;
6) z6 = 5 + 0і; 7) z7 = 7; 8) z8 = – 3і.
Задача № 2 Записать в алгебраической форме комплексные числа, соответствующие точкам на комплексной плоскости (рис. 1).
Im z
M4
2
M2
M1
Re z
О
−10
M6
−6
5
9
−2
M5
M3
Рисунок 1
Задача № 3 Записать аффиксы заданных комплексных чисел и построить
соответствующие им комплексные векторы:
1) z = 2 – 3і; 2) z –2 + 3і; 3) z = –2 – 3і; 4) z = 2 – і 3 ; 5) z = 2і;
6) z = 4і; 7) z = 2 – і 3 ; 8) z = – 6і; 9) z = 3і – 0,5.
Задача № 4 Преобразовать заданные выражения, воспользовавшись числом i:
1) − 3 + 4 27 ; 2) − 5 + 4 − 125 − 5 − 125 .
Задача № 5 Проверить справедливость равенств:
16
4
1) 4 = ±2і; 2) −
= ± і; 3) 0,25 = ±0,5і.
5
25
1
3
Задача № 6 Для заданных чисел 3; 2 ; –4і; 2 – 3і; – + i
назвать: 1)
2
2
противоположное число; 2) комплексно-сопряжённое число.
Задача № 7 Задано число z = –3 + i. Построить на комплексной плоскости точки, которые изображают комплексно-сопряжённое и противоположное
числа для данного числа.
Задача № 8 Задана точка, соответствующая числу z = –2 + і 2 . Каким
числам отвечают точки, симметричные заданной, относительно: 1) действительной оси; 2) мнимой оси; 3) начала координат?
Задача № 9 Изобразить на комплексной плоскости множество точек, которые удовлетворяют условию:
1) Re z = 3; 2) Re z –4 = 0; 3) Re z < 2; 4) Im z = 5; 5) Im z ≥ 5.
78
Задача № 10 Найти множество точек комплексной плоскости, которые
удовлетворяют условию:
1) Re z > 3; 2) Re z < 1; 3) Im z = –3; 4) Im z ≥ 3.
Задача № 11 Выполнить сложение комплексных чисел в алгебраиической и геометрической форме:
1) (5 + 4 і ) + (3 – 7 і );
2) (2 – 8 і ) + (5 – і ); 3) (2 + 5 і ) + (–2 – 2 і );
4) (4 + 3 і ) + (–4 + 3 і ); 5) (2 – 4 і ) + (–2 + 4 і ).
Задача № 12 Выполнить действия над комплексными числами:
1) (0,5 – 3,2і) + (1,5 – 0,8і) + (1 – 4і); 2) 2 + (3 + 4і) + 2і + (–6 – 7і);
 3 2   1 5   3

3) 1 + i  + 1 − i  +  − − 2i  ; 4) (0,12 – 1,4і) + (1,08 + 0,4і) + (2,5 – 0,2і);
 4 3   2 3   4

5) (a – bi) + (c + di); 6) (3x – 4yi) + (–x + 2yi).
Задача № 13 Определить разность комплексных чисел:
1) (5 + 3 і ) – (2 + i); 2) (–2 + 4 і ) – (2 + i); 3) (1 + i) – (5 + 3 і );
4) (2 – 3і) – (2 + 3і).
Задача № 14 Выполнить действия:
1) (5 + 4і) – (2 – 3і); 2) (2 + i) – (3 – 6і) (1 – i);
1 1  3 2  3 5 
3)  − i  −  + i  +  − i  ;
2 4  5 3  4 6 
4) (2c – 8di) – ((5c – 2di) + (c – di) – (–4c + 3di));
5) (m – ni) + (3m – 2ni) – ((– m – ni) – (5m + 10ni)).
Задача № 15 І. При каком условии сумма двух комплексных чисел а1+b1i
и а2 + b2i будет:
1) вещественным числом; 2) мнимым числом?
ІІ. При каком условии разность двух комплексных чисел а1+b1i и а2 + b2i
является:
1) вещественным числом; 2) мнимым числом?
Задача № 16 Выполнить умножение комплексных чисел в алгебраической и геометрической формах.
1) 2і⋅3і; 2) 4і⋅2 2 і; 3) 5і (–4і); 4) 2,5і⋅4і; 5) (3 + 5і) 2; 6) (1 – і) (–4);
7) (–2 – 3і) 5; 8) (–3 + 4і) 2і.
Задача № 17 Выполнить умножение комплексных чисел:
1) (2 – 3і) (4 – і); 2) (1 – 2і) (5 – і); 3) (0,5 + 0,2і) (2 + 3і);
4) ( 2 − i )( 3 + i 2 ) ; 5) (5 + і) (5 – і); 6) (1 – і) (1 – і).
Задача № 18 Выполнить умножение комплексных чисел:
1) (–3 – 4і) 2і; 2) (–8 + 7і)(–3і); 3) (4 – 9і)⋅0; 4) (1 + і) (1 – і);
5) (а + bi) (а – bi); 6) (2 + 3і) (–4 + і); 7) (3 + 5і) (5 + 3і); 8) i3(3 – 2i);
9) (2 – 3i) (–1 – i) (3 + 4i).
Задача № 19
І. При каком условии произведение двух комплексных чисел а1 + b1i и а2 +
b2i является: 1)действительным числом; 2) мнимым числом? Привести примеры.
ІІ. Написать два комплексных числа, для которых сумма и произведение –
действительные числа.
79
Задача № 20 Разложить на комплексные множители такие выражения:
1) a2 + b2; 2) a2 + 9b2; 3) a4 + b4; 4) 4m2 + 25n2; 5) a2 + 1; 6) 5a2 + b;
b2
2
; 8) a + b.
7) a +
9
Задача № 21 Выполнить деление чисел:
5
1) 6і : 2; 2) 10і : (–4); 3) 6і : (–2і); 4) 10і : 2і; 5) 9і : (–0,5і); 6) ;
3i
6
4
7)
; 8)
.
1 − 2i
1 + 2i
Задача № 22 Выполнить деление комплексных чисел в алгебраической и
геометрической форме:
1 + 3i
4 − 2i
5 −i
− 3 + 6i
5 − 2i
;
2)
;
3)
; 5)
;
1)
;
4)
1 − 2i
1 − 3i
1 + 2i
5 − 2i
− 1 + 3i
−3 2 +i
6)
.
1 + 3 2i
Задача № 23 Выполнить действия:
1) (3 + 4і) + 5(2 – 3і) – 3(2 – 7і); 2) (9 + 16і) (8 – 3і) + 7(12 – 5і);
3) (9 + 5і) (4 – 3і) + (6 – і) (6 + і); 4) (3 – 7і) (5 + 6і) – (9 – 8і) (3 + 12і);
7 − i 1+ i
11 − 8i
12 − 5i 4 − i
− (4 + 8i )(2 − 7i ) ; 6)
−
(8 − i )(8 + i ) ; 7)
⋅
;
5)
2 + 3i
12 + 5i 5 + i
3 + i 1− i
3 − i 42 + 2i
 2 − 5i   6 − 7i 
 7 − 2i  1 + 3i
8)
; 10) 
⋅
; 9) 


.
3 + i 3 + 5i
 7 + 2i  4 − i
 4+i   4−i 
Задача № 24 Доказать равенства:
1) (1 – і)2 + (1 – і)2 = 0; 2) (1 + і)3 – (1 – і)3 = 4і; 3) (а – 1 – і)(а + 1 + і) = а2 – 2і;
1
1
4)
−
= −i .
1+ i 1− i
Задача № 25 Выполнить действия:
4
4
 1 + 7i   1 − 7i 
1+ i 1− i
a + bi a + bi
 + 
 ;
1)
; 2)
+
−
; 3) 
1− i 1+ i
c + bi c − bi
2
2

 

3
3
(a + i) − (a − i)
4)
.
(a + i) 2 − (a − i) 2
Задача № 26 Выполнить действия:
a+i n
15 − i 2
a − bi
7 − 3i
6 −i
1)
; 2)
; 3)
; 4)
; 5)
;
b + ai
1 + 3i
a−i n
1+ i 3
6 − 2i
− 2 +i 6
−2 3+i
m
a
a
6)
; 7)
; 8)
; 9)
; 10)
.
a + 6i
−1+ i 3
1+ 2 3i
i m
a + 2i a
Задача № 27 Заданы числа: 2 – 3і и 4 + 5і. Найти модуль:
1) суммы; 2) разности; 3) произведения; 4) частного этих чисел.
80
Задача № 28
I. Доказать, что модуль произведения двух комплексных чисел равняется
произведению модулей, то есть |(a1 + b1i) (a2 + b2i)| = |a1 + b1i| |a2 + b2i|.
II. Доказать, что модуль частного двух комплексных чисел равняется чаa +bi |a +bi|
стному модулей, то есть 1 1 = 1 1 , если а2 + b2i ≠ 0 + 0i.
a2 + b2 i | a 2 + b2 i |
Задача № 29 Найти значение выражений:
1) і6 + і20 + і30 + і36 + і54; 2) і + і2 + і3 + і4 + і5; 3) і + і11 + і21 + і31 + і41;
4) і⋅і2⋅і3⋅і4; 5) і6 + і16 + і26 + і36 + і46 + і56; 6) і3 + і13 + і23 + і33 + і43 + і53;
1 1
7) і + і2 + і3 + і4 + ... + іn; ( n > 4); 8) і⋅і2⋅і3⋅і4⋅ …⋅i100; 9) 3 + 5 ;
i
i
1
1
1
10) 13 + 23 + 33 .
i
i
i
Задача № 30 Найти значение выражений (в алгебраической форме):
3
3
 −1+ 3i 
 −1− 3 i 
 ; 4) 
 ; 5) (0,5 2 + 0,5i ) 2 .
1) (1 – і) ; 2) (1 + і) ; 3) 
2
2




Задача № 31 Найти значение выражений (в алгебраическлй форме):
1) (і(2 – і))2; 2) (2і(3 – 4і))2; 3) ((3і – 5)2і)2; 4) ((5 – і)(5 + і))2;
5) ((6 – 2і)(6 + 2і))2(1 + і)2; 6) (3 + і)2(1 – і)3; 7) (2 + аі)2; 8) (а – 3bi)2;
12
17
2
1 i 3
 ; 13) (1 + i)3;
9) (2c + 3di) ; 10) (1 + i) ; 11) (1 – i) ; 12)  −
2 
2
14) (2 − i 3 ) 3 ; 15) (3 − i 3 ) 3 ; 16) (a + bi)3 – (a – bi)3; 17) (x – 1 – i)(x + 1 + i);
2
4
4
4
4 n +1
6
4n+ 2
1 + i 
1+i 
1+i 
1+i 
; 22) 
18) ((1 + i)(1 – i)) ; 19) 
 ; 20) 
 ; 21) 


1 − i 
1 − i 
1 − i 
 2
Задача № 32 Найти значение выражений (в алгебраической форме):
6
.
−4
 2 
1) (1 + i) ; 2) (4 + 4i) ; 3) (25 – 17i) ; 4) 
 ;
1
−
7
i


–1
–2
2
3
0
(
5) ( (2 + 2i )(2 − 2i ) ) ; 6) ( 3 + i )( 3 − i )
)
−
3
2
.
Задача № 33 Найти действительные числа х и у из уравнений:
1) (х + у) + (х – у)і = 2 + 4і;
2) (х + у) + (х – у)і = 4і;
3) (х + у) + (х – у)і = 2;
4) (у + 2х) + (2у + 4х)і = 0;
5) (х + 1,5у) + (2х + 3у)і = 13і;
6) (х + 2у) + (3х – у) = 5 +і;
2
7) (х + у) + 6 +хі = 5(х + у) + (у + 1)і.
Задача № 34 Найти действительные числа х и у из условия равенства
комплексных чисел:
1) 9 +2іх + 4іу = 10і + 5х – 6у; 2) 2 + 5іх – 3іу = 14і + 3х – 5у;
8i
10
3) (1 + і)х + (1 – і)у = 3 – і; 4)
+ іу – 2 = 7і –
+ у;
x
x
81
5) (4 – і)х + (2 + 5і)у = 8 + 9і; 6) (3 + і)х – (1 – 2і)у = 7;
7) 2іх + 3іу + 17 = 3х + 2у + 18і; 8) 5і – 2у + (х + у)і = 4 + 5і;
1
9) х2 – 5(х – 1) + 4і = уі – 1; 10) – 4іу = 4;
x
y − ix 4 + i
11) (3х – іу)(12 – 8і) = (7 + 5і)(2у – 5іх); 12)
=
.
x + iy 4i − 1
Задача № 35 Найти действительные числа х и у, если:
1) (1 – і)х + (1 + і)у = 1 – 3і; 2) (2 + 3і)х2 – (3 – 2і)у = 2х – 3у + 15і;
1
3) (4х2 + 3ху) + (2ху – 3х2)і = 4у2 – х2 + (3ху – 2у)і.
2
Задача № 36 Найти действительные числа х и у, если:
х −1 у −1
1)
+
= i ; 2) у2 + іу2 + 6 + і = 2х + хі.
3+i 3−i
Задача № 37 Решить уравнение:
1) z2 + 16 = 0; 2) z2 – 2z + 2 = 0;
3) z2 + 2 = 0;
4) 4z2 + 4z + 5 = 0.
Задача № 38 Решить уравнение:
1) 3z2 + 5 = 0; 2) z2 – 14z + 74 = 0; 3) z2 + 2z + 5 = 0; 4) 4z2 – 2z + 1 = 0;
5) z2 + 18z + 81 = 0; 6) z2 + 4z + 3 = 0.
Задача № 39 Решить систему уравнений:
 x + y = 6;
2 x − 3 y = 1;
1) 
2) 
 xy = 45;
 xy = 1.
Задача № 40 Составить квадратное уравнение, если заданы его корни:
3−i
3+i
1) z1 = 2 + i, z2 = 2 – i; 2) z1 =
, z2 =
; 3) z1 = 2 – i, z2 = 3 – 2i;
4
4
2−i
4) z1 =
, z2 = 1 + i.
1+ i
Задача № 41 Составить квадратное уравнение по его корням:
−1+ 4 5 i
−1− 4 5 i
1
1
1) z1 =
, z2 =
; 2) z1 = 3 – i , z2 = 3 + i ;
3
3
2
2
2−i
, z2 = 1 + i.
3) z1 =
1+ i
Задача № 42 Корнями квадратного уравнения являются комплексные
2
числа z1 и z2, такие, что z1 = z2. Какими в этом случае будут коэффициенты
3
квадратного уравнения: действительными или комплексными?
Задача № 43 Составить квадратное уравнение с действительными коэффициентами, один из корней которого задан:
32 − i
1) z1 = 5 – i; 2) z1 = –3i; 3) z1 = (3 – i)(2і – 4); 4) z1 = (4 – і)2; 5) z1 =
.
1 − 3i
Задача № 44 Решить квадратные уравнения и проверить свойство суммы
и произведения корней для каждого из уравнений:
1) х2 – 6х + 13 = 0; 2) х2 – 4х + 6 = 0.
82
Задача № 45
І. При каких действительных значениях а корни квадратного уравнения
(а + 5)х2 + 2ах + (а – 1) = 0 будут комплексными?
ІІ. При каком значении k корни квадратного уравнения х2+2(3 + i)х + k =0 будут равны между собой?
Задача № 46 Заданы модули и аргументы комплексных чисел z1; z2; z3.
Построить на комплексной плоскости векторы по заданным модулям и аргументами.
1) r1 = 3, ϕ1 = 180°; 2) r2 = 3, ϕ2 = –90°; 3) r3 = 3, ϕ3 = –60°.
Задача № 47 По заданным модулям или аргументам комплексных чисел
найти соответственно модули или аргументы: 1) комплексно-спряжённых чисел; 2) противоположных чисел.
π
π
1) |z1| = 3; 2) |z2| = 5; 3) arg z3 = – ; 4) arg z4 = – ; 5) arg z5 = π;
2
3
2π
π
2π
6) arg z6 = – ; 7) < arg z7 <
.
3
6
3
Задача № 48 Найти на комплексной плоскости геометрическое место точек, соответствующее комплексным числам, для которых:
π
1) | z | = 4; 2) | z | < 3; 3) | z | > 4; 4) 0 < arg ϕ < .
4
Задача № 49 Найти на комплексной плоскости множество точек, которые
удовлетворяют таким условиям:
π
1) ρ = 1, ϕ = ; 2) ρ = 2; 3) ρ ≤ 3; 4) ρ < 3; 5) 2 < ρ < 3;
4
π
π
6) ϕ = ; 7) 9 < ϕ ≤ .
3
6
Задача № 50 Где на комплексной плоскости находятся точки, удовлетворяющие условию:
1) |z| = 2; 2) 1 < |z| < 2?
Задача № 51 Где на комплексной плоскости находятся точки, удовлетворяющие условию:
1) |z + і| ≤ 1;
2) |z + 2| < 4, если z = x + iy?
Задача № 52 Найти множество точек комплексной плоскости, для которых выполняются соотношения:
1) Re z > 0; 2) 0 ≤ Re z 1; 3) Im z ≤ 1; 4) |Im z| ≥ 2; 5) |z| 1;
1 ≤ Re z ≤ 2;
1 ≤ | z | ≤ 3;


6) 2 ≤ |z| ≤ 5; 7) |z – i| > 1; 8) 
π 9)  π
3π
0 < arg z ≤ 4 ;
 2 ≤ arg z ≤ 4 ;
2 ≤| z |≤ 4;
10) 
Im z ≤ 2.
83
Задача № 53 Представить заданные числа в тригонометрической и показательной формах:
3 1
− i;
1) 3і; 2) –1 + і; 3) 1 –і 3 ; 4) 3 – і; 5) –3 + 4і; 6)
2 2
7) 5 – 12і; 8) –4 – 3і.
Задача № 54 Представить заданные числа в тригонометрической и показательной формах:
1) z = і; 2) z = –5; 3) z = 3; 4) z = 1 + і; 5) z = 6 – 6і; 6) z = – 3 + і;
7) z = –2 – 2і 3 ; 8) z = –3 + 2i.
Задача № 55 Представить в алгебраической форме комплексные числа:
π
π
3π
3π 


1) 2 cos + i sin  ;
2) 5(сos 0 + i sin 0);
3) 3 cos + i sin  ;
6
6
2
2


π
i
3
π
i
4
π
− i
4
4) сos 60° + i sin 60°; 5) 7e ;
6) 9e ;
7) 16e .
Задача № 56 Выяснить, является ли заданная форма комплексного числа
его тригонометрической формой? Если нет, то представить комплексное число
в тригонометрической форме:

π
 π
1) 5  cos  −  + i sin  ; 2) 6(сos 6π + isin 6); 3) 11(сos 30 °– i sin 30°).
4
 4

Задача № 57 Найти произведение заданных чисел и записать ответ в показательной форме:
1) 2(сos 60°+ і sin 60°) ⋅ 3(сos 45° + і sin 45°);
2) 2 (сos 30° + і sin 30°) ⋅ 2 2 (сos 60° + і sin 60°);
π
π 
π
π

3) 3 cos + i sin  ⋅ 2 cos + i sin  ;
6
6 
3
3

3
4) 3 (сos 120°+ і sin 120°) ⋅
(сos 150° + і sin 150°).
2
Задача № 58 Найти произведение комплексных чисел, ответ записать в
показательной форме:
π
π 
π
π

1)  cos + i sin   cos + i sin  ;
12
12  
4
4

π
π
5π
5π 

2) 3  cos + i sin   cos + i sin  ;
8
8 
24
24 

π
π

 π
 π 
3) 2  cos + i sin   cos  −  + i sin  −   ;
3
3 

 4
 4 
4) (сos 50° + і sin 50°) (сos 40° + і sin 40°);
5) 2 (сos 85° + іsin 85°) ⋅ 6 (сos 95° + і sin 95°);
6) 4(сos 10° + і sin 10°) ⋅ 2(сos 35° + і sin 35°).
Задача № 59 Выполнить умножение комплексных чисел в тригонометрической и показательной форме:
84
2
6
 1 1 
 ; 2) (1 + і 3 ) (–2 – 2і 3 ); 3) (1 + і) (3 + 3і 3 );
1)  + i   −
+i
6 
 4 4  6
4) (5 + 5і) (сos 15° + і sin 15°).
Задача № 60 Выполнить деление чисел в тригонометрической и показательной форме:
π
π  
π
π 

1) 6 cos + i sin  :  2 cos + i sin   ;
2
2  
6
6 

3π
3π  
π
π

2) 3 cos + i sin  :  cos + i sin  ;
4
4 
2
2

3) (сos 210° + і sin 210°):(сos 150° + і sin 150°);
π
π 
π
π

4) 2 cos + i sin  :  cos + i sin  ;
3
3 
4
4

  π
 π    π 
 π 
5) 2 cos −  + i sin  −   :  cos −  + i sin −   ;
 3    6 
 6 
  3
6) 8(сos 150° + і sin 150°) : (4(сos(–120° + і sin(–120°))).
Задача № 61 Найти значение выражений, пользуясь первой формулой
Муавра, ответы записать в тригонометрической и показательной форме:
2
 3 1 
− i  ;
1) 
2
2 

8
π
π 
 
2)  2 cos + i sin   ; 3) (cos 35° + і sin 35°)–12;
8
8 
 
9
 − i π3 
3
3 
iπ 6
4)  −
i  ; 5) ( 7e ) ; 6)  3e  .
2
2




Задача № 62 Выполнить действия, ответ записать в алгебраической фор-
ме:
1)
2 ( cos150o + i sin150o )
; 2)
cos170o + i sin170o
;
4 ( cos100o + i sin100o )
5π
5π 

3  cos + i sin 
6
6 

3) 5(сos 40° + і sin 40°) (сos 50 °+ і sin 50°);
4) 5(сos 20° + і sin 20°) ⋅ 7(сos 100° + і sin 100°);
8π
8π  
2π
2π  
4π
4π 

5)  cos + i sin   cos
+ i sin  ⋅ 2  cos
+ i sin  ;
15
15  
3
3  
5
5 

o
o
o
o
cos130 + i sin130 cos130 + i sin130
6)
;
cos 40o + i sin 40o cos 40o + i sin 40o
− cos100o + i sin100o 2(cos107o + i sin107o )
7)
;
cos 40o − i sin 40o
5(cos 47o + i sin 47o )
8)
7eiπ
i
π
5e 2
i
;
9)
π
12e 7
i
π
6e 6
;
10)
5e −iπ
.
6e − i 0,3π
85
Задача № 63 Упростить по первой формуле Муавра в тригонометрической и показательной формах:
100
8
8
8
 3 i
50
1 
 1
1) 
−  ; 2) 3 + i ; 3) 
−
i  ; 4) 3 + 3i + 3 − 3i ;
2 
 2
 2 2
(5 + 5i ) 5
( 3 − i)5
; 6)
.
5)
(4 − 4i ) 3
(3 + 3i ) 2
Задача № 64 Упростить по первой формуле Муавра в тригонометрической и показательной формах:
1) (3(сos 50° + і sin 50°))6; 2) (2(сos 15° + і sin 15°))4; 3) (сos 50° + і sin 50°)8;
(
)
(
) (
10
)
8
π
π
π
π


4)  cos + i sin  ; 5) (2(сos 60° + і sin 60°))6; 6)  − cos + i sin  .
4
4
4
4


Задача № 65 Доказать тождество:
6
6
5
5
 −1+ i 3   −1 − i 3 
 −1+ i 3   −1− i 3 
 + 
 = 2 ; 2) 
 + 
 = −1 .
1) 
2
2
2
2

 


 

Задача № 66 Проверить справедливость равенств:
1
3
; 2) (2(сos 45° + і sin 45°))8 = 256;
1) (сos 60° + і sin 60°)8 = − + i
2
2
6
3) ( 3 (сos 45° + і sin 45°)) = –27і; 4) ( 2 (сos 56°15′ + і sin 56°15′))8 = 16і.
Задача № 67 Найти все значения корней:
1) 3 − 1 ; 2) 4 − 1 ; 3) 6 8 .
Ответ записать в алгебраической форме.
Задача № 68 Найти все значения корней:
1) 4 16 ; 2) 3 − 2 + 2i ; 3) 4 − 7 − 24i ; 4) 5 cos 225o + i sin 225o ; 5) e i 0,1π .
Ответ записать в тригонометрической форме.
Задача № 69 Найти все значения корней и построить их геометрическое
изображение:
1)
5
cos 225o + i sin 225o ;
o
o
2) 5 16 ( cos 240o + i sin 240o ) ;
4
i
π
6
3) cos120 + i sin120 ;
4) 16e .
Задача № 70 Доказать , что аффиксы комплексных чисел z 1 =8+13i, z2
= 13 + 24i и z3 = 21 + 34i не принадлежат одной прямой.
Задача № 71 С помощью формулы Эйлера еіх = сos х + isin х доказать основное тригонометрическое тождество sin2x + cos2x = 1.
Задача № 72 С помощью формулы Эйлера и равенства еіхеіу = еі(х + у) доказать формулу сложения аргументов для синуса и косинуса:
sin(x + y) = sin x cos y + sin y cos x; cos (x + y) = cos x cos y – sin x sin y.
Задача № 73 С помощью формулы Эйлера и равенства (еіх)2 = е2іх получить формулы для синуса и косинуса двойного угла:
sin 2x = 2sin x cos y; cos 2x = cos2x – sin2x.
3
86
Задача № 74 С помощью формулы Эйлера и равенства (еіх)n = еіnх доказать первую формулу Муавра:
(r(cos ϕ + i sin ϕ))n = rn(cos nϕ + i sin nϕ).
Задача № 75 С помощью комплексных чисел доказать равенство
sin 2 nϕ
sin ϕ + sin 3ϕ + sin 5ϕ + … + sin (2n – 1) ϕ =
.
sin ϕ
87
ОТВЕТЫ К ЗАДАЧАМ
ДЛЯ САМОСТОЯТЕЛЬНОГО РЕШЕНИЯ
№ 1.
Im z
z5
z3
z1
O
z6
z7
Re z
z2
z4
z8
Рисунок 1
№ 2. 1) z1 = 9 + i; 2) z2 = 5 + 2i; 3) z3 = 5 – 2i; 4) z4 = –6 + 2i; 5) z5 = –6 – 2i;
6) z6 = –10 – i.
№ 3. 1) М1(2; –3); 2) М2(–2; 3); 3) М3(–2; –3); 4) М4( 2 ;
6) М6(0; 4); 7) М7(2; –
3 ); 5) М5(0; 2);
3 ); 8) М8(0; –6); 9) М9(–0,5; 3).
Im z
4 М6
М2 М
9
М5
М4
Re z
−2
O
2
− 3
М3
М7
М1
−6 М8
Рисунок 2
№ 4. 1) 12 3 –
3 і; 2) –4 5 і.
1
3
№ 6. 1) –3; – 2 ; 4і; –2 + 3і; − i
; 2) 3;
2
2
№ 8. 1) –2 – і 2 ; 2) 2 + і 2 ; 3) 2 – і 2 .
2 ; 4і; 2 + 3і; −
1
3
−i
.
2
2
88
№ 9.
Im z
Im z
х=4
2)
х=3
1)
Re z
О
О
3
Рисунок 3
Im z
О
4
Рисунок 4
Im z
у=5
4)
х=2
3)
)
Re z
Im z
у=5
5)
Re z
Re z
Re z
2
О
О
Рисунок 6
Рисунок 5
Рисунок 7
№ 10.
Im z
Im z
О
х=1
2)
х=3
1)
Re z
О
3
1
Рисунок 9
Рисунок 8
Im z
Im z
3)
Re z
Re z
4)
у=3
О
Re z
у = –3
О
Рисунок 10
Рисунок 11
89
№ 11.
Im z
Im z
2) 7 – 9і
1) 8 – 3і
Re z
5
2
О
–1
7
Re z
О
3
5
8
8 – 3і
–3
7 – 9і
–8
–9
–7
Рисунок 12
Рисунок 13
Im z
3) 3і
5i
Im z
6i
4) 6і
3і
3і
Re z
–2
О 2
–2i
Re z
–4
Рисунок 14
О
4
Рисунок 15
5) 0
Im z
4
Re z
–2 О
2
–4
Рисунок 16
5
– 3і; 4) 3,7 – 1,2і; 5) (а + с) + (d – b)i; 6) 2x – 2yi.
2
№ 13. 1) 3 + 2і; 2)–4 + 3і; 3) –4 – 2і; 4) –6і.
13 7
№ 14. 1) 3 + 7і; 2) –2 + 8і; 3)
− i ; 4) –8с – 2di; 5) 10m + 8ni.
20 4
№ 15. І 1) b1 = –b2; 2) a1 = –a2; II 1) b1 = b2; 2) a1 = a2.
№ 12. 1) 3 – 8і; 2) –1 – і; 3)
90
№ 16.
1) –6
2) –8 2 і
Im z
Im z
4
2 2
3
2
Re z
Re z
О
–6
О
–8 2
Рисунок 17
Рисунок 18
Im z
5
3) 20
4) –10
Im z
4
2,5
Re z
О
Re z
О
20
–4
Рисунок 20
Рисунок 19
Im z
5) 6 + 10і
5
6) –4 + 4і
6 + 10і
4
–4 – 4і
Re z
О
3
Re z
О
–1
–4
1
Рисунок 22
Рисунок 21
Im z
7) –10 – 15і
8) –8 – 6і
5
О
–10
Im z
Im z
Re z
4
–2
–3
Re z
–8
–10 – 15і
–3
–8 – 6і
–15
Рисунок 23
О
–6
Рисунок 24
91
№ 17. 1) 5 – 14і; 2) 3 – 11і; 3) 0,4 + 1,9і; 4) ( 6 +
2 ) + і(2 –
3 );
5) 26; 6) –2і.
№ 18. 1) 14; 2) 21 + 24і; 3) 0; 4) 2; 5) а2 + b2; 6) –11 –10i; 7) 34i; 8) –2 – 3i;
9) –19 – 17i.
№ 19. І. 1) а1b2 = – b1а2; 2) а1а2 = b1b2; ІІ. 1) z1 = a – bi; 2) z2 = a + bi.
№ 20. 1) (a + bi) (a – bi); 2) (a + 3bi) (a – 3bi); 3) (a2 + b2i) (a2 – b2i);
4) (2m + 5ni) (2m – 5ni); 5) (a + i)(a – i); 6) ( 5 a +
b 
b 

7)  a + i   a − i  ; 8) ( a +
3 
3 

b i)( a –
b i)( 5 a –
b i);
b i).
5
5
6 12
4 8
№ 21. 1) 3i; 2) − i; 3) –3; 4) 5; 5) –18; 6) − i; 7) + i ; 8) − i .
2
3
5 5
5 5
№ 22.
1) −
1
3
+
i
2 2
2)
Im z
2 5 2
−
i
3
3
Im z
3
−
2
1
3
+
i
2 2
Re z
Re z
–0,5 О
1
О
−5 2
3
9 8
+ i
5 5 Im z
4)
7
5
+
i
9 9
Im z
9 8
+ i
5 5
Re z
О
1
–2
2 5 2
−
i
3
3
Рисунок 26
Рисунок 25
8
5
4
– 2
– 3
3)
1
5
5
9
О
7
5
+
i
9 9
7
9
–2
Рисунок 27
Рисунок 28
Re z
5
92
5)
3 +3 2 3− 6
+
i ≈ 1,49 − 0,14i
4
4
6) і.
Im z
Im z
3 2
6
3
i
Re z
–
3
1,49
–1 О
1,49
–
0,14
i
–0,14
Рисунок 29
Re z
О
3 2
1
Рисунок 30
834 107
+
i;
13
13
1581 7365
409 55
6) −
+
i ; 7) 1 + 2і; 8) (2 – 3 3 ) + (–3 – 2 3 )і; 9)
+
i;
338 338
901 901
23 44
− i.
10) −
17 17
3a 2 − 1
2b 2
2ab
№ 25. 1) 0; 2) 2
−
i ; 3) 1; 4)
.
2a
c + b2 c2 + b2
a 2 − n 2a n
15 − 6 15 3 + 2
№ 26. 1) 2
+ 2
i ; 2)
i;
−
4
4
a +n a +n
1 12
4
6
2 + 18
3) –і; 4) − − i ; 5) +
i ; 6)
; 7) і; 8) –і m ;
5 5
5 10
4
a2
6a
a
2
9) 2
−
− 2
i ; 10)
i.
a+4 a+4
a + 36 a + 36
1
№ 27. 1) 2 10 ; 2) 2 7 ; 3) 533 ; 4)
533 .
41
№ 29. 1) –1; 2) і; 3) і; 4) –1; 5) 0; 6) 0;
0, åñëè n = 4k ;
i, åñëè n = 4k + 1;

7) 
8) –1; 9) 0; 10) –і.
i
−
1,
åñëè
n
=
4
k
+
2;

−1, åñëè n = 4k + 3, ãäå k ∈ N ;
1
2
№ 30. 1) –64; 2) 256 + 256і; 3) 1; 4) 1; 5) +
i.
4 2
№ 31. 1) –3 + 4і; 2) 28 +96і; 3) –64 + 120і; 4) 676; 5) 3200і; 6) –4 – 28і;
7) (4 – а2) + 40і; 8) (а2 – 9b2) – 6abi; 9) (4c2 – 9d2) + 12cdi; 10) –4;
1
3
11) –4; 12) − −
i ; 13) –2 + 2i; 14) –10 – 9 3 ⋅i; 15) –24 3 ⋅i;
2 2
16) 2b(3a2 – b2)i; 17) x2 – 2i; 18) 64; 19) –1; 20) –1; 21) i; 22) –1.
№ 23. 1) 7 + 10і; 2) 204 + 66і; 3) 88 – 7і; 4) –66 – 101і; 5) −
93
1 1
1
1 3 7
1
− i ; 2) − i ; 3) 1; 4) +
i ; 5) 4; 6) .
2 2
32
2
2
8
 x = t;
№ 33. 1) (3; –1); 2) (2; –2); 3) (1; 1); 4) 
де t ∈ R; 5) ∅; 6) (1; 2);
y
=
−
2
t
,

№ 32. 1)
3 1
7) (2; 1);  ;  .
 2 2
№ 34. 1) (3; 1); 2) (4; 2); 3) (1; 2); 4) (2; 3); 5) (1; 2); 6) (2; –1); 7) (3; 4);
 x = t;

1 
де t ∈ Z ; 12) ∅.
8) (2; –2); 9) (3; 4); (2; 4); 10)  ; 0  ; 11) 
1
y
=
t
,
4 

2
 15 
№ 35. 1) (2; –1); 2)  0;  ; (1; 6); 3) (0; 0).
 2
№ 36. 1) (–4; 6); 2) (5; 2); (5; –2).
1
1
№ 37. 1) –4і; 4і; 2) 1 – і; 1 + і; 3) – 2 і; 2 і; 4) − − i ; + i .
2
2
5
5
1
3
№ 38. 1) − ⋅ i ;
⋅ i ; 2) 7 – 5і; 7 + 5і; 3) –1 – 2і; –1 + 2і; 4) −
⋅i ;
4 4
3
3
1
3
+
⋅ i ; 5) –9; 9; 6) –3; –1.
4 4
 3 2
№ 39. 1) (3 – 6і; 3 + 6і); (3 + 6і; 3 – 6і); 2) (–1; –1);  ;  .
 2 3
2
2
2
№ 40. 1) z – 4z + 5 = 0; 2) 8z – 12z + 5 = 0; 3) z – (5 – 3i)z + (4 – 7i) = 0;
4) 2z2 – (3 – i)z + (4 – 2i) = 0.
№ 41. 1) 3z2 + 2z + 27 = 0; 2) 4z2 – 24z + 37 = 0; 3) 2z2 – (3 – i)z + (4 – 2i) = 0.
№ 42. Комплексные.
№ 43. 1) z2 – 10z + 26 = 0; 2) z2 + 9 = 0; 3) z2 + 20z + 200 = 0; 4) z2 – 30z + 289 = 0;
5) 2z2 – 14z + 205 = 0.
№ 44. 1) 3 – 2і; 3 + 2і; 2) 2 – 2 і; 2 + 2 і.
5
№ 45. 1) а > ; 2) k = 8 + 6i.
4
Im z
№ 46.
Re z
z1
О
z2
Рисунок 31
z3
94
№ 47. 1) | z1 | = 3; |–z1| = 3; 2) | z 2 | = 5; |–z2| = 5; 3) arg z3 =
π
π
; arg (– z3 )= – ;
2
2
π
2π
; arg (– z 4 )=
; 5) arg z5 = π; arg (– z 5 )= 0;
3
3
2π
π
2π
π π
5π
6) arg z 6 =
; arg (– z 6 )= ; 7) –
< arg z 7 < – ; < arg (– z 7 ) <
.
3
3
3
6 3
6
4) arg z 4 =
№ 48.
Im z
1)
Im z
2)
х2 + у2 = 4
х2 + у2 = 4
Re z
Re z
О
О
2
Рисунок 32
Рисунок 33
Im z
Im z
х2 + у2 = 16
3)
ϕ = π/4
4)
Re z
О
О
4
Рисунок 35
Im z
О
π/4
Рисунок 36
Im z
2)
z
π/4
Re z
π/4
Рисунок 34
№ 49.
1)
3
х2 + у2 = 4
Re z
Re z
–2
О
2
Рисунок 37
95
Im z
3)
Im z
4)
х2 + у2 = 9
х2 + у2 = 9
Re z
О
Re z
О
3
Рисунок 39
Рисунок 38
Im z
х2 + у2 = 4
5)
3
2
6)
Im z
ϕ = π/3
2
х +у =9
Re z
О 2
Re z
π/3
3
О
Рисунок 41
Рисунок 40
Im z
7)
ϕ = π/6
π/6
О
Re z
ϕ=0
Рисунок 42
О
Рисунок 43
х=1
х=0
№ 50. 1) Искомое множество точек принадлежит окружности х2+у2=4.
2) Искомое множество точек находится между окружностями х2 + у2 = 1 и
х2 + у2 = 4.
№ 51. 1) Искомое множество точек находится на окружности х2 + (y + 1)2 = 1 и
на плоскости круга, ограниченного этой окружностью.
2) Искомое множество точек находится на плоскости круга, ограниченного
окружностью (х + 2)2 + у2 = 16, но самой окружности не принадлежит.
№ 52.
Im z
Im z
1)
2)
Re z
О
Re z
1
Рисунок 44
96
3)
Im z
4)
у=1
Im z
у=2
Re z
Re z
О
Рисунок 45
5)
Рисунок 46
О
Im z
х2 + у2 = 4
6)
Im z
–1
х2 + у2 = 1
х2 + у2 = 25
Re z
Re z
О 2
1
5
Рисунок 48
Рисунок 47
Im z
7)
О
у = –2
Im z
8)
ϕ = π/4
х2 + (у2 –1)2 = 1
2
Re z
Re z
1
О
1
2
О
Рисунок 50
Рисунок 49
Im z
х2 + у2 = 1
9)
х2 + у2 = 9
ϕ = 3π/4
10)
у =2
Im z 2
х + у2 = 4
4
х2 + у2 = 16
Re z
О
1
Рисунок 51
3
Re z
О
2
Рисунок 52
4
97
π
3π
i
π
π

№ 53. 1) z = 3  cos + i sin  = 3e 2 ; 2) z =
2
2

i
3π
3π 

2  cos + i sin  = 2 e 4 ;
4
4 

π
π
i
−i
  π
  π
 π 
 π 
3) z = 2 cos −  + i sin −   = 2 e 3 ; 4) z = 2 cos −  + i sin  −   = 2 e 6 ;
 3 
 6 
  3
  6
π
o
 π
 π  −i
5) z ≈ 5 ( cos126,9o + i sin126,9o ) = 5ei126,9 ; 6) z = cos −  + i sin −  = e 6 ;
 6
 6
o
7) z ≈ 13 cos(−67, 4o + i sin(−67, 4o ) = e− i 67,4 ;
(
)
(
)
o
8) z ≈ 5 cos ( −143,1o ) + i sin ( −143,1o ) = e −i143,1 .
π
i
π
π
№ 54. 1) z = cos + i sin = e 2 ; 2) z = 5(cos π + i sin π ) = 5e iπ ;
2
2
π
i
π
π

2  cos + i sin  = 2 e 4 ;
4
4

i0
3) z = 3(cos 0 + i sin 0 ) = 3e ; 4) z =
π
−i
  π
 π 
5) z = 6 2  cos −  + i sin −   = 6 2 e 4 ;
 4 
  4
5π
i
5π
5π 

6) z = 2 cos + i sin  = 2 e 6 ;
6
6 

2π
−i
  2π 
 2π  
7) z = 4 cos −  + i sin  −   = 4 e 3 ;
 3 
  3 
(
)
8) z ≈ 13 cos (146,3o ) + i sin(146,3o ) = 13ei146,3 .
№ 55. 1)
o
1
3
4 7 3
+
i ; 5) +
i;
2 2
2
2
3 + i; 2) 5; 3) –3і; 4)
9 2 9 3
+
i;
2
2
6)
7) 8 2 − 8 2 i .
π
π

№ 56. 1) 5 cos + i sin  ; 2) 6(cos 0 + i sin 0 ) ; 3) 11 cos ( −30o ) + i sin ( −30o ) .
4
4

(
№ 57. 1) 6 e
№ 58. 1) e
№ 59. 1) e
i105o
i
π
3;
i
π
3;
i
π
3
; 2) 4 e
i
π
3
i
π
3;
i 90o
; 3) 6 e
2) 3 e ; 3) 10 e
2) 8e
i
π
4
i
π
2;
π
12 ;
3) 6 2 e
№ 60. 1) 3e ; 2) 3e ; 3) e
i
i 60o
i
7π
12
)
3 −i 90o
e
.
2
4)
o
o
o
4) 4ei 90 ; 5) 4ei 90 ; 6) 8ei 45 .
; 4) 5 2 e
; 4) 2e
i
π
12
i
π
3.
; 5) e
−i
π
6
o
; 6) 2e −i 90 .
98
π
−i
 π
 π
№ 61. 1) cos −  + i sin −  = e 3 ; 2) 256(cos π + i sin π ) = 256e iπ ;
 3
 3
π
i
π
π

3) 11 cos(−60 ) + i sin(60 ) = e
; 4) 243 cos + i sin  = 243e 3 ;
3
3

5) 117649(cos 0 + i sin 0 ) = 117649e 0i ;
(
o
o
−60o i
)
6) 19683(cos( −π) + i sin( −π) ) = 19683e −iπ .
2
2
№ 62. 1) ; 2) ≈ 0,085 + 0,235і; 3) 15і; 4) –7 + 7 3 ⋅ i; 5) 2; 6) –1; 7) – ;
3
5
7
8) i; 9) ≈ 1,994 + 0,149і; 10) ≈ –0,489 – 0,674і.
5
2π
π
−i
i
π
π
 2π 
 2π 
50 
50 3
3
№ 63. 1) cos −  + i sin −  = e
; 2) 2  cos + i sin  = 2 e ;
3
3
 3 
 3 

3) cos 0 + i sin 0 = e 0i ; 4) 20736 ( cos0 + i sin 0 ) = −20736e0i ;
2π
i
2π
2π 

+ i sin  = 1,171e 3 .
5) 97,65625(cos 0 + i sin 0 ) = 97,65625e ; 6) 1,171 cos
3
3 

0i
(
o
)
№ 64. 1) 729 cos(−60o ) + i sin(−60o ) = 729e − i 60 ;
(
)
o
o
2) 16 cos60o + i sin 60o = 16ei 60 ; 3) cos 40o + i sin 40o = ei 40 ;
π
i
π
π
4) cos + i sin = e 2 ;
2
2
5) 64(cos 0 + i sin 0) = 64e i 0 ; 6) cos 0 + i sin 0 = e i 0 .
№ 66. 1) Равенство справедливо; 2) равенство справедливо; 3) равенство справедливо; 4) равенство справедливо.
1
3
1
3
№ 67. 1) + i
; –1;
−i
;
2
2
2
2
2
2
2
2
2
2
2
2
2) −
−i
;
+i
; −
+i
;
−i
;
2
2
2
2
2
2
2
2
6
2
6
2
6
2
6
2
3)
+i
;
−i
; –
+i
; –
−i
; i 2 ; –i 2 .
2
2
2
2
2
2
2
2
π
π
3π
3π 
  3π 


 3π  
№ 68. 1) 2 cos + i sin  ; 2 cos + i sin  ; 2 cos −  + i sin −   ;
4
4
4
4
 4 


  4
  π
 π 
2 cos −  + i sin  −   ;
 4 
  4
π
π
11π
11π  6   5π 
 5π  


2) 6 8  cos + i sin  ; 6 8  cos
+ i sin
 ; 8  cos −  + i sin −   ;
4
4
12
12 


 12  
  12 
(рис. 53);
99
Im z
3)
ω0
ω1
o
Re z
1,4
О
(
)
5 ( cos63, 43 ) + i sin 63, 43 ) ) ;
5 ( cos153,43 ) + i sin153,43 ) ) ;
5 ( cos(−116,56 ) + i sin(−116,56 ) ) ;
5 cos(−26,56o ) + i sin(−26,56o ) ;
o
o
o
o
ω2
o
4) cos(−27 o ) + i sin(−27 o ) ; cos 45o + i sin 45o ;
Рисунок 53
cos117 o + i sin 117 o ;
cos(−171o ) + i sin(−171o ) ;
cos(−99o ) + i sin(−99 o ) ; (рис. 54);
π
π
 19π 
 19π 
+ i sin ; cos −
 + i sin  −
.
20
20
 20 
 20 
Re z № 69 1) ω0 = cos 40° + і sin 40°;
ω1 = cos 160° + і sin 160°;
1,74
О
ω2 = cos (–80°) + і sin (–80°).
ω0
2) ω0 = 5 16 (cos(–24°) + і sin(–240°)) ≈
ω2
≈ 1,74(cos(–24°) + і sin(–240°));
ω4
ω1 = 5 16 (cos 48° + і sin 48°) ≈
Рисунок 54
≈ 1,74(cos 48° + і sin 48°);
Im z
ω2 = 5 16 (cos 120° + і sin 120°) ≈
ω1
ω2
≈ 1,74(cos 120° + і sin 120°);
ω3 = 5 16 (cos(–168°) + і sin(–168°)) ≈
ω0
Re z
≈ 1,74(cos(–168° + і sin(–168°));
1
О
ω4 = 5 16 (cos(–96°) + і sin(–96°)) ≈
≈ 1,74(cos(–96° + і sin(–96°)).
ω5
3) ω0 = cos 10° + і sin 10°;
ω4
ω1 = cos 70° + і sin 70°;
Рисунок 55
ω2 = cos 130° + і sin 130°;
ω3 = cos(–170°) + і sin(–170°);
ω 1 Im z
ω4 = cos(–110°) + і sin(–110°);
ω5 = cos(–50°) + і sin(–50°); (рис. 55).
Im z
ω2
ω3
ω3
5) cos
ω1
ω0
ω2
2
О
ω3
Рисунок 56
Re z
i
π
24 ;
i
13π
24 ;
−i
23π
24
−i
11π
24
4) 2 e
2e
2e
;
2e
;
(рис. 56).
№ 70. Указание. Пусть М1, М2, М3 – аффиксы соответственно комплексных чисел z1, z2, z3. Доказать,
что векторы М 1 М 2 и М 2 М 3 не являются коллинеарными.
100
CПИСОК РЕКОМЕНДУЕМОЙ ЛИТЕРАТУРЫ
1. Свешников А.Г., Тихонов А.Н. Теория функций комплексного переменного. – М.: Наука, 1974. − 320 с.
2. Лаврентьєв М.А., Шабат Б.В. Методы теории функций комплексного переменного. – М.: Наука, 1987. − 688 с.
3. Привалов И.И. Введение в теорию функций комплексного переменного. –
М.: Наука, 1984. − 432 с.
4. Маркушевич А.И. Краткий курс теории аналитических функций. − М.:
Наука, 1978. − 416 с.
5. Сидоров Ю.В., Федорюк М.В., Шабунин М.И. Лекции по теории функций комплексного переменного. − М.: Наука, 1976. − 408 с.
6. Маркушевич А.И., Маркушевич Л.А. Введение в теорию аналитических
функций. − М.: Просвещение, 1977. − 320 с.
7. Краснов М.Л., Киселев А.И., Макаренко Г.И. Функции комплексного
переменного. Операционное исчисление. Теория устойчивости (задачи и
упражнения). – М.: Наука, 1971 − 256 с.
8. Данко П.В., Попов А.Г., Кожевникова Т.Я. Высшая математика в упражнениях и задачах. − М.: Высшая школа, 2006. Ч.1. − 304 с.
9. Минорский В.П. Сборник задач по высшей математике. − М.: Наука, 1987.
− 351 с.
10. Шнейдер В.Е., Слуцкий А.И., Шумов А.С. Краткий курс высшей математики. Учебное пособие для втузов. − М.: Высшая школа, 1978.−
Т. 1− 384 с.
11. Писменный Д.Т. Конспект лекций по высшей математике. Ч.1. − М.: Айрис-пресс, 2005. − 256 с.
12. Фильчаков П.Ф. Справочник по высшей математике. − К.: Наукова думка,
1973. − 743 с
13. Богомолов М.В. Практические занятия по математике. – К.: Высшая школа, 2007. − 495 с.
14. Выгодский М.Я. Справочник по высшей математике. – 4-е вид. – М., 1973.
870 с.
Download